NEURO ACTUAL

Réussis tes devoirs et examens dès maintenant avec Quizwiz!

Entorhinal cortex belongs to archicortex and it is three layered

+

Epilepsy is mostly bound to temporal lobe

+

Epiphysis produces hormone melatonin

+

Existence of vomeronasal organ is proved in predatory mammals

+

Fossa rhomboidea is located on posterior part of brain stem

+

Free nerve endings carries sharp pain

+

Frontal eye field is located in frontal lobe

+

Gama motoneurons are located in ventral spinal horns

+

Gyrus precentralis belongs to neocortex

+

Hypothalamus is part of diencephalon

+

In Parkinson disease is impaired mostly sensori-motorick˝ circuit of basal ganglia

+

It DOES belong to medial row of motor nuclei on the bottom of 4th ventricle - n.III, n.IV, n.VI a n.XII

+

Lissauera zone is continuation of radix posterior of spinal nerve

+

Medial side of thalamus forms wall of 3rd ventricle

+

Medulla oblongata belongs to brain stem

+

Medulla spinalis is also anchored by filum terminale

+

Mesencephalon belongs to brain stem

+

Nc. albus is formed by neuronal fibers of nc. ruber

+

Nc. mesencephalicus V. contains pseudounipolar neurons for proprioception

+

Nc. suprachiasmaticus participates in regulation of sleep cycle

+

Neocortex is composed of six layers

+

Nucleus basalis Meynerti is part of substantia innominata

+

Nucleus ruber is part of mesencephalon

+

Oligodendrocytes are similar to Schwann cells on the periphery

+

Part of substantia nigra belongs to basal ganglia

+

Planum temporale belongs to asymmetrical part of CNS

+

Primary motor area is area 4

+

Pyramidal motor tract is 2-neuronal

+

Rapheal system of reticular formation is serotoninergic

+

Reticular formation continues into nc. reticularis thalami

+

Reticular formation runs through brain stem

+

Rexed zones describe spinal gray matter

+

Rombencephalon differentiates into metencephalon and myelencephalon

+

Schaffer collaterals are located in hippocampus

+

Schizophrenia research is focused on posterior part of cingulum

+

Sylvius fissure is synonym for fissura lateralis cerebri

+

Telencephalon has four lobes

+

Temporal lobe is located in middle skull fossa

+

Thalamus is part of diencephalon

+

Third brain ventricle is not paired

+

Ventral border of diencephalon to cortex is lamina terminalis

+

Ventral spinal horns do not reach to the spinal surface

+

Vertebromedullar topography is mapping projection of spinal cord segments on vertebral column

+

Visual cortex is supplied mostly via a.cerebri posterior

+

Weight of adult brain is cca 2 kg

+

White matter is formed by dendrites and axons

+

On spinal cord are located 2 intumescences

+ intumescences = swelling

Nuclei of posterior fasciculi are located on the transition of medulla spinalis and brain stem

+ posterior fasciculi= posterior column

Nucleus basalis Meynerti is cholinergic

+ Nucleus basalis of Meynert -> important in alzheimers

Prosencephalon differentiates into telencephalon and diencephalon

+ Prosencephalon - forbrain

Plexus choroideus DOES not passes from lateral ventricles into third ventricle

+ RR: it DOES pass (should be wrong here)

Fissura calcarina is located in occipital lobe

+ The calcarine sulcus is where the primary visual cortex (V1) is concentrated. Near occipital pole.

Fissura horizontalis cerebri separates left and right hemisphere

+ (??? cerebelli would be true)

Spinothalamic tract runs in posterior spinal fascicles

+ (???) RR: should be antero-laterally

Colliculi superiores are located on the posterior part of brain stem

+ (control reflex movements of eye, head, neck)

Neurons are visible by naked eye

-

Nuclei of cranial nerves are located only in mesencephalon

-

Nucleus dentatus of cerebellum is smallest of all cerebellar nuclei

-

Olfactory cortical area is located close to fissura calcarina

-

Pallidum ventrale is another name forr claustrum

-

Rat has also cortical gyrification, similarly to human

-

STRESS AND TRAUMA

-

Septum verum is larger in rat then in the human

-

Spinal cord is not covered by pia mater

-

Subfornical organ serves for parolfaktory perception

-

Sulcus posterior medullae spinalis is deeper then fissura mediana anterior

-

Synchronization of eyeball movement is regualted by fasciculus longitudinalis posterior (Schutzi)

-

Tectal plate is part of oblongata

-

Canalis centralis has diameter cca 2 cm

- 1,7

Medulla spinalis continues caudally even into canalis sacralis

- MA: Ends between L1 - L2

The epidural bleeding is a life-threatening intracranial bleeding often associated with a traumatic fissure of skull and a damage to bridging meningeal veins

False

The first neuron of the pain pathway is located in the ventral columns of spinal cord

False

Alexythymia is what?

A personality construct characterized by the sub-clinical inability to identify and describe emotions in the self

The hippocampus is an important structure for the regulation of visual pereception

False

The lifetime prevalence of depression ranges between 40-50%

False

The melatonine theory of depression and the theory of circadian rhythm disturbance led to the implementation of electroconvulsive therapy

False

The myasthenia gravis is characterized by irreversible muscle weakness

False

The occurrence of panic attack is predictable and thus it can be prevented by monitoring the early warning signs

False

The so-called rapid cycling in a bipolar disorder is defined by six or more distinct episodes of mood disorder (depression, mania, hypomania) per year

False

The syringomyelic dissociation syndrome may be caused by a metastasis involving a vertebral body

False

The uraemic encephalopathy develops in patients with a hepatic failure

False

The withdrawal of stimulants leads to somatic symptoms, such as pain the muscles and joints

False

There is an ipsilateral loss of pain and temperature below the lesion level in the Brown-Sequard syndrome

False

Though withdrawal is typical symptom of bipolar disorder?

False

A patient with hypochondria disorder complains about multiple, repeated and frequently changing somatic symptoms

False

A reduced protein concentration in the cerebrospinal fluid can be expected in a young paraparetic woman, diagnosed with multiple sclerosis

False

A sound field includes ultrasound and infrasound frequencies, too

False

A stereotactic surgery on the dorsal roots of the spinal cord is one of the most effective neurosurgical procedures for intractable pain

False

A subjective tinnitus manifests also as optical visions or hallucinations about its sources

False

A superficial sensation is tested with a tuning fork along with the vibration sense

False

A typical symptom of dementia in Alzheimer´s disease is an early speech disorders

False

Amygdala is located in parietal lobe

False

Amyloid neuropathy is most commonly associated with amyloid-beta peptide

False

An injury to the cervical enlargement may cause a central paresis of arms and a peripheral paresis of legs

False

Anisocoria in an unconscious patient may be due to a tentorial herniation (a temporal conus), which is contralateral to the mydriasis

False

Anosognosia - the abscence of insight into the disease is typical only for a delusional disorder

False

Antidepressants affect neither gene expression nor neurotrophic factors

False

Antidepressants of the SSRI group are preferred to those of SNRI group in the treatment of neuropathic pain

False

Common characteristic of classical and operant conditioning is association between conditioned and unconditioned stimuli

False

Compulsions and obsessions are symptoms of panic disorder

False

Corpus callosum is the smallest commissure of the brain

False

Dissociative convulsions are accompanied by urinary incontinence, tongue biting and loss of consciousness

False

Drugs similar to antiepileptics are used in the treatment of Alzheimer´s disease

False

In a unilateral cerebellar lesion, ataxia and adiadochokinesia present contralaterally

False

In bulbar syndrome, the central palsy of n. XII is present

False

In order to diagnose the bipolar disease, at least one epidose of depression must be present in addition to manic phases

False

Infarction of subthalamic nucleus causes ipsilateral hemiballism

False

Medications containing ginkgo can significantly alleviate migraine, especially in warfarinized patients

False

Metastatic brain tumours constitute 5-10% of all brain tumors

False

Metencephalon is developmentally youngest part of brain

False

Microscopic picture of Parkinson´s disease includes amyloid deposits in the brain vessel wall (congophilic angiopathy)

False

Muscle weakness, hypotonia and hyperreflexia of afferent limb present in peripheral paresis

False

Objective tinnitus is treated with vasodilators

False

Obsessions and phobias are formal thought disorders

False

PTSD patient remembers every detail of experienced traumatic event

False

Pain sensation is transmitted through the dorsal columns of the spinal cord

False

Paracetamol is used as an antiaggregans in the secondary prevention of ischaemic stroce recurrence

False

Paralysis, atonia, areflexia and a progressive muscle atrophy are signs of damage in syringomyelia

False

Patient with delirium have no problems to do the seven substraction test ?

False

Patients with aggressive or sexual obsessions are dangerous to others

False

Penumbra is the zone of brain tissue surrounding the intracerebral haemorrhage, which is endangered by a reperfusion injury

False

Peripheral muscle relaxants can positively influence the symptoms of Huntington´s disease

False

Plexus choroideus does not spread from lateral ventricles into the third ventricle

False

Primary CNS angiitis is diagnosed by notch-3-protein in skin biopsy

False

Radiculography can be used to diagnose osteochondrosis

False

SSRIs have more adverse effects as compared to tricyclic antidepressants

False

Schizophrenia is in medical terminology a synonym for the split personality disorder

False

Shrunk or "sinsemilla" stands for the hashish rich in cannabidiol (CBD)

False

Smell and touch are first processed in thalamus and then proceed to amygdala

False

So called "red neuron" is hallmark of acute intracerebral hemorrhage

False

The Duchenne muscular dystrophy is an autosomal dominant disease

False

The Guillain-Barre syndrome is often associated with parasitic infections

False

The astorcytic plaque of alpha-synuclein is characteristic for the corticobasal degeneration

False

The brainstem death may be comfrmed by extensor response of limbs to painful stimuli

False

The deviation of both eyes to one side is caused by the bilateral oculomotor palsy

False

Chorea is commonly found in the Creutzfeldt-Jakob disease

False (RW is found but not common, evidently)

A patient with amnestic (anomic) aphasia stumbles and says incorrect words

False (RW: Anomic aphasia (also known as dysnomia, nominal aphasia, and amnesic aphasia) is a mild, fluent type of aphasia where an individual has word *retrieval failures* and cannot express the words they want to say (particularly nouns and verbs). Anomia is a deficit of expressive language.)

The progressive multifocal leukoencephalopathy is caused by the JC virus of herpes family

False (RW: JC virus is a paloma virus)

Sorafenib is used to treat macular degeneration

False (RW: Nein, used for kidney, thyroid, other cancers)

If a tympanic membrane is perforated, the tympanometry yields type C tympanogram

False (RW: Type C suggests borderline problem - i think Type B would be correct http://www.utdallas.edu/~thib/rehabinfo/de.htm)

Capsaicine is a selective antagonist of TRPV1 receptors

False (RW: agonist!)

A patient with meningeal syndrome cannot retroflex his head and flex his knees

False (RW: anteroflex)

Tetrazepam is classified as a general anaesthetic

False (RW: benzo)

A finding of generalized spike-wave rhythm of EEG is typical for partial seizures

False (RW: i think they are looking for absence seizures?)

The experimental autoimmune encephalitis is a murine model of multiple sclerosis prepared by the immunization of experimental animals by glial fibrillary acidic protein

False (RW: looking for mylein basic protein?)

The Marburg form of multiple sclerosis is a relatively slowly progressive disease with a good prognosis

False (RW: nooo, quick progression to death)

The paleocerebellar syndrome includes limb ataxia and dysdiadochokinesia

False (RW: paleo: truncal ataxia, postural instability / neocerebellar- dysdiadochokinesia

The gemistocytic astrocytoma is a typical form of astrocytoma of childhood

False (RW: pilocytic)

The herpetic encephalitis is characterized by the presence of intracytoplasmatic inclusions in glial cells

False (RW? http://emedicine.medscape.com/article/341142-overview)

The SMA gene is affected in the Werdnig-Hoffmann spinal muscular atrophy

False (SMN 1 gene -RW)

The direction of nystagmus is described in terms of the slow component

False (fast component rw)

The absence of gliosis supports the hypothesis of the neurodegenerative origin of schizophrenia

False (gliosis --> Neuro degen)

Patients with anorexia nervosa have a specific phobia of food

False (im just vain)

Myasthenia gravis is often associated with a thyroid cancer

False (it's thymus! not thyroid)

Disorders of protein degradation associated with the ubiquitin system are typical for Alzheimer´s disease

False (not 'typical)

Triptans stimulate 5-HT3 receptors

False (rw 5Ht1B and 5HT1D)

Donepezil can be used as an anxiolytic

False (rw alz drug)

Illusions and pseudoillusions are thought disturbances

False (rw perception)

Flumazenil is an inhibitor of catechol-O-methyltransferase (COMT), acting against decomposition of dopamine

False (rw reverses benzos)

The hamartin peptides takes part in the pathophysiology of neurodegeneration

False (rw-this is a tuberous sclerosis protein - nthing to do with neuro degen)

Primary natural rewards

Food Water Sexual contact Fresh air

Mesolimbic dopaminergic pathway

From VTA to nucleus accumbers, amygdala and hippocampus

Mesocortical dopaminergic pathwaay

From VTA to prefrontal cortex (cognitive functions)

Baclofen is an GABA-B receptor antagonist?

No, presynaptic gaba-b agonist

Social phobia is a fear of social interaction without vegetative symptoms?

No, there are vegetative symptoms

Mouse models of MS

Injection of myelin basic protein may induce a similar as MS disease in mice. (pp notes, only thing about mice)

Idiognosia (Self-awareness) is typical only for patients with schizophrenia?

No

In parkinson's disease "alien hand syndrome" is a sign?

No

Is Balo-concentric sclerosis realted to cancer and alcoholism?

No

Ischemia and hypoxia is the same?

No

Lesions of cerebellum appears on the opposite side of the body?

No

Liquorrhea is typical for tinnitus?

No

Medulloblastoma is linked to the oncogene c-ras?

No

Posttraumatic stress reactions starts immediately after the stress?

No

Triptans are safe during pregnancy?

No

Are Clopidogrel and Plavix thrombolytic drugs?

No, they are antiplatelet

Crow divided schizophrenia into 2 types, based on the numbers of attacks?

No, they were based on positive and negative symptoms, ventricular size and response to therapy.

What is the Mechanism of Alteplase?

It is a SERINE PROTEASE that activates PLASMINOGEN to plasmin to degrade fibrin clots

In dermatomyositis there is increase of CD8+ Tcells ?

No, CD4+ T-cells RW (In polymyositis there is increase in CD8+ T-cells)

Subdural hematoma is assessed by x-ray of the skull?

No, CT

Mood is long term and unchangable?

No, longterm but not unchangeable

When a patient is in complete coma is GCS must be 0 ?

No, minimum is 3

In polymyositis there are B-cells ?

No, mostly CD8+ Tcell

Ictus are always hemorrhagic ?

No, only 20% rest is ischemic

Lesion of cerebellopontine angle causes central lesion of VII?

No, peripheral lesion Other tests say it shuld be CN VIII (5,7,8 th nerve goes out through cerebellopontine angle)

derealisation or depersonalisation, fear of losing control, going insane, dying are typical symptoms of what?

Panic attack

Most important structures for working memory

Prefrontal cortex Parietal cortex, hippocampus, thalamus, basal ganglia

Healthy brain development - imporant components

Proliferation Pruning Myelinization

Accidentally transmitted CJD

Progressive *cerebellar syndrome* after recieving hormones from cadaverous pitutary glands, dura mater grafts, corneas, or neurosurgical operations

What does JC polyoma virus cause?

Progressive multifocal leukoencephalitis A disease commonly affecting immunocompromised. Affect white matter. Death within 6 months

What is the mechanism of action of amphetamine?

Release of -> DA -> NA -> 5-HT into synaptic cleft

What is the function of metamphetamines?

Release of DA, NA and serotonin

How long dose acute stress reaction last?

Symptoms are gone within 48h

What is the difference between temperament and character?

Temperament is inherited, whereas character is an acquired component of personality.

Hyperreflexia, hypertonia, positive pyramidal signs and muscle weakness are evident on the affected limb in a central (spastic) paresis

True

Hypoglycaemia may lead to epileptiform seizures

True

In a conus medullaris lesion, a retention of urine and a faecal incontinence may develop

True

In case of a central disorder of oculomotor innervation, conjugated gaze movements fail and the patient has no diplopia

True

In central hemiparesis, the abdominal skin reflexes are low or absent on the same side

True

Quick switches from one unrelated idea to another (flight of ideas) are typical for mania?

True

Rise in cerebral blood flow is expected with paCO2= 8 kPA

True

SCIWORA is accompanied by a positive neurological findings

True

Selegiline is also used as an antidepressant

True

Signs of cerebellar dysfunction: scanning speech, nystagmus, macrographia

True

Subdural hematoma can cause a change of pupillary diameter

True

Subdural hematoma: shows crescent shape on transversal CT scan

True

Symptoms of peripheral palsy accompanies damage to peripheral nerves

True

The uraemic neuropathy typically presents as a distal axonal sensomotoric polyneuropathy

True

The ventral tegmental area, nucleus accumbens and prefrontal cortex are all parts of the reward system

True

Tizanidin is an alpha2 presynaptic CNS receptor agonist, whereby it lowers the BP

True

To assess the presence of delusions, we ask the following question: Is there anything strange or unusual going on around you?

True

Triptans stimulate 5-HT1B receptors

True

Broadmann area 4 refers to primary motor cortex?

Yes

Bulbospinal atrophy is the same as Kennedy disease?

Yes

Buspiron used for anxiety attacks?

Yes

Can triptans cause strokes?

Yes

Can you find Cowdry type A inclusions in herpetic encephalitis?

Yes

Can you have an epileptic seizure without having epilepsy?

Yes

Citalopram is an SSRI ?

Yes

Decortication is due to a lesion above mesencephalon?

Yes

Do we have hair cells in cochlea?

Yes

Does central pontine demyelination (Osmotic demyelination syndrome) occur in diabetics?

Yes

Does hypothyreosis cause depression?

Yes

Does memantin improve the short term memory?

Yes

Drug seeking and craving is connected to dopamine, prefrontal cortex and amygdala?

Yes

For acute MS we can use methylprednisolon?

Yes

Frontotemporal dementia belong among ubiquitinopathies?

Yes

Hypochondrial disorders, the patient is sure he is sick?

Yes

In glioblastoma multiforme there is alot on anaplasia and atypical nuclei ?

Yes

Is Marburg type of "dementia" the fulborne type? Should be Marburg type of MS, not dementia.. ?!?!?

Yes

Is atropin a mydriatic agent?

Yes

Is dementia associated with deposition of beta protein structure ?

Yes

Is myasthenia gravis related to thymic cancer?

Yes

Is spike and wave pattern on EEG typical of absence seizure?

Yes

Lesion of the basal ganglia leads to a static tremor? (is this a resting tremor)

Yes

Lithium has a very narrow therapeutic window?

Yes

Marihuana can cause hallucinations and paranoia?

Yes

Methadone increases the QT interval.(in test)

Yes

Somatization syndrome is connected in medicinal unexplained syndrome? (Munchausen)

Yes

Subarachonideal bleeding is between pia and arachnoidea

Yes

Subjective effect is more important than objective results of tonometry?

Yes

Sumitriptan causes vasoconstriction in the brain?

Yes

Tauprotein is hyperphosphorylated ?

Yes

Vascular type of dementia is rapidly progrsesing

Yes

Visceral pain can cause vegetative changes?

Yes

Wernicke Korsakoff syndrome is due to a lesion of mammillary bodies?

Yes

i.v. drug addict need lemon juice (acid), lighter and opioids to shoot themselves?

Yes

What is PML?

a rare disorder that damages the material (myelin) that covers and protects nerves in the white matter of the brain.

Effect of stress on the mental health

anxiety, anger, rage, apathy, depression, cognitive dysfunction

- The lifetime prevalence of depression is between 40-50%

false, 15%

Limbic system

group of structures that control emotions, motivation and behavior and vegetative functions

Mechanism of action of Buspiron?

i. 5-HT1A receptor agonist for *anxiety attacks.* ii. NO hypnotic effects (compared to traditional anxiolytic/hypnotic drugs: BZP and Barbiturates) iii. Needs 3 weeks to effect (like antidepressants) so in case that ACUTE treatment is required you have to use BZP that will act within 30 min!!

Brachial plexus lesion can lead to both motor and sensoric damage?

yes

Can N. meningitis cause purulent meningitis?

yes

Can a meningeoma cause meningeal signs?

yes

Can hypercalcemia cause encephalopathy?

yes

Can we find Lewy bodies in parkinsons?

yes

Contrast substance is applied intrathecal through lumbal puncture or suboccipital?

yes

Corticosteroids are used in acute relapse of MS?

yes

Does ALS attack both UMN and LMN ?

yes

Does hyponatremia cause encephalopathy?

yes

For addiction is the most important dopamine and glutamate?

yes

Frovatriptane has the longest t1/2 of the triptans?

yes

Hypernatremia can cause encephalopathy?

yes

Hyponatremia can cause brain edema?

yes

In Alzheimers disease the cholinergic neurons are diseased ?

yes

In Devic's disease there is Ig against aquaporins?

yes

In strong meningeal irrtiation the patient bends is knees if you flex his head to sternum?

yes

Is Huntington an AD disease?

yes

Is a depressed person most pessimistic in the morning?

yes

Is depression linked to increased inflammatory markers ?

yes

Lesion of cortex in occipital lobe will lead to contralateral homonymous hemianopsia?

yes

Levodopa should be prescribed with an inhibitor of peripheral decarboxylase ?

yes

MS affects both spine and brain ?

yes

Myasthenia gravis is an autoimmune disease?

yes

PCP, ketamin, amphetamin can cause psychosis?

yes

PML is caused by the JC virus in immunologicall compromised patients?

yes

Parkinson patients salivate more?

yes

Phobia - linked to situations or subjects that are normally not dangerous?

yes

Progressive supranuclear palsy belongs to tauopathies?

yes

Progressive supranuclear palsy includes atypical parkinsonian syndrome?

yes

Prostaglandins analogues increase the uveoscleral outflow?

yes

Schwannoma is found most commonly on CN VIII?

yes

Speech is a symbolic function?

yes

Stimulants decrease appetite, make people excited and increases self esteem ?

yes

Triptans can be used in combination with prokinetics ?

yes

Unilateral occipital lesion --> Contralateral homonymous hemianopsia?

yes

Water intoxication can cause coma?

yes

We can use NSAIDs in a low intensity migraine with pain

yes

Huntington is an AD disorder?

yes "ADHD"

Can a major depression be superimposed on dysthmia?

yes (Dysthymia - chronic depression)

Cerebellar gyri are named folia

+

Cerebellum is located on dorsal part of brain stem

+

Conditioned taste aversion travels also via nc. parabrachialis

+

Most neurons regenerate well

-

N. XI is faintest cranial nerve

-

Megalomanic delusions are present in mania and in depressions?

"Grandeous delusions" No

Is half life of Zolpidem 2-4h ?

(2-3h) so, yes

Alzheimer vs frontotemporal dementia vs subcortical dementia - manifestations

*Alzheimers disease* = memory dysfunctions, aphasia, apraxia, acalculia, agnosia *Frontotemporal dementia* = early changes in personality, disinhibition *Subcortical dementia* = bradypsychism, executive dysfunction

What is cataplexy?

(strong emotion or laughter causes a person to suffer sudden physical collapse though remaining conscious, a sudden and transient episode of muscle weakness)

Examples of motor neuron disorders

*ALS* Primary lateral sclerosis Spinal muscular atrophy

What is bulbar syndrome? Symptoms?

*Bilateral affection* of the lower IX-XII cranial nerves or their nuclei in medulla oblongata 1. LMN - peripheral palsy 2. Dysarthria, dysphagia 3. Absent masseter reflex 4. Tongue atrophy, fasciculations

Regional (local) ischemia

*Brain infarct* - encephalomalacia (white or red) Size and extension depend on site and speed of occlusion/thrombosis Final result is postnecrotic (postmalatic) pseudocyst

Procedural learning - important structures

*Cerebellum* - closed-loop learning with visual feedback *Basal ganglia* - open-loop learning, impaired in PD and Huntingtons (motor planning is necessary) *Motor cortex* - movements autmation With advancing practice - brain activity decreases and moves into performing areas of the brain (e.g. motor cortex)

Typing of primary lesions - types of tumors

*Gliomas* • Astrocytary • Oligodendroglial • Ependymal *Neuronal tumours* *Poorly differentiated tumors* *Meningiomas* Other • Pinealomas, germinal tumours • Nerve sheat tumours • Primary lymphomas of CNS

Danger in global ischemia

*Global edema* - increased intracranial pressure and blocked perfusion --> "coma vigile", brain death, autolysis, or "respiratory brain"

Examples of inflammatory neuropathies

*Guillain-Barre syndrome* - segmental demyelination with relative sparing of axons *Vasculitic neuropathy* - e.g. polyarteritis nodosa, *Infectious polyneuropathies* - diphteria toxin, Varicella, HIV, Leprosy (Hansen disease)

Processes included in implicit memory

*Habituation, reflexes* - no association, only one stimulus *Priming* - perceptual learning (automated recognition), a stimulus affects the repsonse to a second stimulus *Procedural/sensimotor learning* - automated skills obtained by training, basal ganglia *Emotional memory* - fear conditioning, amygdala *Conditioning* - associative learning

Examples of hereditary neuropathies

*Hereditary motor and sensory neuropathies (HMSNs)* - Charcot-Marie *Familial amyloid polyneuropathies* - Amyloid deposition in small vein in the epineural space

Examples of trinucleotide repeat disorders

*Huntingtons disease* Spinocerebellar ataxias Friedreich´s ataxia Kennedy´s disease (SBMA) Dentirubral-pallidoluysian atrophy

Causes of spinal cord infarcts

*Local compression* - spine injuries, disc herniation, tumors *Multiple arterial occlusions* - aortal dissection, air embolisms Maximum decomposition centromedullary (grey matter)

Canalis centralis continues cranially to the 4th ventricle

+

Tumors from oligodendrocytes

*Oligodendroglioma* - grade II - adults, white matter of telencephalon - well-circumscribed, gray, cystic tumor with fried egg cells with perinuclear halo - CALCIFICATIONS - *psammoma bodies*, low mitotic rate *Anaplastic oligodendroglioma* - grade III

Examples of synucleinopathies

*Parkinson´s disease* *Dementia with Lewy bodies* Multiple system atrophy (MSA P and C)

Non-infectious inflammatory myopathies

*Polymyositis* *Dermatomyositis* *Inclusion body myositis (IBM)*

10. choose the right statement a) For right-handed is dominant the right cerebellar hemisphere b) Patient with paleocerebellar syndrome staggers and falls with eyes opened c) During examination taxe (ataxic? Gate?) patient with cerebellar syndrome will not perform the whole bending, but will stop d) Adiadochokinesia is a disorder of the mutual coordination of limbs

*b)* Patient with paleocerebellar syndrome staggers and falls with eyes opened *c)* During examination taxe (ataxic? Gate?) patient with cerebellar syndrome will not perform the whole bending, but will stop...???? a) Right-handed pat. has ipsilateral dominant cerebellar hemisphere.

Alfa motoneurons of medulla spinalis are located in ventral spinal horns

+

Andersen circuit is internal connection inside hippocampal formation

+

Arachnoidea is located between dura mater and pia mater

+

Basal ganglia participates mostly in movement regulation

+

Brain stem is seat of vital reflexes

+

Brain stem runs through foramen magnum

+

Brodmann areas serves to brain cortex mapping

+

CSF is produced by plexus choroideus

+

Lumbal puncture we do at the level L2/L3 vertebras

+ L3/4 or L4/L5 according to essential general neurology (spinal cord can terminate around L2 so it is safer to do it lower). TH

Periakveductal gray carries nociceptive impulses

+(NO IT DOES NOT) TH: Looks like it does? https://en.wikipedia.org/wiki/Periaqueductal_gray

Chiasma opticum is located behind infundibulum hypophysis

+(in front not behind)

Amygdala is located in parietal lobe

-

Biggest neurons in humans are pseudounipolar by type

-

Blood from cerebrospinal sinuses flows mostly through plexus pterygoideus

-

Broca speech center is located in the parietal lobe

-

CFS volume is cca 4 liters

-

CSF flows fyziologically into inner ear

-

Callosal body is smallest commissure of the brain

-

Centromedial nucleus of amygdala developmentally originates in cerebral cortex

-

Cerebellum is located in middle skull fossa

-

Cingulum contains gyrus paraterminalis

-

Climbing fibers from oliva inferior into cerebellum run ipsilaterally

-

Cortical auditory area is located in frontal lobe

-

Dura mater is not sensitively innervated

-

Fibers from nc. accumbens continue into septal nuclei as stria terminalis

-

Foramina Luschkae are located on calva

-

Fornix runs with hippocampus as far as into amygdala

-

Grey matter is located on the surface od medulla spinalis

-

Hypophysis is located in tegmen tympani

-

Insula is located on medial part of hemisphere

-

Lemniscus lateralis carries ingormations via corpus trapezoideum

-

Lesion of Wernicke center is called expressive aphasia

-

Lesion off temporal lobe leads to behavior impairment

-

Metencephalon is developmentally youngest part of brain

-

Mossy fibers runs only from brain stem

-

Corpora mammillaria are part of brain stem

- MA: Mammillary bodies (only part of hypothalamus that is visible) - mammilary nuclei, part of the limbic system.

Capsula extrema is located between claustrum and putamen

- MA: external capsule does?

Gyrus parahippocampalis belongs to limbic system

- RR: It DOES belong to the limbic system

Prefrontal cortex is place of spatial memory formation

- hippocampus

Decussatio pyramidum is located on dorsal part of brain stem

- ventral

Dura mater is located externally to pia mater

- It is external in relation to pia mater, but maybe the question means it is not DIRECTLY EXTERNAL to the pia mater? TH

Corpus geniculatum laterale is auditory

- MA: LGB - lateral geniculate body - visual from optic tract MGB - medial geniculate body - auditory from inf. colliculus

Astrocytes belong to macroglia

- (??? macroglia=astrocytes +oligodendrocytes)

Axons carry information from neuronal body to the periphery

- (???)

Examples of tauopathies

- *Frontotemporal Dementia* and Parkinsonism Linked to Chromosome 17 (FTDP-17T) - Progressive supranuclear palsy (Steele- Richardson-Olszewski Disease) - Corticobasal degeneration - Pick´s disease - Argyrophilic grain disease

Helmintes - 2 examples

- *Taenia solium* (pork tapeworm) --> cysticericosis, larval stage infect muscles and CNS - *Echinococcus granulosus* (dogs) --> echinococcosis, hydatid disease, formation of cysts in CNS

Protozoa - examples

- *Toxoplasma gondii* (cats) --> oocyst, formation of enveloped cyst in CNS, NB! pregnancy, IS (AIDS, malignancies, transplant) - *Plasmodium* - *Trypanosoma* - *Amoebas* - Naegleria fowleri (water) - via olfactory nerve -> acute purulent meningoencephalitis (fulminant)

Chlorpromazin

- 1st generation antipsychotic

Haloperidol (Haldol)

- 1st generation antipsychotic - Competitive blocking of dopamine D2 receptors - More likely to be associated with movement disorders known as extrapyramidal symptoms

Risperidon

- 2nd generation antipsychotic - Blocks 5-HT2A receptors to a greater extent than it does D2 receptors - Approved for the management of disruptive behavior and irritability secondary to autism - Long-acting injectable (LAI) formulations of antipsychotics

Clozapine

- 2nd generation antipsychotics - high affinity for D1, D4, 5-HT2, muscarinic, and α-adrenergic receptors, but it is also a weak dopamine D2 receptor antagonist

Neurofibromatosis type 2

- AD - bilateral schwannoma of CN 8, multiple meningiomas, ependymomas of spinal cord NB! NO neurofibromas!

Von Hippel-Lindeu disease

- AD - capillary hemangioblastomas of cerebellum, retina, medulla oblongata and spinal cord - cysts in pancreas and liver, kidney carcinoma

Tuberous sclerosis

- AD - cortical hamartomas, subependymal giant cell astrocytoma - angiolipoma (kidney), rhabdomyoma (heart), retinal hamartomas, lymphangioleiomatosis (lung)

Neurofibromatosis type 1

- AD, more common than type 2 - neurofibromas, optical glioma, Lisch nodues (iris) and skin cafe-au-lait spots

What is Clopidogrel?

- ANTIPLATELET drug.

What is Dipyridamole?

- ANTIPLATELET drug.

What are the adverse effects of Clopidogrel?

- Abdominal pain. - Nausea.

What is Donepezil?

- AchE Inhibitor. - To treat mild to moderate Alzheimer's disease. - Do NOT give in pregnant/ breast-feeding women, in COPD/asthma pts and in pts with cardiovascular disease.

Complete brain ischemia

- In somatic death - Irreversible and involve the entire brain - Connected with ongoing autolysis

Capsaicin

- Active component of chili peppers - Relieves pain - Works by first stimulating and then decreasing the intensity of pain signals in the body

What are the indications for Biperiden?

- Acute drug induced dystonias - Treating Parkinsonism - Also used to reduce sweat in methadone users

PML - clinical

- Affects IS patients --> dementia, weakness, visual loss, ataxia - Death often within 6 months

What is Alteplase?

- Alteplase, aktivator av det FIBROLYTISKE SYSTEMET, fremstilt ved genetikk. - Brukes ved oppløsing av blodpropp bl.a ved hjerteinfarkt

Mesalazine

- Aminosalicylate anti-inflammatory drug used to treat inflammatory bowel disease, including ulcerative colitis, inflamed anus or rectum, and to maintain remission in Crohn's disease - Side effects are primary gastrointestinal but may include headache; GI effects include nausea, diarrhoea and abdominal pain

Nicergoline

- An ergot derivative used to treat senile dementia and other disorders with vascular origins - It has been found to increase mental agility and enhance clarity and perception - It decreases vascular resistance and increases arterial blood flow in the brain, improving the utilization of oxygen and glucose by brain cells - It is used for vascular disorders such as cerebral thrombosis and atherosclerosis, arterial blockages in the limbs, Raynaud's disease, vascular migraines, and retinopathy - Acute and chronic cerebral metabolic-vascular disorders (cerebral arteriosclerosis, thrombosis and cerebral embolism, transitory cerebral ischaemia). Acute and chronic peripheral metabolic-vascular disorders (organic and functional arteriopathies of the limbs), Raynaud's disease and other syndromes caused by altered peripheral irrigation - Migraines of vascular origin - Coadjutant therapy in clinical situations accompanied by platelet hyper-aggregability, arterial tension - Corio-retinal vascular disorders: diabetic retinopathy, macular degeneration and retinal angiosclerosis - Oto-vestibular problems of a vascular nature: dizziness, auditory hallucinations, hypoacusis

What is the function of Capsaicin?

- Analgesic derived from chilli-plants - Probably acts by Depleting Neurotransmitters like Substance P within tissues.

Diffuse axonal damage

- Angular acceleration of the head (traffic accidents) - Multiple mechanical disruption of axons, swelling of their ends - Unconsciousness (long duration), desorientation, focal neurological symptoms - Recovery - needs very long time, may be incomplete - posttraumatic dementia in the most severe cases - Another morphologic (microscopical) development: proliferation of microglia, degeneration of the involved tracts (different severity from case to case)

Memantine

- Anti-Alzheimer drug - NMDA receptor antagonist indicated for moderate to severe Alzheimer's disease - It acts by blocking the NMDA receptor and limiting Ca2+ influx into the neuron, such that toxic intracellular levels are not achieved - Memantine is well tolerated, with few dose-dependent adverse events - Expected side effects, such as confusion, agitation, and restlessness, are indistinguishable from the symptoms of Alzheimer's disease - Given its different mechanism of action and possible neuroprotective effects, memantine is often given in combination with an AChE inhibitor

Galantamine

- Anti-Alzheimer drug - Reversible AChE inhibitor

Natalizumab

- Anti-Multiple Sclerosis drug - Is a monoclonal antibody indicated for MS in patients who have failed first-line therapies

Amantadine

- Anti-Parkinson & antiviral drug - It increasing the release of dopamine, blocking cholinergic receptors, and inhibiting NMDA type of glutamate receptor - Action at NMDA receptors is the primary action at therapeutic concentrations - May cause restlessness, agitation, confusion, and hallucinations, and, at high doses, it may induce acute toxic psychosis. Orthostatic hypotension, urinary retention, peripheral edema, and dry mouth also may occur

Levodopa

- Anti-Parkinson drug - Metabolic precursor of dopamine. It restores dopaminergic neurotransmission in the neostriatum by enhancing the synthesis of dopamine in the surviving neurons of the substantia nigra. In early disease, the number of residual dopaminergic neurons in the substantia nigra is adequate for conversion of levodopa to dopamine. - The effects of levodopa on the CNS can be greatly enhanced by coadministering carbidopa, a dopamine decarboxylase inhibitor that does not cross the blood-brain barrier - Dopamine does not cross the blood-brain barrier, but its immediate precursor, levodopa, is actively transported into the CNS and converted to dopamine - It decreases rigidity, tremors, and other symptoms of parkinsonism - Half-life is 1-2 hr. - Side effects: anorexia, nausea, and vomiting, tachycardia and ventricular extrasystoles, hypotension, visual and auditory hallucinations and abnormal involuntary movements (dyskinesias) may occur

Ropinirole

- Anti-Parkinson drug - Nonergot dopamine agonist that is approved for the treatment of Parkinson's disease - Orally active agents - Half-life 6 hr.

Tolcapon

- Anti-Parkinson drug - Selectively and reversibly inhibit COMT - Long duration of action

NEURODEGENERATIVE DISEASES

-->

Biperiden (Akineton)

- Anti-Parkinson drug - Used for the adjunctive treatment of all forms of Parkinson's disease and for reduced sweating in methadone users - Relieves muscle rigidity, reduces abnormal sweating related with clozapine and methadone use and salivation, improves abnormal gait, and to lesser extent, tremor - Side effects: drowsiness, vertigo, headache, and dizziness. Blurred vision, dry mouth, impaired sweating. Mydriasis with or without photophobia

Carbidopa

- Anti-Parkinson drugs - dopamine decarboxylase inhibitor, diminishes the metabolism of levodopa in the periphery, thereby increasing the availability of levodopa to the CNS - The addition of carbidopa lowers the dose of levodopa needed by four- to fivefold and, consequently, decreases the severity of the side effects arising from peripherally formed dopamine

Betahistine

- Anti-vertigo drug - Commonly prescribed for balance disorders or to alleviate vertigo symptoms associated with Ménière's disease - Side effects: headache, low level of gastric side effects, insomnia

Dutasteride

- Antiandrogens - Counter male hormonal action by interfering with the synthesis of androgens or by blocking their receptors - Inhibit 5α-reductase resulting in decreased formation of dihydrotestosterone - Used for the treatment of benign prostatic hyperplasia

Chlorhexidine

- Antibacterial used as an antiseptic and for other applications - Used in disinfectants (disinfection of the skin and hands), cosmetics (additive to creams, toothpaste, deodorants, and antiperspirants), and pharmaceutical products (preservative in eye drops, active substance in wound dressings and antiseptic mouthwashes

Atropine

- Anticholinergic drug - Block muscarinic receptors, causing inhibition of muscarinic functions. In addition, these drugs block the few exceptional sympathetic neurons that are cholinergic, such as those innervating the salivary and sweat glands - Actions: mydriasis, cycloplegia in the eyes, antispasmodic in GIT, xerostomia etc. - May cause dry mouth, blurred vision, "sandy eyes," tachycardia, urinary retention, and constipation. Effects on the CNS include restlessness, confusion, hallucinations, and delirium, which may progress to depression, collapse of the circulatory and respiratory systems, and death.

Benzatropine

- Anticholinergic drug - Useful as adjuncts with other antiparkinsonian agents to treat Parkinson's disease and other types of parkinsonian syndromes, including antipsychoticinduced extrapyramidal symptoms - Can induce mood changes and produce xerostomia (dryness of the mouth), constipation, and visual problems typical of muscarinic blockers - They interfere with gastrointestinal peristalsis and are contraindicated in patients with glaucoma, prostatic hyperplasia, or pyloric stenosis

Homatropine

- Anticholinergic medication that is an antagonist at muscarinic acetylcholine receptors and thus the parasympathetic nervous system - Also given as an atropine substitute given to reverse the muscarinic and CNS effects associated with indirect cholinomimetic (anti-AChase) administration - Used before eye examinations (e.g., refraction), before and after certain eye surgeries, and to treat certain eye conditions

What is Benzatropin?

- Anticholinergic selctive for M1 receptor - Is an ANTICHOLINERGIC drug used in the treatment of Parkinson´s disease, Parkinsonism, and dystonia.

Warfarin

- Anticoagulant - Used in the prevention and treatment of DVT and PE, stroke prevention, stroke prevention in the setting of atrial fibrillation and/or prosthetic heart valves, protein C and S deficiency, and antiphospholipid syndrome - The principal adverse effect of warfarin is hemorrhage

Vigabatrin

- Anticonvulsant - Acts as an irreversible inhibitor of γ-aminobutyric acid transaminase (GABA-T) - GABA-T is the enzyme responsible for metabolism of GABA. Vigabatrin is associated with visual field loss ranging from mild to severe in 30% or more of patients

Gabapentin

- Anticonvulsant - Analog of GABA. However, it does not act at GABA receptors, enhance GABA actions or convert to GABA - It is approved as adjunct therapy for focal seizures and treatment of postherpetic neuralgia - May also be a good choice for the older patient because there are few drug interactions

Phenytoin

- Anticonvulsant - Blocks Na+ channels - Side effects: gingival hyperplasia, confusion, slurred speech, double vision, ataxia, sedation, dizziness, and hirsutism. Stevens-Johnson syndrome—potentially life threatening - Not recommended for chronic use - Primary treatment for status epilepticus (fosphenytoin)

Lamotrigine

- Anticonvulsant - Blocks sodium channels, as well as high voltage-dependent calcium channels -Effective in a wide variety of seizure types, including focal, generalized, absence seizures, and Lennox-Gastaut syndrome. It is also used to treat bipolar disorder

Topiramate

- Anticonvulsant - It blocks voltage-dependent sodium channels, reduces high-voltage calcium currents, is a carbonic anhydrase inhibitor, and may act at glutamate (NMDA) sites - Effective for use in partial and primary generalized epilepsy. It is also approved for prevention of migraine - It inhibits CYP2C19 and is induced by phenytoin and carbamazepine - Adverse effects include somnolence, weight loss, and paresthesias. Renal stones, glaucoma, oligohidrosis (decreased sweating), and hyperthermia have also been reported

Primidone

- Anticonvulsant - The primary mechanism of action of phenobarbital is enhancement of the inhibitory effects of GABA-mediated neurons - Primidone is metabolized to phenobarbital (major) and phenylethylmalonamide, both with anticonvulsant activity - Phenobarbital is used primarily in the treatment of status epilepticus when other agents fail

Ethosuximide

- Anticonvulsant (anti-epileptic) - Reduces propagation of abnormal electrical activity in the brain, most likely by inhibiting T-type calcium channels - It is only effective in treating absence seizures - Side effects: drowsiness, hyperactivity, nausea, sedation, GI upset, weight gain, lethargy, SLE, and rash. Blood dyscrasias can occur; periodic CBCs should be done. Abrupt discontinuance of drug may cause seizures

Global ischemia

- In the clinical death, severe heart arrhythmia, hypotension or shock ("low flow" states) - Most vulnerable neurons are affected first

What is Carbamazepin?

- Anticonvulsant and mood stabilizer (primarily used in eoulepsy and bipolar disease as weel as trigeminal neuralgia) Functions to STABILIZE the membrane by: - Openning the K+ channels. - Prolonging the time Na+ channels are in inactive state.

What is Amitryptiline?

- Antidepressivum (TCA) - tricyclic antidepressent - blocking reuptake of serotonin and norepinephrin) - Forholdsvis sterk sedativ og ANTIKOLINERGIC virkning. Av den grunn dårlig egnet hos eldre. - Relativt høyt akutt toksisitet - Foretrekkes ved AGITERTE eller ENGSTELIGE deprisjoner

N - acetylcysteine

- Antidote for acetaminophen (paracetamol) toxicity, N-acetylcysteine (NAC), initially works as a glutathione precursor and glutathione substitute and assists with sulfation - Later on, NAC may function as an antioxidant to aid in recovery. NAC is the most effective when initiated 8 to 10 hours postingestion

What is Betahistine?

- Antivertigo used in Meniere´s disease - It acts on histamine receptors and cause vasodilation and increase permeability to release the pressure from excess endolymph. - It is an Histamine analogue - Used as a hydrochloride salt (with vasodilator?) to reduce the frequency of vertigo in Meniere´s disease

Interferon

- Antiviral agents - Interferon is not active orally, but it may be administered intralesionally, subcutaneously, or intravenously

New variant CDJ

- Anxiety, depression, behavioral changes - Progresive cerebellar syndrome - Myoclonus, and later chorea - Dysesthesis of limbs and face - Pyramidal and extrapyramidal symptoms - Unspecific EEG - Transmitted alimentary (probably) from BSE contaminated cows - prions can be detected in lymphatic tissue before clinical manifestation - All positive cases are homozygots of PRNP

Buspiron

- Anxiolytic psychotropic drug - Useful for the chronic treatment of generalized anxiety disorders - Actions appear to be mediated by serotonin (5-HT1A) receptors, although it also displays some affinity for D2 dopamine receptors and 5-HT2A serotonin receptors - Lacks the anticonvulsant and muscle-relaxant properties of the benzodiazepines - Side effects: headaches, dizziness, nervousness, nausea, and light-headedness

Alzheimer neuropathology

- Atrophy and neuronal loss in isocortex (first entorhinal cortex) - Atrophy of cholinergic systems (nc. basalis Meynerti, amygdala) - Atrophy of dendritic systems

Ziconotide

- Atypical analgesic agent for the amelioration of severe and chronic pain - Acts as a selective N-type voltage-gated calcium channel blocker. This action inhibits the release of glutamate, calcitonin gene-related peptide, and substance P in the brain and spinal cord, resulting in pain relief - Must be administered intrathecally

What is Devic's disease?

- Autoimmune, inflammatory disorder in which a person's own immune system attacks the optic nerves and spinal cord. This produces an inflammation of the optic nerve (optic neuritis) and the spinal cord (myelitis the legs or arms, loss of sensation (including blindness), and/or bladder and bowel dysfunction. - An immunological disorder that may be confused with multiple sclerosis (MS)

What are adverse effects of Atropine?

- Avoid in patients with Acute MI, heart failure or hyperthyroidism (since it increases HR!). - NOTE: Intermittent atropine treatment can be used, especially, after a MI to treat the transient symptomatic bradycardia that follows. Ask if the heart rate is adequate to perfuse the body and maintain an adequate BP. - Tachycardia. - Blurred vision, pupillary dilation, constipation, urinary retentions..anticholinergic effects. „Stimulant" CNS symptoms: a. Hallucinations. b. Confusion. c. Restlessness. • Be careful when patients is also given TCA, since they both have antimuscarinic effects!!

Phenobarbital

- Barbiturate that works by increasing the activity of the inhibitory neurotransmitter GABA - Used in the treatment of all types of seizures except absence seizures - First-line choice for the treatment of neonatal seizures - Sometimes used for alcohol detoxification and benzodiazepine detoxification for its sedative and anticonvulsant properties

Benzalkonium (benzalconio)

- Benzalkonium chloride solution is a topical antiseptic

Diazepam (Valium, Diastat)

- Benzodiazepine - Facilitate amnesia while causing sedation, enhancing the inhibitory effects of various neurotransmitters, particularly GABA - Drugs of choice in terminating status epilepticus - Useful in the acute treatment of alcohol withdrawal and reduce the risk of withdrawal-related seizures - Useful in the treatment of skeletal muscle spasms, such as occur in muscle strain, and in treating spasticity from degenerative disorders, such as multiple sclerosis and cerebral palsy

Midazolam

- Benzodiazepine - Used to facilitate amnesia while causing sedation prior to anesthesia

Clonazepam

- Benzodiazepine - Used to prevent and treat seizures, panic disorder, and for the movement disorder known as akathisia - Should not use clonazepam if you have narrow-angle glaucoma or severe liver disease - Side effects: sleepiness, poor coordination, and agitation. Long-term use may result in tolerance, dependence, and withdrawal symptoms if stopped abruptly

Flumazenil

- Benzodiazepine antagonist - GABA receptor antagonist that can rapidly reverse the effects of benzodiazepine - Available for intravenous administration only - Onset is rapid, but the duration is short, with a half-life of about 1 hour - Side effects: dizziness, nausea, vomiting, and agitation

Tetrazepam

- Benzodiazepine derivative with anticonvulsant, anxiolytic, muscle relaxant and slightly hypnotic properties - Used to treat muscle spasm, anxiety disorders such as panic attacks, or more rarely to treat depression, premenstrual syndrome or agoraphobia - Used therapeutically as a muscle relaxant

What is Clonazepam?

- Benzodiazepine.

To prevent migraine we can use beta blockers, antidepressant, anticonvulsant, calcium channel blockers or botox ?

- Beta-blockers - Calcium channel blockers - Anticonvulsant - Antidepressants - type of barbiturates - Anti-epileptic drugs - Botox

Select the correct statements: - Delusion is a symptom of panic anxiety disorder - Delusion influences behavior, is irrefutable and is a symptom of obsessive and compulsive disease - Between paranoid delusions include persecutory delusions, delusions of influencing and controlling, delusional megalomaniac delusions among makromanické - The formation of error applies mezolimbický dopaminergic system

- Between paranoid delusions include persecutory delusions, delusions of influencing and controlling, delusional megalomaniac delusions among makromanické - The formation of error applies mezolimbický dopaminergic system

Pregabalin (lyrica)

- Binds to the α2-δ site, an auxiliary subunit of voltage-gated calcium channels in the CNS, inhibiting excitatory neurotransmitter release - The exact role this plays in treatment is not known, but the drug has proven effects on focal-onset seizures, diabetic peripheral neuropathy, postherpetic neuralgia, and fibromyalgia - Side effects: sleepiness, confusion, trouble with memory, poor coordination, dry mouth, problem with vision, and weight gain

Povidon

- Broad spectrum antiseptic for topical application in the treatment and prevention of infection in wounds, using the antiseptic properties of iodine. It may be used in first aid for minor cuts, grazes, burns, abrasions and blisters

Chloroxylenol

- Broad-spectrum antimicrobial chemical compound used to control bacteria, algae, fungi and virus - It is used in hospitals and households for disinfection and sanitation - It is also commonly used in antibacterial soaps, wound-cleansing applications and household antiseptics

Indicaitons for giving Capsaicin?

- CHRONIC pain - Postherpetic neuralgia - Painful diabetic neuropathy - Osteoarthritits NOTE! Applied TOPICALLY ONLY!! Has to be used regularly (3-4x daily) to prevent burning sensation from reoccuring Cue tensa: 8% capsaicin for treatment of pain

Flunarizine

- Calcium antagonist - It is effective in the prophylaxis of migraine, occlusive peripheral vascular disease, vertigo of central and peripheral origin, and as an adjuvant in the therapy of epilepsy - Contraindicated in patients with depression, in the acute phase of a stroke, and in patients with extrapyramidal symptoms or Parkinson's disease. It is also contraindicated in hypotension, heart failure and arrhythmia - Side effects include drowsiness, weight gain, as well as extrapyramidal effects and depression in elderly patients

Subarachnoid bleeding

- Can be non-traumatic (rupture of *berry aneurysm*) - If traumatic, there is brain contusion as well - Manifest clinically as acute brain attack that requires surgery (NB! danger of recurrence)

Subdural bleeding (hematoma)

- Can be non-traumatic and without skull fracture - Bleeding from *emissarial veins* - Latency: long, variable - Period from first symptoms to irreversible damage can be long - Surgery in case of clincal symptoms, but can be asymptomatic and resolve

Tolperisone

- Centrally acting muscle relaxant - Indicated for use in the treatment of pathologically increased tone of the cross-striated muscle caused by neurological diseases (damage of the pyramidal tract, multiple sclerosis, myelopathy, encephalomyelitis) and of spastic paralysis and other encephalopathies manifested with muscular dystonia - Should not be used in patients with myasthenia gravis

What is Donepezil?

- Centrally acting reversible acetylcholinesterase inhibitor. - Palliative treatment of mild to moderate Alzheimer's disease. - Common side effects include gastrointestinal upset.

Rivastigmine

- Cholinergic agonist - Used as first-line treatments for Alzheimer's disease

Pyridostigmine

- Cholinergic agonist (indirect acting) - Cholinesterase inhibitors that are used in the chronic management of myasthenia gravis - Durations of action are intermediate (3 to 6 hours and 4 to 8 hours, respectively)

Donepezil

- Cholinergic agonist (reversible) & Anti-Alzheimer - Used as first-line treatments for Alzheimer's disease, though confers modest benefit - Can be used with memantine (N-methyl-D-aspartate antagonist) with moderate to severe disease - Reversible AChE inhibitor

Multiple sclerosis

- Chronic demyelinating disease - Relapsing-remittent cource - Affect young adults, 1/1000

What is Azatioprine?

- Classic immunosuppressor - Used in MS treatment

Methylprednisolone

- Corticosteroid - Used for its anti-inflammatory effects - Common uses include arthritis therapy and short-term treatment of bronchial inflammation or acute bronchitis due to various respiratory diseases. It is used both in the treatment of acute periods and long-term management of autoimmune diseases, most notably systemic lupus erythematosus. It is also used as a treatment for multiple sclerosis

What is Bupropion?

- DNRI (dopamin noradrenaline reuptake inhibitor) - treat neuropathic pain

Select the correct statements: - Disorientation are among formal thought disorder - Delirium is a disturbance of consciousness - Auditory hallucinations are a typical symptom of dementia - Structure of the hypothalamus is important for the regulation of visual perception

- Delirium is a disturbance of consciousness

NEUROPATHOLOGY OF DEMENTIA

-->

PATHOLOGY OF DEMYELINATION

-->

Select the correct statements: - Withdrawal of ideas is a typical symptom of bipolar disorder - Delusion consistent with mood occurs with mood disorders - Monitoring by secret police is a nonbizarre ilusion if other criteria are met. - Serotonergic psychosis theory is based on the results of experiments with marijuana

- Delusion consistent with mood occurs with mood disorders - Monitoring of secret police is a fallacy nebizardní if other criteria are met and delusion, then the fallacy nebizarní. LSD - not Marijuana on the last statement

What is Balo-concentric sclerosis?

- Demyelinating Disease - White matter of brain is damaged in concentric layers, leaving axis-cylinder intact - "leuko-encephalitis periaxialis concentrica" - Borderline form of MS

Examples of acquired metabolic and toxic neuropathies

- Diabetic neuropathy - Metabolic and nutritional - Toxic - Associated with malignancy

Acute disseminated encephalomyelitis (ADEM, perivenous encephalomyelitis)

- Diffuse monophasic demyelinating disease following a viral infection (or smallpox/Rabies vaccination) - AI response against myelin basic protein - Rapid clinical course, many die, some recover - Symptoms: headache, lethargy, coma 1-2 weeks after infection

Nimodipine

- Dihydropyridine calcium channel blocker originally developed for the treatment of high blood pressure - Good results in preventing a major complication of subarachnoid hemorrhage (a form of cerebral hemorrhage) termed vasospasm - It is contraindicated in unstable angina or an episode of myocardial infarction more recently than one month

Tiapride

- Drug that selectively blocks D2 and D3 dopamine receptors in the brain - It is used to treat a variety of neurological and psychiatric disorders including dyskinesia, alcohol withdrawal syndrome, negative symptoms of psychosis, and agitation and aggression in the elderly

What is adverse effects of nasal alpha 1 mimetics? o Dryness of nasal mucosa o Burning sensation o Rhinorrhea o Paradoxial hypotension

- Dryness of nasal mucosa - yes - Burning sensation - yes Not hypo, but paradoxial HYPERtension

What are the indications for use of Benzatropin?

- Dystonias(especially drug induced)-oculogyric crisis (look biperiden, clonazepam for more). So it is used against Extrapyramidal Side effects due to drugs e.g. metoclopramide! - Parkinson's disease especially if there are Mild symptoms and tremor predominates.

Clinical features of frontotemporal dementia (FTD)

- Early behavioral change and desinhibition - Apathy - Perservation, stereotypia, compulsivity - "Collecting" - Hyperorality - Neuropsychological fronto-temporal profile - Good neuropsyhological tests

Select the correct statements: - Behave calmly and try to talk delusion in a patient with delusions - Adequate inquiry to determine peremptory hallucination: Who speaks for you? - Fail Association - rhyming, play on words, the patient connects words based on acoustic similarity - Verbigerace - repetition of words and phrases without sense

- Fail Association - rhyming, play on words, the patient connects words based on acoustic similarity - Verbigerace - repetition of words and phrases without sense (typical in schizophrenia and Alzheimers) Peremptory: That you must obey without any questions or excuses

Notes about Carbamazepin:

- For pregnant women it can cause NTD(give folate) and Neonatal Bleeding(give vitamin K). - For long-term control of seizures it is given ORALLY. - NOTE: It induces its own metabolism in the liver by CYP3A4. After 2 weeks the dose may not be enough to control the seizures. CARBAMAZEPINE (find in the word the other drugs which induce their own mtb as well) C for carbamazepine, R for rifampicin, B for barbiturates, Z for St John's Wort and P for Phenytoin.

ena cerebri magna runs from 4th ventricle via foramen Magendie

- Foramen of magendie/median aperture does drain 4th ventricle and empty into cerebri magna, think it should be true. TH

What is the function of Cloazepam?

- Function: It enhances GABAergic transmission by binding to a site ADJACENT to GABA receptor. - anxiolytic, muscle relaxant, anticonvulsant, amnestic, sedative and hypnotic properties - Primarly prescribed to acute management of epileptics and sometimes panic disorders

How does Dentrolene work?

- Functions by decreasing the calcium concetration in the ER causing dissociation of the excitation-contraction coupling. - Acts peripherally rather than centrally to produce muscle relaxation.

What kind of drug is Baclofen? - Receptor? - pre or postsynaptic? - used for what?

- GABA-B receptor agonist - Postsynaptic (RW: and presynaptic!) - Used for spasticity-muscle relaxant

Examples of human prion diseases

- Gerstmann-Sträussler-Scheinker syndrome (GSS) - Fatal familial insomnia (FFI) - Sporadic fatal insomnia (FSI) - Kuru - *Creutzfeldt-Jakob disease* (CJD) - sporadic, iatrogenic, familial, new variant

ADEM morphology

- Grey discoloration around white-matter vessels - Myelin loss with relative preservation of axons, lesions appear similar (monophasic) in contrast to MS

Select the correct statements: - Heat of passion is intense and violent emotional reaction - Recurrent depression is characterized by at least one hypomanic episode and one depressive - Moria is morbidly mischievous mood with jokes fatuous, frivolous and euphoric mood - Individuals with a history of aggressive and impulsive behavior have increased levels of 5-HIAA in the CSF

- Heat of passion is intense and violent emotional reaction - Moria is morbidly mischievous mood with jokes fatuous, frivolous and euphoric mood *Agression* • Serotonin levels show negative correlations with aggression o Destruction of 5-HT axons in forebrain facilitates aggressive attack. o Diminished 5-HIAA levels in CSF of people with history of violence and impulsive aggression

What is the function of Amitryptiline?

- Hemming av gjennoptaket av noradrenalin (NA) og seratonin (5-HT) i nerveterminalen (Central monaminergic neurons) - In fact the antidepressant effect is due to the change in monaminergic transmission that leads to adaptive changes! Thats why TCA takes about 2 weeks to work!!

When should Amantadine NOT be used?

- In PSYHIATRIC patients, - Patients with DEMENTIA or - PREGNANT women. The elderly have an increased risk of confusion.

What are the indications for use of Baclofen?

- Indications as a MUSCLE RELAXANT is to alleviate muscle spasms due to: - CP - MS. - Stroke. - Spinal cord tumors. - NOTE: NOT for muscle spasms due to Acute injury! - The efficacy of Baclofen in stroke, cerebral palsy, and Parkinson's disease has not been established and, therefore, it is not recommended for these conditions.- Avoid in pregnancy.

What is Zolpidem used for?

- Insomnia; short-acting nonbenzodiazepine hypnotic of the imidazopyridine class that potentiates GABA by binding to GABA A receptors at the same location as benzodiazepines - Works quickly, usually within 15 minutes, and has a short half-life of two to three hours.

Criteria for PTSD

- Intense, stress reaction to a traumatic event. - Persistent, re-experiencing - Avoiding thoughts, feelings or conversations linked to a trauma - Increased arousal

Pegaptanib

- Is an anti-angiogenic medicine for the treatment of neovascular (wet) age-related macular degeneration - It is a pegylated anti-vascular endothelial growth factor (VEGF) aptamer, a single strand of nucleic acid that binds with specificity to a particular target - Works as an antagonist to VEGF, which when injected into the eye blocks the actions of VEGF. This then reduces the growth of the blood vessels located within the eye and works to control the leakage and swelling

What is the function of Baclofen?

- Is an derivative of GABA. (Agonist for the GABA-B receptors) - Selective agonist at GABA-B - It is primarily used to treat spasticity and is in the early research stages for use for the treatment of alcoholism.

What is Benzalconium?

- It is a Quartenary ammonium compound. (4 organic groups covalently linked to the nitrogen molecule) - Its Germicidal activity is determined by the organic groups!! Greatest activity is seen when the compounds have 8-18 carbon long groups.

What is the function of Atropine?

- It is an MUSCARINIC RECEPTORS ANTAGONIST (competetive antagonist of muscarinic receotors) = parasympatholytic actions - Acetylcholin antagonist

What is Alteplase derived form?

- It is derived from the NATURAL SUBSTANCE "Tissue plasminogen activator" - Single-chain recombinant tPA-FIBRINOLYTIC - Very short halft life - 6 minutes

What is the function of Benserazide when combined with L-DOPA?

- Levodopa is a precursor to the neurotransmitter dopamine which is administered to increase its levels in the central nervous system. However, most levodopa is decarboxylated to dopamin before it reaches the brain, and since dopamin is unable to cross blood-brain barrier, this translates to little therapeutic gain with strong peripheral side effects. - Benserazide inhibits the mentioned decarboxylation, and since it itself cannot cross the blood-brain barrier, this allows dopamin to build ip solely in the brain instead. Adverse effects caused by peripheral dopamin, such as vasoconstricion, nausea, and arrythmia, are minimized.

PATHOLOGY OF MUSCLES AND NERVES

-->

Benzydamine

- Locally-acting NSAIDs with local anaesthetic and analgesic properties for pain relief and anti-inflammatory treatment of inflammatory conditions of the mouth and throat

Furosemide

- Loop diuretic - Act promptly by blocking sodium and chloride reabsorption in the kidneys, even in patients with poor renal function or those who have not responded to thiazide diuretics - Loop diuretics cause decreased renal vascular resistance and increased renal blood flow - Loop diuretics increase the Ca2+ content of urine - Inhibit the cotransport of Na+/K+/2Cl− in the luminal membrane in the ascending limb of the loop of Henle. Therefore, reabsorption of these ions is decreased - Drug of choice for reducing acute pulmonary edema and acute/chronic peripheral edema caused from heart failure or renal impairment. Because of their rapid onset of action, particularly when given intravenously, the drugs are useful in emergency situations such as acute pulmonary edema. Loop diuretics (along with hydration) are also useful in treating hypercalcemia, because they stimulate tubular Ca2+ excretion. They also are useful in the reatment of hyperkalemia. - Can cause hypokalemia

What does a central cord lesion lead to?

- Loss of pain and temperature at the levels of lesion - LMN weakness at level of lesion - If spinothalamic tract is affected there is pain and temperature loss below the lesion - In corticospinal tract is affected there is UMN weakness below the lesion The other modalities of sensation is preserved (or not as much affected at least) - "dissociated sensory loss!"

What are indications for use of Carbamazepin?

- MOOD stabilizer.(Used in Bipolar disorder if unresponsive to Lithium) - Seizures. - Trigeminal neuralgia.

Valproate

- Medications primarily used to treat epilepsy and bipolar disorder and to prevent migraine headaches - Useful for the prevention of seizures in those with absence seizures, partial seizures, and generalized seizures - It can be given intravenously or by mouth - Side effects: nausea, vomiting, sleepiness, and a dry mouth

Hydroxypropofol

- Metabolite of Propofol, a phenol derivative that is suggested to modulate the sensitivity of capsaicin to the transient receptor potential vanilloid subtype-1 (TRPV1) receptor - TRPV1 is a Capsaicin receptor that behaves as a non-selective cation channel that is mainly found in the sensory neurons, and specifically detects painful stimuli, low pH and heat

Ranibizumab

- Monoclonal antibody fragment - Approved to treat the "wet" type of age-related macular degeneration, a common form of age-related vision loss - Inhibits angiogenesis by inhibiting Vascular endothelial growth factor A

PML - morphology

- Multiple demyelinating foci near grey-white junction in hemispheres and brainstem - Typical lesion is a central area without myelin (with residual axons) and a few large oligodendrocytes with hyperchromatic, ground-glass intranuclear inclusion, macrophage infiltration and pleomorphic astrocytes

What is Dentrolene?

- Muscle Relaxant.

Baclofen

- Muscle relaxant - It is a GABA receptor agonist, specifically of the GABA-B receptors - It is primarily used to treat spasticity

What are indications to give Distigmine?

- Myasthenia gravis treatment. (R. Neostigmine and physostigmine_long-term action. Also, SOS: neostigmine is a hydrophobic molecule, whereas physostigmine is a lipophilic molecule and therefore ONLY physostigmine can crosss the BBB!!!) ii. Distigmine has the longest duration of action, but also the highest risk of cholinergic crisis dvp. Therefore it is RARELY used!! iii. Neurogenic bladder due to UMN lesion. c. NOT for postoperative acute urinary retention because of the Heart AE!!

What is the difference between Buspiron and traditional anxiolytic drugs (e.g BZP and Barbiturates)?

- NO hypnotic effects!! (compared to traditional anxiolytic/hypnotic drugs: BZP and Barbiturates) - Needs 3 weeks to effect(like antidepressants) so in case that ACUTE treatment is required you have to use BZP that will act within 30 min.!!

What is Benzydamin?

- NSAID with local anestetic properties Lokalbedøvende. Available as the hydrochloride, is a locally-acting nonsteroidal anti-inflammatory drug with local anaesthetic and analgesic properties for pain relief and anti-inflammatory treatment of inflammatory conditions of the MOUTH and THROAT.

Select the correct statements: - Nonbizarre delusion can talk using rational arguments - Nihilistic delusions are mistaken belief that a patient or other world does not exist, or ends - Content of bizarre delusion is absurd, unrealistic and special - Dopamine theory of psychosis is based on the finding that psychotropic drugs induce psychotic symptoms

- Nihilistic delusions are mistaken belief that a patient or other world does not exist, or ends (yes) - Content bizarre delusion is absurd, unrealistic and special (yes) - Dopamine theory of psychosis is based on the finding that psychotropic drugs induce psychotic symptoms (yes) Why isn't A correct? Because they mean that the delusion talks to the patient?! Non-bizzare are "possible" delusions, so the arguments could maybe be rational? Klara: Maybe they mean that you cant talk people out of the delusion with rational arguments :)

What are the symptoms of conus lesion?

- No typical motor problems - only flexors of toes affected - Perianal, perigenital sensory disturbance - Sphincter and sexual motor dysfunction RW PPT: conus lesion: s3-s5 paraparesis...is not present! * sensory disturbance - saddle anesthesia * pain over the "saddle area" * sphincteric paralysis: * disturbance of urination (denervated autonomous bladder - retention) * faecal incontinence * sexual dysfunction - impoten From pathophys: motor function in legs and feet may be impaired, but sensory function is preserved.

Pilocarpine

- Nonselective muscarinic agonist used in clinical practice to treat xerostomia and glaucoma - Beneficial in promoting salivation in patients with xerostomia resulting from irradiation of the head and neck - Sjögren syndrome, which is characterized by dry mouth and lack of tears, is treated with oral pilocarpine tablets - Produces rapid miosis and contraction of the ciliary muscle. When the eye undergoes this miosis, it experiences a spasm of accommodation. The vision becomes fixed at some particular distance, making it impossible to focus

Timolol

- Nonselective β antagonists - Reduces the production of aqueous humor in the eye. It is used topically in the treatment of chronic open-angle glaucoma and, occasionally, for systemic treatment of hypertension

What is Dihydroergotoxin?

- Nootropic drug used in dementia and recovering from strokes - Dihydroergotamine=brain vasodilator. NOOTROPIC. - R. Also flunarizine which is also a nootropic. - Stimulate dopaminergic and seratonergic receptors while blocking alpha-adrenoreceptors - Adverse effects are minimal

Who get's sciwora?

- Occurs most often in pediatric population; childhood cervical injuries - Spine is immobilized for one to three weeks

What are indications for use of Cloazepam?

- Oculogyric crisis unresponding to anticholinergics: benzatropine, biperiden, phencyclidine!! - Seizures. - As with baclofen, NOT suitable for muscle spasm due to acute injury.

Tramadol

- Opioid - Is a centrally acting analgesic that binds to the μ opioid receptor - It weakly inhibits reuptake of norepinephrine and serotonin - It is used to manage moderate to moderately severe pain - Naloxone can only partially reverse the analgesia produced by tramadol or its active metabolite. - Has been associated with misuse and abuse

Naloxone

- Opioid antagonist - Used to reverse the coma and respiratory depression of opioid overdose. It rapidly displaces all receptor-bound opioid molecules and, therefore, is able to reverse the effect of a morphine overdose - Within 30 seconds of IV injection of naloxone, the respiratory depression and coma characteristic of high doses of morphine are reversed, causing the patient to be revived and alert - Half-life of 30 to 81 minutes

Oxycodone

- Opioids (strong agonists) - Semisynthetic derivative of morphine - It is orally active and is sometimes formulated with aspirin or acetaminophen

Which nerves are most often affected in MS?

- Optic nerve and chiasm - Periventricular white matter

Hexylresorcinol

- Organic compound with local anaesthetic, antiseptic and anthelmintic properties

Central pontine myelinolysis

- Osmotic demyelination syndrome - caused by too rapid correction of hyponatremia, or in alcoholics, malnutrition and renal failure - Discrete areas of selective demyelination, often so small that they don´t manifest clinically. However, it may lead to quadriparesis, locked-in-syndrome

What is Atropine?

- PARASYMPATOLYTIC - Atropin, er et legemiddel som framstilles av belladonnaplanten of er en potensielt giftig medisinsk urt. - Atropin brukes til behandling av en lang rekke lidelser: Det lammer nerve-ender og virker avslappende på innvollsmuskulatur, som er forbundet med dem. - Det kan brukes til å tørke ut lunge og strupe før det gis narkose - Utvider pupillene

Dipyridamole

- Platelet inhibitor - Coronary vasodilator, increases intracellular levels of cAMP by inhibiting cyclic nucleotide phosphodiesterase, thereby resulting in decreased thromboxane A2 synthesis - Used for stroke prevention and is usually given in combination with aspirin - Patients with unstable angina should not use dipyridamole because of its vasodilating properties, which may worsen ischemia - Side effects: bleeding, dizziness, GI discomfort, rash, headache and can lead to orthostatic hypotension

Clopidogrel

- Platelet inhibitor - Inhibit the binding of ADP to its receptors on platelets and, thereby, inhibit the activation of the GP IIb/IIIa receptors required for platelets to bind to fibrinogen and to each other - Max. inhibition of platelet aggregation is achieved in 3 to 5 days with clopidogrel - Approved for prevention of atherosclerotic events in patients with a recent MI or stroke and in those with established peripheral arterial disease. It is also approved for prophylaxis of thrombotic events in acute coronary syndromes (unstable angina or non-ST-elevation MI). Additionally, clopidogrel is used to prevent thrombotic events associated with percutaneous coronary intervention (PCI) with or without coronary stenting

Which sensory modalities are affected by posterior column syndrome

- Posterior column syndrome: loss of deep sensation of vibration, propioception, discriminative touch

Dantrolene

- Postsynaptic muscle relaxant that lessens excitation-contraction coupling in muscle cells - Blocks release of Ca2+ from the sarcoplasmic reticulum of muscle cells, reducing heat production and relaxing muscle tone - Used in treatment of malignant hyperthermia

Select the correct statements: - Presence of derilia find these questions: Name? Day? Where are we? - Adequate inquiry to determine imperative hallucinations: Thinking about suicide? - Puzzling is the sudden interruption of visual perception - Auditory hallucinations are a typical symptom of Wernicke-Korsack syndrome

- Presence derilia find these questions: Name? Day? Where are we? - Auditory hallucinations are a typical symptom of Wernicke-Korsack syndrome

Select the correct statements: - Presence of delusion investigating the question: Is there anything special about? - Mikrozoopsie is typical of delirium tremens - Illusion is real distorted perception of external stimulus - Perseveration is persistent response to previous stimulant after being presented a new stimulus

- Presence of delusion investigating the question: Is there anything special about? - Mikrozoopsie is typical of delirium tremens - Illusion is real distorted perception of external stimulus - Perseveration is persistent response to previous stimulant after being presented a new stimulus

Methylphenidate

- Psychomotor stimulants - CNS-stimulant properties similar to those of amphetamine and may also lead to abuse, although its addictive potential is controversial - Methylphenidate is presently one of the most prescribed medications in children - Dopamine and norepinephrine transport inhibitor and may act by increasing both dopamine and norepinephrine in the synaptic space - Methylphenidate has been used for several decades in the treatment of ADHD. It is also effective in the treatment of narcolepsy

Select the correct statements: - Episode of schizophrenia is a period without symptoms - Relapse is a return of symptoms after a period of rest - The positive symptoms of schizophrenia are flat affect, depressed mood and suicidal considerations - Bleulers 4a's Association, autism, affect and ambivalence

- Relapse is a return of symptoms after a period of rest - Bleulers 4a's Association, autism, affect and ambivalence

Select the correct statements: - Schizophrenia in medical terminology is split personality - Schizophrenia begins between 30-40 years - Relapse within 5 years 80% of patients with schizophrenia - Crow schizophrenia divided into 2 types according to the number of schizophrenic episodes

- Relapse within 5 years 80% of patients with schizophrenia

Select the correct statements: - Remission of schizophrenia is a period without symptoms - Positive schizophrenia symptoms recede after administration of antipsychotic medication - Schizophrenia is a disorder of mood - Schieiderova's 4A are aggression, anhedonia, affect, apathy

- Remission of schizophrenia is a period without symptoms - Positive schizophrenia symptoms recede after administration of antipsychotic medication

Moclobemide

- Reversible inhibitor of monoamine oxidase A drug primarily used to treat depression and social anxiety

What is Citalopram?

- SSRI: Inhibitor of reuptake of 5-HT in central monoaminergic neurons. - Adaptive changes, check TCA, same mechanism of action (so SSRIs take also about 2 weeks to effect), BUT SSRIs DO NOT have marked central and peripheral anticholinergic effects!!

Select the correct statements: - Schizoaffective disorders among psychotic illness - Psychosis characterized by impairment of consciousness - Early warning signs announce the onset of psychotic illness remission - Crow schizophrenia divided into 2 types according to response to antipsychotic treatment, ventricular size on CT, the presence of the + and - signs

- Schizoaffective disorders among psychotic illness - Crow schizophrenia divided into 2 types according to response to antipsychotic treatment, ventricular size on CT, the presence of the + and - signs

What are the indications to use Dipyridamole?

- Secondary prevention of stroke after stroke or TIA. - Adjunct to oral anticoagulants_in pts with prosthetic heart valves. Prophylaxis of thromboembolism. - Less bleeding compaired to other drugs(anticoagulants..) - Given with aspirin for secondary prevention of stroke/TIA.

What are indications for use of Clopidogrel?

- Secondary prevention of stroke/TIA. - Acute coronary syndrome and following PTCA. d. Consider if patient is hypersensitive or unable to tolerate aspirin. e. Needs controlled BP. f. Risk of bleeding. (think pregnancy, hemophiliacs) g. Most effective for „white clots", which means platelets and arteries.

Zolpidem

- Sedative primarily used for the treatment of insomnia - The hypnotic zolpidem is not structurally related to benzodiazepines, but it selectively binds to the benzodiazepine receptor subtype BZ1 - Zolpidem has no anticonvulsant or muscle-relaxing properties - Zolpidem is rapidly absorbed from the GI tract, and it has a rapid onset of action and short elimination half-life (about 2 to 3 hours). - It provides a hypnotic effect for approximately 5 hours - Side effects;: nightmares, agitation, anterograde amnesia, headache, GI upset, dizziness, and daytime drowsiness - Flumazenil, a benzodiazepine receptor antagonist, which is used for benzodiazepine overdose, can also reverse zolpidem's sedative/hypnotic and memory-impairing effects

What are indications for use of Diazepam?

- Seizures. - Status epilepticus-1st line treatment!!! - Treatment and prophylaxis of febrile convulsions. - Severely agitated patients are given i.v. diazepam e.g. those withdrawing from alcohol.

Selegiline

- Selective irreversible monoamine oxidase inhibitor (MAOIs) - Used to reduce symptoms in early-stage Parkinson's disease. A transdermal patch is used to treat depression - Treatment of depression and anti-Parkinson´s

Fluoxetine

- Selective serotonin reuptake inhibitor - Also approved for bulimia - Half-life is 50 hr.

Escitalopram

- Selective serotonin reuptake inhibitor - Approved to treat childhood depression

Citalopram

- Selective serotonin reuptake inhibitor (SSRI) - The SSRIs block the reuptake of serotonin, leading to increased concentrations of the neurotransmitter in the synaptic cleft - may cause QT prolongation - Is licensed for depressive episodes and panic disorder with or without agoraphobia - Frequently used off-label to treat anxiety, panic disorder, dysthymia, premenstrual dysphoric disorder, body dysmorphic disorder and obsessive-compulsive disorder - Sexual dysfunction is often a side effect with SSRIs - Side effects: drowsiness, insomnia, nausea, weight changes (usually weight gain), increase in appetite, vivid dreaming, frequent urination, decreased sex drive, anorgasmia, dry mouth, increased sweating, trembling, diarrhea, excessive yawning, and fatigue

Sertraline (Zoloft)

- Selective serotonin reuptake inhibitors - Antidepressants

Pyritinol

- Semi-synthetic water-soluble analog of vitamin B6 - Approved for "symptomatic treatment of chronically impaired brain function in dementia syndromes" and for "supportive treatment of sequelae of craniocerebral trauma"

Dihydroergotoxin

- Semisynthetic derivative of ergot alkaloid ergotoxine - Causes dilation of blood vessels (vasodilation) taken esp. for the treatment of circulatory disorders (peripheral, cerebral, ophthalmic, etc.)

Outcome of global ischemia

- Sequelae depend on the duration, condition of vessels, body temp, age, perfusion (reperfusion) pressure, saturation of oxygen - The infarcts manifest as amnesia, blindness, motoric disorders

What is Serotonin syndrome?

- Serotonin syndrome is a potential symptom of any number of life-threatening drug interactions which may follow therapeutic drug use, combination, overdose of drugs, or the recreational use of certain drugs. - It is not an ideopathic drug reaction; it is a predictable consequence of ecxess serotonin on the CNS and/or peripheral nervous system.

Venlafaxin

- Serotonine/Norepinephrine reuptake inhibitor (SNRIs) - Inhibit the reuptake of both serotonin and norepinephrine - Side effects: nausea, headache, sexual dysfunction, dizziness, insomnia, sedation, and constipation - At high doses, there may be an increase in blood pressure and heart rate

Types of vascular dementia

- Strategic infarct dementia (bilateral thalamus, frontal diencephalon) - Multi-infarct dementia - Subcortical vascular dementia (small vessel disease)

Vit B12 deficiency (pernicius anemia)

- Subacute combined degeneration of spinal cord (dorsal and lateral part) -> myelin and axonal loss - Rapidly progressive and poorly reversible neurologic disorder - Gliosis and atrophy of spinal cord with time

Metoclopramide

- Substituted benzamides with antiemetic activity - Inhibition of dopamine in the CTZ (chemoreceptor trigger zone) - Antidopaminergic side effects, including extrapyramidal symptoms, limit long-term high-dose use - Metoclopramide was previously used as a prokinetic drug for the treatment of GERD. However, due to the adverse effect profile and the availability of more effective drugs, such as PPIs, it should be reserved for patients with documented gastroparesis

Which sensory modalities are affected by syringomyelia ?

- Syringomyelia: loss of pain and temp in a "cape like" distribution

Select the correct statements: - A typical and frequent somatic symptom of depression is waking at least two hours after the usual time of getting up - The mood is a long and lasting emotional tuning - Alexithymia - the inability to understand emotions - Kluver-Bucy syndrome-loss fear in lesion orbitofrontal region

- The mood is a long and lasting emotional tuning - Alexithymia - the inability to understand emotions *Kluver-Bucy*: a syndrome resulting from bilateral lesions of the medial *temporal lobe* (including amygdaloid nucleus). Klüver-Bucy syndrome may present with hyperphagia, hypersexuality, hyperorality, visual agnosia, and docility.

Streptokinase

- Thrombolytic agent - Extracellular protein purified from culture broths of group C β-hemolytic streptococci - It forms an active one-to-one complex with plasminogen. This enzymatically active complex converts uncomplexed plasminogen to the active enzyme plasmin - In addition to the hydrolysis of fibrin plugs, the complex also catalyzes the degradation of fibrinogen, as well as clotting factors V and VII - Streptokinase is rarely used and is no longer available in many markets

Alteplase

- Thrombolytic agent - Formerly known as tissue plasminogen activator (tPA) - Approved for the treatment of MI, massive PE, and acute ischemic stroke - Very short half-life (5 to 30 minutes), and therefore, 10% of the total dose is injected i.v. as a bolus and the remaining drug is administered over 60 minutes - May cause orolingual angioedema, and there may be an increased risk of this effect when combined with ACE-inhibitors

Define OCD ?

- Time consuming (>1hour per day) of obsessions and compulsions. For diagnosis the OCD must be present most days of the week, for 2 weeks, it must be distressing and interfering with normal activities.

Lithium

- Treatment of mania and bipolar disease - Lithium salts are used acutely and prophylactically for managing bipolar patients. Lithium is effective in treating 60% to 80% of patients exhibiting mania and hypomania - The therapeutic index of lithium is extremely low, and lithium salts can be toxic - Common adverse effects may include headache, dry mouth, polydipsia, polyuria, polyphagia, GI distress (give lithium with food), fine hand tremor, dizziness, fatigue, dermatologic reactions, and sedation - Adverse effects due to higher plasma levels may indicate toxicity and include ataxia, slurred speech, coarse tremors, confusion, and convulsions - Thyroid function may be decreased and should be monitored - Unlike other mood stabilizers, lithium is renally eliminated, and though caution should be used when dosing this drug in renally impaired patients, it may be the best choice in patients with hepatic impairment

What i dysthymia?

2 years of persistent or recurrent mild depressive mood

Amitriptyline

- Tricyclic antidepressant (TCA) - The TCAs block norepinephrine and serotonin reuptake into the presynaptic neuron - The TCAs elevate mood, improve mental alertness, increase physical activity. The onset of the mood elevation is slow, requiring 2 weeks or longer - Have been used to help prevent migraine headache and treat chronic pain syndromes

Sumatriptan

- Triptans - Given subcutaneously, intranasally, or orally - The drug has a short duration of action, with an elimination half-life of 2 hours

Frovatriptan

- Triptans - Longest-acting triptan, with a half-life of more than 24 hours - Treatment of migraine headaches and for short term prevention of menstrual migraine - Inhibits excessive dilation of arteries that supply blood to the head - 5HT receptor agonist, with high affinity for the 5-HT1B/1D receptors - Has no significant effects on the GABAA mediated channel activity and benzodiazepine binding sites

Naratriptan

- Triptans - Used for the treatment of migraine headaches - Due to their activity as 5HT (serotonin) agonists

Sorafenib

- Tyrosine kinase inhibitor - Used mainly in renal cell carcinoma. Also part of the treatment strategy for hepatocellular carcinoma - Target cell surface kinases that are involved in tumor signaling, angiogenesis, and apoptosis, thus slowing tumor growth - Adverse effects include diarrhea, fatigue, hand and foot syndrome, and hypertension

Tizanidine

- Used as a muscle relaxant - It is a centrally acting α2 adrenergic agonist - It is used to treat the spasms, cramping, and tightness of muscles caused by medical problems such as multiple sclerosis, ALS, spastic diplegia, back pain, or certain other injuries to the spine or central nervous system - Also prescribed off-label for migraine headaches, as a sleep aid, and as an anticonvulsant. It is also prescribed for some symptoms of fibromyalgia

Carbamazepine

- Used primarily in the treatment of epilepsy and neuropathic pain - Blocks sodium channels, thereby inhibiting the generation of repetitive action potentials in the epileptic focus and preventing their spread - Effective for treatment of focal seizures and, additionally generalized tonic-clonic seizures, trigeminal neuralgia, and bipolar disorder - Should not be prescribed for patients with absence seizures because it may cause an increase in seizures

Paracetamol (acetaminophen)

- Used to treat pain & fever

Acetylsalicylic acid

- Used to treat pain, fever, and inflammation - Aspirin given shortly after a heart attack decreases the risk of death - Aspirin is also used long-term to help prevent heart attacks, strokes, and blood clots in people at high risk - Common side effects include an upset stomach. More significant side effects include stomach ulcers, stomach bleeding, and worsening asthma

Intracerebal bleeding (hematoma)

- Usually non-traumatic, *arterial* bleeding due to hypertension in basal ganglia and internal capsule, AVM, hemangioma, brain metastasis - Complications: brain edema, progression into brain chambers and subaracnoid space - Surgery is considered according to circumstances and prognosis

Epidural bleeding (hematoma)

- Usually traumatic with skull fracture and bleeding from *middle meningeal artery* - In the epidural space, lentiform shape on CT - Latency: hours - Fast development form first symptoms to irreversible brain damage due to asymmetric compression, herniation and edema --> require urgent surgery!

Types of vascula dementias

- Vascular dementia - types • Strategic infarct dementia • Multi-infarct dementia • Subcortical vascular dementia (Binswanger disease) • Microangiopathy (CADASIL)

Meningeal syndrome can manifest at which diseases?

- Viral and bacterial meningitis - Subarachnoidal bleeding (OBS! One Q tries to trick you with epidural bleeding!!) - Meningeal carcinomatosis, lung, breast or malignant melanoma, cervical cancer

Toxonutritive demyelination

- Vit B12 deficiency - Central pontine myelinolysis - (Solvent abuse)

Latanoprost

- are PGF2α analogs that are indicated for the treatment of open-angle glaucoma - administered into the eyes to control the progression of glaucoma or ocular hypertension by reducing intraocular pressure - lowers the pressure by increasing the outflow of aqueous fluid from the eyes through the uveoscleral tract

Pilocytic astrocytoma

- grade I - children/young adults, cerebellum - cystic tumor with bipolar cells, Rosenthal fibers and granular bodies

Glioblastoma multiforme (GBM)

- grade IV, poorly differentiated - adults, in telencephalon - infiltrating gray mass with *hemorrhage and necrosis*, vascular proliferation - can multicentric = brain gliomatosis - can progress from fibrillary or anaplastic astrocytoma

52. acidum hyaluronicum?

?

What is the function of Dipyridamole?

- inhibits reuptake of adenosine (ADP) which lowers the ADP-induced aggregation of platelets.

Capsaicine (neuropathic pain drug) o Is selective antagonist of TPVR1 o Peripheral desensitization of nerve fibers o IV or rectally applied o Initial phase of capsaicin - associated with burning feeling of the area

- no (it's agonist) - yes -No, only applied topically! (3-4times daily) -yes

NEURODEGENERATIONS 2 - BASAL GANGLIA

-->

What is Buspiron?

- partial 5-HT1A receptor agonist for anxiety attacks. - Anxiolytic azapirone (NOT related to benzodiazeoines or barbiturates!) - Used in general anxiety disorders with NO danger of dependence - It also have some antidepressant effects

Causes of transverse lesions of spinal cord?

- traumatic or ischemic; inflammation or by compression tumour (rare!)

Benserazide

- used in the management of Parkinson's disease in combination with L-DOPA - DOPA decarboxylase inhibitor, which is unable to cross the blood-brain barrier - Often given with LEVODOPA in the treatment of parkinsonism to prevent the conversion of levodopa to dopamine in the periphery, thereby increasing the amount that reaches the central nervous system and reducing the required dose - It has no antiparkinson actions when given alone

What kind of drug is Tizanidine?

- α2 adrenergic agonist. - Relieves spasticity associated with multiple sclerosis and spinal cord injury!! - Muscle relaxant

Lesions in 1. Brain 2. Brainstem 3. Spinal cord Where and what ? :P

--

CEREBROVASCULAR DISEASES AND INJURY

-->

CNS INFLAMMATIONS

-->

CNS TUMORS

-->

Avahuasca - a love potion - crack cocaine free base

-Ayahuasca is brewed from leaves/trees, containing DMT. -Entheogenic drug(used in religious/spiritual ceremonies) -Experimental drug for treating opiod dependency -Serotonin agonist -Needs MAO-Inhibitors to work

Antiglaucoma drug ?

-Cholinergics -Beta-blockers -Alpha2-agonists

Lassegne sign is present in (do they mean lasegue sign?))

-Inflammatory radiculitis and others -In meningitis (according to neuro dept) -In lumbar disc herniation (according to wiki)

The occlusion of left middle cerebral artery leads to?

-weakness/paralysis of contralateral face and arm -sensory loss contralateral face and arm -damage dominant hemisphere leads to afasia -damage non-dominant hemisphere-contralateral neglect syndrome -deviation conjugate-gaze preference towards side of lesion + contralateral homonymous hemianopsia

108. GABA A-Cl- whereas GABA B-K+!!!

.

67. Naratriptan

.

71. Noradrenalin

.

88. Serotonin.

.

1) in irritation of gyrus precentralis there is discovery spasms on the contralateral half of the body and face 2) decerebracni rigidity of the clinical manifestations flexed position of the upper extremity and extended position of lower extremities. 3) lesions in the mesencephalon may occur with central hemiparesis contralateral with hemiataxia and homolateral perifereal lesiom of n.3 4) in the affected cauda we find peripheral paresis of the lower extremities, sensory disturbances and perianal disorders sphincter.

1 - True ? If the whole precentral gyrus is affected we get UMN lesion of contralateral body and face.. I would say *TRUE* 2 - False, decerebrate -> extension if UE and LE 3 - True? Contralateral UMN lesion, hemiataxia, ipsilateral peripheral lesion on CN III 4 - True ?

1) in spinal cord lesion below the cervical intumescence (C4-T1) fault movement occurs only at lower extremities 2) to irritation of parietal lobe in postcentral gyrus patient has hemiparestesii contralaterally 3) acute transverse spinal lesion in the thoracic region may cause urinary retention at first, then automatically bladder and mass reflex

1 - false 2- True! Hemiparestesia, not paresis!!!! 3 - true

1) meningeal syndrome occurs when bleeding from meningeal artery and is a sign of subdural bleeding? 2) Central LE paraparesis without sensory disturbances may be caused by parasagittal lesions (eg. Meningioma) 3) For parkinson syndrome is usually micrographs, hypersalivation and hypomimie 4) When the affected brachial plexus is present HEAVY limb paresis without sensory disturbances

1 - false, should be subarachnoid 2 - ? 3 - true (macrographia is in cerebellar dysfunction) 4 - false (?)

1) Moria is a disorder of mood taken up with inept joking, frivolity and euphoric mood 2) The typical and frequent somatic accompanying symptoms in depression is woken up at least two hours of normal waking time 3) The tune is a long-term and sustained overall emotional tuning 4) The reaction of seemingly fury (sham rage) is a phenomenon observed in animal models, which originates result bilaterliniho impaired amygdala

1 - true 2 - true (waking up 2h earlier) 3 - (mood is long lasting, but changable) 4 - false - decortication (separation of cortex from hypothalamus according to our powerpoint! Bilateral temporal lobe/amygdala lesion --> Kluver Bucy syndrome!

1) Ischemia some areas of the brain may be a consequence of intracerbral bleeding? 2) aphasia most often occurs when there is a stroke in the left hemisphere chicken? 3) Stroke has only ischemia character? 4) aphasia develops as a consequence of ischemia in the supply of the right middle cerebral artery

1 - true Reflex vasoconstriction after intracerebral bleeding 2 - True (aphasia - dominant-hemisphere!) 3 - false, ischemic + hemorrhagic 4 - true (K: shouldnt it be false, should be left middle c. a?)

What is the dosage of Donepezil?

1 time daily, due to pharamcodynamics

1) in the pathophysiology of neurodegenerative disease processes are applied genetic influences 2) among peptides which are involved in the pathophysiology of neurodegeneration belong amyloid beta peptide 3) protein TDP-43 is physiologically localized in the nucleus of the neuron 4) the underlying pathophysiological disorder tau protein dehydration

1, 2, 3

1) among disease with repeating triplet belongs Kennedy bulbar spinal muscular atrophy 2) alzheimer disease not typical neurodegenerative diseases 3) Pick disease with macroscopically evident atrophy, principally located in the frontal and temporal 4) The typical morfological substrate in Pick disease is Lewy plaque

1, 3 Pick bodies (tau-positive spherical cytoplasmic neuronal inclusions, composed of straight filaments

1) to various stimuli are first CONSCIOUS of the mouse and then activates fear 2) in the unwinding of depression demonstrate activation of subgenual area in cingulate region (ba25) 3) potentiation of the startle response is the context in anxious patients at increased 4) The principal research paradigm study of distress is a startle response

1- 2 - True 3 - 4 - The subcallosal cingulate gyrus CG25 which consists of BA25 as well as parts of BA24 and BA32 has been implicated as playing an important role in major depression and has been the target of deep brain stimulation to treat the disorder. 2 was marked as false in the newest test. It is involved but not in the unwinding of depression, that is what I think. TH

Prevalence of Alzheimer

1-3/1000 per year in population over 60 The prevalence is doubling every 5th year after age of 65

Tumors from astrocytes

1. Diffuse (fibrillary) astrocytoma 2. Glioblastoma 3. Pilocytic astrocytoma

Drugs increasing cognition in Alzheimers are?

1. Inhibitors of acetylcholine esterase 2. Inhibitors of NMDA glutamate receptors 3. precursors of acetylcholine 4. Improvement of utilizing the precursors of acetylcholine 5. Agonists of cholinergic receptors

Name the variants of MS

1. Marburg type 2. Concentric sclerosis of Blo 3. Schilder's disease 4. Tumefactive MS 5. Neuromyelitis optica (Devic's disease) - formerly categorized as MS, not anymore. (RW? variants should be relpasing, remitting, progressive, etc RR: agree with RW

Which of these drugs can be used for macular degeneration? 1. Pegaptanib 2. Ranibizumab 3. Timolol

1. Pegaptanib 2. Ranibizumab Timolol is used in glaucoma.

Name 3 drugs used for glaucoma therapy?

1. Pilocarpine (parasympathomimetic) 2. Beta blockers - timolol 3. Prostaglandin analogues - latanoprost

1. Pregabalin acts synergecilly with opiods 2. Pregabalin is a calcium channel blocker 3. Pregabalin is a precursor of GABA

1. YES 2. Yes 3. No (?) It is an analogue of GABA, but says nothing in pp about being a precursor! Powerpoint notes: - P/Q calcium channel blockers (alpha2delta1 subunit, in CNS) - Gabapentine is a structural analogue of GABA, *does not* interact with its receptors, enhance GABA action or convert to GABA. Mechanism is unknown Pregabalin har strukturell likhet med nevrotransmittoren GABA. Virkningsmekanisme: Pregabalin bindes til en subenhet (α2-δ-protein) på spenningskontrollerte kalsiumkanaler i sentralnervesystemet.

1) The dislocation of the central structures of the brain with subdural hematoma suggests an association subarachonidalni ... 2) .. peracute brain ischemia can use the method of diffuse weighted MRI 3) for the evaluation of brain perfusion CT is an appropriate parameter MTT (mean transit time) 4) Curtailment subarachnoidani is a pathological admixture of blood in the space among the arachnoid and the pia mater

1. dislocation of brain structures is more typical for the epidural bleeding 2. True 3. True 4. True (poor english.. obsobs)

1) None of the antidepressants affect significant circadian rhythmicity 2) for the diagnosis of bipolar disorder is needed, in addition to manic of phases, present the minimum of one episode of depression 3) in depression reduces the total sleep time, but not more significant disruption of sleep architecture 4) genotype of the serotonin transporter affects reactivity to stress, not the formation of depression

2

1) A typical feature of dementia in Alzheimer's disease is an early speech disorder 2) Typical symptoms of frontotemporal dementia is a type of speech disorder 3) in Parkinson's disease disappear primarily cholinergi nigra neurons in substanitia 4) with genetic forms frontotemporanich lobar degeneration are pathogenic mutatce gene presenilin 1

2 - true 3 also true 4 - False? Presenilin 1 is associated with Alzheimers (not FTD according to newest test)

What is cyclothymia?

2 years of instable mood, several episodes of insignificant depression or hypomania

ADEM (acute disseminated encephalomyelitis) occurs in 1. AIDS patients 2. After viral infections vs PML 3. vPJC infection (viral Pneumocystis jiroveci Pneumonia) 4. white matter 5. immunocompromissed

2) After viral infections vs PML 4) White matter - Occurs after viral infections (1-2weeks) or rarely after vaccination (smallpox, rabies) - Affects white matter (AutoAb against myelin basic protein) 20% rapid death 80% full recovery

How much THC do you need to "get high" ?

2-3mg in recreational users

What is t1/2 of Zopiklone?

2-4 hours

1) quantitative disturbance of consciousness we evaluate short-TERT cognitive function (eg. MMSE) 2) patients with peroneal paresis, while walking ?hoisting and t entire leg? in the knee and hip, because seriously foot dorsiflexion 3) to give objective history, we can use of lie detector 4) simple cognitive tests are actually questions on all kinds of time orientation (date, day of week, season, eventually the current time)

2. True "walking like a rooster" 4. True 1 - wrong Quantitative - Glasgow coma scale Qualitative - MMSE

What is the function of flunarizine?

32. Flunarizine: a. Function: i. Calcium channel blocker and ii. Voltage gated Na+ channel blocker.

What is the function of dopamine?

25. Dopamine: a. Acts on β1-adrenoceptors. b. Increases HR and Force of heart contraction. c. Avoid in phaeochromocytoma pts since it can induce a catecholamine storm. d. In: i. LOW dosage it causes vasodilation and increasesthe renal perfusion. Used for incipient renal failure. ii. HIGH dosage it causes vasoconstriction and exacerbates a heart failure.

What is Fluoxetine?

33. Fluoxetine: a. SSRI. b. See citalopram.

What is the function of L-DOPA?

26. Levodopa=L-DOPA: a. Dopamine precursor. b. Treatment of Parkinson's disease. c. Mostly alleviates bradykinesia and rigidity, NOT tremor! d. Works only if the pt has still functioning dopaminergic neurons! e. Rapidly absorbed from the small intestine. Some of the drug will be broken down in the blood and peripheral tissues but a percentage passes the BBB and into the nerve cells. f. At the Nerve Cell Junction: Levodopa converted to Dopamine. g. Reliable effects last only for 5 years, after which increasing complications arise. Therefore it has to be used strategically: i. From 50mg/day slowly increased dose up to 800mg/day(even 2 mg). ii. Given in small doses throughout the day. h. R. „on-off" effect (periods of akinesia) and „end-of-dose" effect.

What is Dutasteride?

27. Dutasteride: a. Sex hormone/5 a-reductase INHIBITOR!! b. Treatment of BRH in men with enlarged prostate.

What is Escitalopram?

28. Escitalopram: a. SSRI. b. See citalopram.

What is ethosuximide?

29. Ethosuximide: a. Anticonvulsant. b. Selective T-type Ca2+ channel blocker. c. Treatment of: Absence seizures. (R. Thalamic neurones are involved!) d. Contraindicated if the pt has hypersensitivity to the drug!

1)Tremor is always a manifestation of Parkinson's disease 2) for parkinson syndrome is usually intention tremor and positive phenomenon of a clamping leg 3) meningeal syndrome accompanies both viral and bacterial meningitis 4) to syringomyelic dissociation syndrome patient has a disorder deep sensation, thermal and maintaining pain

3

1) Typical early disability in Parkinson's disease dementia of frontal type 2) inkluce huntingtin are typically in the nuclei of oligodendrocytes in atrofying nc. caudatus 3) Huntingtionova chorea is a disease with auozomalne domnantni inheritance 4) syndrome, "a foreign hand" is characteristic of Parkinson disease

3 - True 1 - false (demenita comes late)

What is Flumazenil?

31. Flumazenil: a. BZP antagonist!! b. Competitive antagonist, meaning that Higher concentrations will occupy the receptor! c. Indication: Reversal of sedative effects of BZP.(check also OTA questions..)

Biological halflife of THC?

30 hours, can be detected 2-8 days in the urine (Chronic users 14-42days)

What is Phenytoin?

30. Phenytoin: a. Anticonvulsant. b. Function: i. Na+ channel blocker and ii. Modulation of GABAergic neurotransmission. c. In the brain it inhibits the Initiation and Propagation of AP!! d. In the heart it exerts its antiarrhythmic effect by blockade of fast sodium channels. e. Treatment and prevention of seizures, but NOT for absence seizures!!

What is frovatriptan?

34. Frovatriptan: a. Antimigraine drug! b. Serotonin 5-HT1 receptor agonist. i. Mainly found in cranial blood vessels and they cause VASOCONSTRICTION_5-HT1B. ii. 5-HT1D causes a decrease in the PPE=positive protein extravasation! iii. (check oxford for ++)

Migraine funfact:

35. MIGRAINES: R. B-blockers are said to prevent a migraine, maybe because they increase serotonine in the synaptic clefts!! E.g. propanolol.

What is Furosemide?

36. Furosemide: a. Loop diuretic for chronic heart failure.

What is Gabapentin?

37. Gabapentin: a. Structural analogue of GABA.

What is Galantamine?

38. Galantamine: a. AchE Inhibitor b. Treatment of AD. R. DRG initials. c. From the roots of Galanthus rivalis.

What is Ginkgo biloba?

39. Ginkgo biloba: a. Extract of Ginkgo biloba leaves. Egb761 extract. b. Antagonizes PAF=platelet activating factor! c. Evidence of its efficacy in dementia is questionable at best very moderate effect. NOOTROPIC. d. Also for tinnitus?! e. OTC available.

What is Chlorhexidine?

45. Chlorhexidine: a. Topical antiseptic! b. Surgical scrub, hand rinse. c. Effective against G+, G- organisms, aerobes, facultative anaerobes and yeast.

What is Chlorxylenol?

46. Chloroxylenol: a. Chlorinated phenol antiseptic used in: i. presurgical preparation of skin and for cleaning wounds. ii. Treatment of bacterial, fungal and yeast infection of skin and nails.

What is glucosamin chloride?

40. Glucosamin chlorid(...) a. Isnt found in eye drops.(in test)

What is Haloperidol?

41. Haloperidol: a. Conventional Antipsychotic. b. Dopamine receptor ANTAGONIST.

What is Hexylresorcinol?

42. Hexylresorcinol: a. Phenol compound. b. Bactericidal properties, antiseptic and antihelminthic. c. Used in mouthwashes and skin wound cleansers.

What is Homatropine?

43. Homatropine: a. Anticholinergic. b. Similar to atropine, but weaker! c. Parasympathetic blockade: i. Used in opthalmic drops, MYDRIATIC. ii. Inhibitor of gastric spasms and secretion!

What is hydroxypropofol?

44. Hydroxypropofol: (internet.. 4-Hydroxy Propofol is a metabolite of Propofol (sc-202774), a phenol derivative that is suggested to modulate the sensitivity of capsaicin (sc-3577) to the transient receptor potential vanilloid subtype-1 (TRPV1) receptor)

What is Chlorpromazine?

47. Chlorpromazine: a. Conventional antipsychotic.

What is interferon?

48. Interferon: a. Naturally occuring peptide cytokines! b. Usually produced in response to viral infections. i. Antiviral actions. ii. Antiproliferative actions. iii. Immunomodulatory actions. c. IFN-α: Chronic hepatitis B or C. d. IFN-β: Treatment of remitting-relapsing MS e. IFN-β1b: for use in secondary progressive MS. f. IFN-γ: in chronic granulomatous disease g. IFN-α and IFN-γ are used in cancer treatment. h. Interferons are given by subcutaneous injection!

What is Carbidopa?

49. Carbidopa: a. Peripheral dopa-decarboxylase inhibitor. b. Co-careldopa= levodopa and carbidopa. c. R benserazide and Co-beneldopa.

Tryptans work on which recepetors?

5-HT1B 5-HT1D

What is Clozapine?

50. Clozapine: a. Atypical antipsychotics. b. Clozapine and olanzapine belong to MARTA=Multi Acting Receptor Targeted Agents. (multireceptor D antagonist) c. Treatment of Acute and Chronic Schizophrenia. d. Contraindications: i. Highly effective but can cause AGRANULOCYTOSIS!! Sometimes even lethal! So very restricted use. e. Doesnt cause as much sedation as do the conventional antipsychotics. f. Do NOT use if: i. Hepatic insuficiency. ii. Pt has Parkinsonian symptoms. iii. Phaeochromocytoma. iv. Pregnancy.

What is Lecithin?

55. Lecithin: a. precursor to Acetycholine. b. Given in AD!!

Can you use triptans if you are pregnant?

No

What is Acidum Acetylosalicylicum (aspirin)?

51. Acidum acetylosalicylicum(aspirin): a. ANTIPLATELET drug.

What is Lamotrigine?

53. Lamotrigine: a. Na+ channel blocker. b. Anticonvulsant.

What is Latanoprost?

54. Latanoprost: a. For open-angle glaucoma. b. Available in eye drops.

What is metoclopramide?

62. Metoclopramide: a. Action: i. Antagonist at 5-HT3 receptors. ii. Dopamine receptor antagonist(like domperidone)! b. Function: i. Antiemetic. ii. Prokinetic(increases gastric emptying.) c. Crosses the BBB and therefore has Extrapyramidal

What is Moclobemide?

64. Moclobemide: a. Reversible MAOI-A! b. MAO act intracellularly, SOS!! c. Treatment of PD!!

What is the decibel of loud talk?

80db Normal conversation: 60-65db, so loud talk should be somewhere in between 60-80.

What is Selgiline´s function?

87. Selegiline: a. Functions: i. Specific irreversible MAOI-B!!! ii. Antioxidant effects. iii. „Neuroprotection" iv. Inhibits degradation of DA, 20% lower doses required. b. This delays the need for levodopa by 9-12 months(same for amantadine!!): i. EARLY phase of PD: monotherapy. ii. LATER phase of PD: Adjuvant-combination of selegiline+levodopa.

What is Sumatripan?

92. Sumatriptan: a. First triptan to have been used!! b. Tmax=2-2,5hours, aka onset of action!!

NEUROSCIENCE

: )

Know everything about parkinsons syndrome

:)

Neuroradiology

:)

Pathophys

:)

Psychiatri

:)

What is lissauers tract?

= Posterolateral tract is a small strand situated in relation to the tip of the posterior column close to the entrance of the posterior nerve roots. It is present throughout the spinal cord, and is most developed in the upper cervical regions. The neurons and axons that make up Lissauer's tract are involved mainly in transmitting information from pain- and temperature-sensitive afferents to the neighboring segments of the spinal cord. Collaterals of unmyelinated and thinly myelinated axons of small primary sensory neurons (C and A-delta fibers, respectively) both ascend and descend for at least a few cord segments in Lissauer's tract

Sample test psychiatri

=)

how long must the phase of hypomania last to call it "hypomania?

>4 days

What does a lesion of the righ hemisphere look like?

? - You'll be affected on your left side - If right is your dominant hemisphere, you can get aphasia - If right is your non-dominant hemisphere, you can get neglect

Myasthenias as a group of diseases

? Autoimmune maybe? Acetylcholine release/receptor function? (Myasthenic syndromes: Myasthenia Gravis, Lambert-Eaton myasthenic syndrome)??

Vascular dementia

? In vascular dementia, changes in thinking skills sometimes occur suddenly following strokes that block major brain blood vessels. Thinking problems also may begin as mild changes that worsen gradually as a result of multiple minor strokes or other conditions that affect smaller blood vessels, leading to cumulative damage. Vascular brain changes often coexist with changes linked to other types of dementia, including Alzheimer's disease and dementia with Lewy bodies. Vascular dementia is widely considered the second most common cause of dementia after Alzheimer's disease, accounting for 20 to 30 percent of cases. No approved drugs for this. Some alzheimer´s drugs are in trial.

Somnollence is ebriety stage of alcoholic

? Maybe referring to alcohol withdrawal? In that case, insomnia is an important feature Klara: somnolence too

MS is AI against myelinic protein not against some basic glial protein??

? The trigger of MS is unknown The immune system reacts towards myelin or myelin producing cells.

In epilepsy therapy you start with low doses of medication, and slowly increase the dose?

? probably true

Glaucoma can be treated with anti _______ ??

?? It just said: wrong.. *Drugs for treating glaucoma are:* 1. Miotic drugs 2. Sympatolytic 3. Beta blockers 4. Carbonic anhydrase inhibitors 5. Osmotic diuretics 6. Prostaglandin analogues 7. Laser surgery

TDP41 is physiologic inside nucleus? TDP41 is pathology outside the cell?

????? --> Should probably be TDP43, which is normally a nuclear protein! --> so it is physiological in the nucleus --> is pathological if it is hyperphosphorylated --> Is pathological if found in the cytoplasm

a) the main sign of huntingtons d. is choreifom movements and dementia b) parkinson sy. is caused by high dopamine and low ACh c) rigidity is the main symptom of huntingtons d) chorea is a sign of CJD

A

a) area L5 radicular compression witnesses of foremost root l5 (Doesn't make sence, but the picture is too unclear to see what it says ) b) traumatic listhesis of Cervical vertebra with 1 cm endangers the patient of spastic quadriparesis c) Retention of urine after the fall on his back with normal X-ray of Lumbosacral spine, is no reason for MRI. d) herniated lumbar intervertebral disc causes spastic paresis of the limbs on its side

A B

a) Pain perception is transmitted by myelinated and unmyelinated fibers b) Biliary colic are examples of visceral pain c) Neuropathic pain is rather the result of damage to or dysfunction of the peripheral or central neural system rather than stimulation of pain receptors d) Phantom pain arises in hypoxia affected limb

A B C

a) Tizanidin is an agonist of presynaptic alpha2 receptor in the CNS, thereby blocking the release of excitatory smiokyselin(could not read what it said) and polysynaptic reflexes in Mission (I suggest to read about the drug) b) centralni muscle relaxants by different mechanisms inhibit polysynaptic signal on intraneuronal spinal level. c) to inordinately increase muscle activity central muscle relaxants are used d) Multiple sclerosis is an autoimmune disease of the CNS

A B C --> wrong D

a) according to vulnerability hypothesis people with smaller hippocampus volume have a higher risk of PTSD development after traumatic events b) feeling of isolation, emotional blunting and increased reactivity are among others signs of PTSD c) critical time for maturation of frontal cortex is 10-12y d) pruning is a process in which lesser used synapses are eliminated in order to increase elective function of the brain areas

A B D

a) Dissociative sensory loss is a condition which affective perception of a definite modality is impaired, while other modalities are preserved b) damaging the dorsal column tract cord leads to malfunction tractus pyramidalis c) dissociative disorder typically accompanies some spinal lesion d) The most likely clinical consequence of damage to the right parietal cortex is apraxia

A C (??) Really K: think they mean as in dissociative dis. as in dissociative sensory loss, not as in the psychiatric dis :) D

a) In the central hemiparesis capsular type (I'm guessing internal capsule) the patient has circumduction (a type of walking) during walking. b) patients with motor (expressive) aphasia does not answer the question because he does not understand c) patients who do not respond to pain by opening the eyes either verbally or motor, has a GCS score of 0 points d) patients say "Yest, Yest, yesterday, it, it , was Sun, Sun-day" is a type of APHASIA

A D Is D) a type aphasia or only stuttering?

a) Delirium is a disturbance of consciousness b) Depending on the neurobiology of motivational circles play a key role in the neurotransmitters dopamine and glutamate c) for social phobia is typical of anxiety in social situations that are not accompanied by somatic vegetative symptoms d) antagonism at 5-HT 3 receptor is the primary mechanism of the anxiolytic effects of alcohol

A (Not true according to most recent test, marked as false. TH) B Anxiolytic effect of alcohol is due to increased GABA! Alcohol + 5HT3 --> Nausea!

a) drugs lowering excitotoxicity can slow the progression of dementia b) tiapride is a selective antagonists of dopamine receptors c) memantine blocking NMDA glutamate receptors improves the transmission of neuronal excitation. d) increased production and decreased metabolism of beta-amyloid is one of the mechanisms leading to disability of Alzheimer's dementia

A - Drugs decreasing excitotoxicity - slows the progression of the dementia (from pp!) B - block D2,D3 used in: dyskinesia, alcohol withdrawal syndrome, negative symptoms of psychosis, and agitation and aggression in the elderly C - think false, but not sure, it protects neurons in alzheimers and it does block NMDA receptors but this would lead to decreased neuronal excitation? TH D - true

Choose the correct one a) Microangiopathy is a characteristic image of polymyositis b) Myasthenia gravis is an autoimmune disease characterized by anti NMDA circulating antibody c) Polyneuropathy in critically ill patients is a paraneoplastic syndrome associated with end stage cancer disease d) Peripheral polyneuropathy is most common in granulomatous sarcoidosis

A - False (microangiopathy is typical for dermatomyositis) B - false (AChAb) C-True (should be false according to test viewing from Sasha) D- false

a) For delirium among others are typical qualitative disturbances of consciousness, hallucinations and increase suggestibility type mikrozoopsie b) Decrease of dopamine in the nucleus accumbens underlies craving c) somatic drug withdrawal state after withdrawal of stimulants include joint and muscle pain d) On non-benzodiazepine hypnotics (zolpidem, zopiclone) there is no addiction

A - True (Really?) B - True (not acc. to our pp) C - False D - False Withdrawal from stimulants - lethargy, fatigue, bizarre or unpleasant dreams, increased appetite, slowed down mental and physical functioning, agitation, strong craving, insomnia or hypersomnia Withdrawal from *opiates* (NOT STIMULANTS!) causes the typical muscle and joint pain! K: I would def. say A, B and C are correct :) - RR: stimulants don't cause withdrawal

a) hypesthesia (tuning fork) can accompany nerve damage, root damage, dorsal column and soemthing else.. and it is in the parietal lobe b) parkinsonian sy. has increased basal postural reflexes and cogwheel phenomenon c) astereognosia means that the patient doesnt orient well in space d) in meningeal sy patient cant tilt the head, flex LE

A - false B- true C - false it's the INability to identify the object by active touch without other sensory input D - false (poor english) In meningeal syndrom patient has troubles with head tilt, and head tilt can lead to LE flexion... (I think it is written poorly but it is true. TH)

Choose the correct a) Rapid cycling bipolar disorder is defined by the presence of 6 or more mood episodes (depression, mania, hypomania) within 1 year b) Display and functional imaging methods found in depressive patients changes in the activity or volume of anterior cingulate, prefrontal cortex and amygdala c) Currently there are not known biological markers (eg. Blood tests, imaging studies of the brain etc) that would allow positively and unequivocally diagnose depressive and bipolar disorders, and would be in diagnostic criteria d) Strongest predictor of suicidal behavior in depressed patients is the presence of psychotic symptoms

A - false (4) B) Display and functional imaging methods of depressive patients found changes in the activity or volume of anterior cingulate, prefrontal cortex and amygdala TRUE C) Currently there are not known biological markers (eg. Blood tests, imaging studies of the brain etc) that would allow positively and unequivocally diagnose depressive and bipolar disorders, and would be in diagnostic criteria TRUE

What is TDP-43?

A DNA-binding protein, highly conserved Mutations in ALS and FTLD

What drug is Ropinirol ?

A dopamine agonist of "non-ergoline" class of medications. Used in treatment of Parkinson's disease and restless leg syndrome + pramipexol

What is motivation?

A drive to act

What is delusion of poverty?

A false belief of a person that he or she is impoverished or will be deprived of material posessions (financially bankrupt) Person strongly believes they are financially incapacitated.

What is the alien hand syndrome?

A rare neurological disorder that causes hand movement without the person being aware of what is happening or having control over the action. It is best documented in cases where a person has had the *two hemispheres of their brain surgically separated* (Corpus callosotomy), a procedure sometimes used to relieve the symptoms of extreme cases of epilepsy. It also occurs in some cases after brain surgery, stroke, infection, tumor, aneurysm and specific degenerative brain conditions such as Alzheimer's disease and Creutzfeldt-Jakob disease

Medio-temportal lobe lesion leads to..

Declarative memory dysfunction

Brain reward system

A strong motivational component. A collection of brain structures that attempts to regulate and control behavior by inducing pleasurable effects. Mediates the effect of reinforcement Stores info about good or bad implications of our behavior --> Modifies future behavior

Kluver Bucy syndrome - is what?

A syndrome resulting from bilateral lesions of anterior temporal lobe (including amygdaloid nucleus) Presents with: Hyperphagia, hypersexuality, hyperorality, visual agnosia and docility. Visual agnosia is an impairment in recognition of visually presented objects. Docility: Tameness and lack of fear

a) (A finger is covering the first word)... is by cannon-bard's theory considered the theory behind homeostatic central body b) (A finger is covering the first word)... associated with pain and pleasure are part of the body system homostasis c) (A finger is covering the first word)... domain consists of serotonergic pathways d) Among human emotions belongs happiness sadness anger disgust fear and surprise

A,B,C - I don't know D - true *Cannon-Bard theory* are that emotional expression results from the function of hypothalamic structures, and emotional feeling results from stimulations of the dorsal thalamus. The physiological changes and subjective feeling of an emotion in response to a stimulus are separate and independent; arousal does not have to occur before the emotion. Thus, the thalamic region is attributed a major role in this theory of emotion.

66. The minimal lethal dose of TCAs: A. 10-20x therapeutic dose B. 21-30x therapeutic dose C. 31-40x therapeutic dose D. 41-50x therapeutic dose

A. 10-20x therapeutic dose

50. In genetic research, the most promising susceptibility genomic regions associated with bipolar disorder are: A. 13q B. 52p C. 22q D. None of the above

A. 13q C. 22q

16. Diagnosis of mania is based on: A. Abnormally or persistently elevated, expansive, or irritable mood, lasting at least one week B. Increased need for sleep C. Flight of ideas or subjective experience that thoughts are racing D. Brain CT scan

A. Abnormally or persistently elevated, expansive, or irritable mood, lasting at least one week C. Flight of ideas or subjective experience that thoughts are racing

34. The three most common causes of dementia are: A. Alzheimer's Disease B. Schizophrenia C. Multi infarct dementia D. Delirium E. Lewy body dementia

A. Alzheimer's Disease C. Multi infarct dementia E. Lewy body dementia

41. Which drug is not suitable in the treatment of behavioural symptoms in elderly patients with dementia: A. Amitriptyline B. Haloperidol C. Melperone D. Tiaprid

A. Amitriptyline (causes postural hypotension, not good in elderly)

23. The four A's in schizophrenia according to Bleuer do not include: A. Arrogance B. Associational disturbances C. Affective disturbances D. Autism E. Ambivalence

A. Arrogance

1. Mood stabiliser is not: A. Biperiden B. Lithium C. Lamotrigine D. cotrimaxazole

A. Biperiden - Parkinsons disease D. cotrimaxazole - antibiotic

36. Acamprosat: A. Decreases alcohol craving B. Eliminates depression C. Eliminates psychomotor inhibition D. Is used in the treatment of anxiety disorders

A. Decreases alcohol craving

47. The following mechanisms are hypothesized to play a role in the pathophysiology of depression: A. Dysfunctional regulation of Hypothalamic-pituitary-axis together with dysregulation of hippocampus B. Impairment f neurotrophic mechanisms C. Impaired reward system D. None of the above

A. Dysfunctional regulation of Hypothalamic-pituitary-axis together with dysregulation of hippocampus B. Impairment f neurotrophic mechanisms C. Impaired reward system

37. Cannabis: A. Effects are mediated via CB1 and CB2 receptors B. Effects are mediated via D3 and D5 receptors C. Effects are mediated via HB receptor D. Is of no importance in psychiatry

A. Effects are mediated via CB1 and CB2 receptors

8. Diagnosis of schizophrenia is based on: A. Interview with the patient B. Information provided by people that know the patient C. Antidopaminergic blood test D. MRI scan of the prefrontal lobes

A. Interview with the patient B. Information provided by people that know the patient

12. Patient refuses admission to psychiatric hospital: A. Examine mental status: if the patient is dangerous toward himself or others because of mental illness the hospitalisation against their will is possible B. Examine mental status: if the patient is dangerous toward himself or others because of mental illness the hospitalisation against their will is possible only after spouses consent C. Examine mental status: if the patient is dangerous toward himself or others because of mental illness the hospitalisation against their will is possible only after contact with their lawyer D. Examine mental status: if the patient is dangerous toward himself or others because of mental illness the police are called first

A. Examine mental status: if the patient is dangerous toward himself or others because of mental illness the hospitalisation against their will is possible

1. Manic patient shouts at the doctor some vulgar words. As a first step the doctor will: A. In a calm voice, ask the patient to use acceptable language B. Leave the situation immediately C. Give the injection as aggressive behaviour might follow D. Call another staff member for help

A. In a calm voice, ask the patient to use acceptable language

46. Changes in hippocampus volume were found: A. In bipolar disorder B. In PTSD C. In ADHD D. In depressive disorder

A. In bipolar disorder B. In PTSD D. In depressive disorder Klara: studies also say ADHD

60. For diagnosis of mania crucial is: A. Inappropriate mood elation B. Excessive somnolence C. Feeling of perpetual energy D. Increased self esteem

A. Inappropriate mood elation C. Feeling of perpetual energy D. Increased self esteem

48. CRH A. Is corticotrophin releasing factor B. Is released from cerebellum during stress C. Is released from paraventricular nucleus of hypothalamus during stress D. Plays a role in affective disorders

A. Is corticotrophin releasing factor C. Is released from paraventricular nucleus of hypothalamus during stress D. Plays a role in affective disorders

4. Cognitive behavioural psychotherapy: A. Is effective in the treatment of depressive disorder B. There is a manual for therapists available C. Is a therapeutic option in delirium tremens D. Is not combined with medication

A. Is effective in the treatment of depressive disorder B. There is a manual for therapists available

71. Macromanic and grandiose delusions are typical for: A. Mania B. Depression C. Schizophrenia D. Persistent delusional disorder

A. Mania I would say *schizophrenia* as well? That's where we typically see delusions.. GDs appeared more commonly in patients with bipolar disorder (59%) than in patients with schizophrenia (49%), followed by presence in substance misuse disorder patients (30%) and depressed patients (21%).

51. In bipolar disorder can be diagnosed: A. Manic phase B. Depressive phase C. Hypomanic phase D. Mixed phase E. Anxiety phase

A. Manic phase B. Depressive phase C. Hypomanic phase D. Mixed phase

1. Non pharmacological therapeutic intervention for patients with Alzheimer's disease include: A. Psychiatric rehabilitation B. Carrying out the activity of the patient C. Working with caregivers and family members D. Stay at a health resort

A. Psychiatric rehabilitation B. Carrying out the activity of the patient C. Working with caregivers and family members

21. Treatment of anxiety disorders include: A. SSRIs B. Benzodiazepines C. Cognitive behavioural therapy D. Lithium

A. SSRIs B. Benzodiazepines C. Cognitive behavioural therapy

3. Suicide may occur in: A. Schizophrenia B. Bipolar disorder C. Depressive disorders D. Antisocial personality disorder

A. Schizophrenia B. Bipolar disorder C. Depressive disorders I would think all 4.. Suicide can occur in all people

19. SSRI is: A. Selective serotonin reuptake inhibitors B. Nootropic drug C. Second serotonin research information D. Group of antidepressants

A. Selective serotonin reuptake inhibitors D. Group of antidepressants

45. Hallucinogens act mainly via receptors of: A. Serotenergic system B. Dopaminergic sstem C. Noradrenergic system D. GABA Systme E. Glycine system

A. Serotenergic system Eva: PPT says that dopamin is also involved

58. Physical (somatic) symptoms of depression include: A. Significant loss of appetite B. Weight loss C. Significant decrease in libido D. Impaired perception E. Illusions

A. Significant loss of appetite B. Weight loss C. Significant decrease in libido

25. Dependence syndrome is defined as a: A. Syndrome characterised by preferential abuse of specific psychoactive substance with higher priority given to drug use than to other activities and obligations B. Syndrome characterised by hallucinations C. Syndrome characterised by thought disorders D. Syndrome characterised only in association with alcohol use

A. Syndrome characterised by preferential abuse of specific psychoactive substance with higher priority given to drug use than to other activities and obligations

CADASIL - type of inheritance? (AD, AR, X-linked?) - Increased risk of infarcts?

AD dominant disease Increases risk for infarcts "Cerebral autosomal dominant arteriopathy with subcortical infarcts and leukoencephalopathy" Associated with progressive dementia, mood disorders, migraine, and recurrent subcortical cerebral infarctions. MA: (from PP) Subcortical *vascular dementia* (Binswanger ́s disease) Microinfarcts White matter lesions (leukoencephalopathy) Amyloid angiopathy

Types of MCI

Amnestic type (aMCI) - memory loss without dementia, 80% develop into Alzheimer Non-amnestiv type (naMCI) - non-Alzheimer dementia

What are the common adverse effects in ALL SSRI´s?

AE common in ALL SSRIs: • Weight gain. • Sexual dysfunction. Loss of libido.NOTE that depression also causes loss of libido, so combo..not good! • Serotonin syndrome(tremor, agitation, diarrhoea, tachycardia) • Inhibition of CYP2D6(codeine).

In which disease do we find Hirano bodies?

ALS (no??) Alzheimers and Creutzfeldt Jacobs RR: Alzheimers for sure (BRS pathology) - any other ideas?? Pubmed: "Hirano bodies are initially observed in Guamanian (Chamorro) patients with *amyotrophic lateral sclerosis* and parkinsonism-dementia complex". Hva tenker dere her?! Ida E: Finnes flere artikler om hirano bodies i forbindelse med ALS, så da er vel alle 3 riktige? Synne: enig med Ida, funnet flere artikler der både ALS og hirano bodies er connected.

Types of long-term memory

Declarative (explicit) Non-declarative (implicit)

HIV

Aseptic meningitis Subacute encephalitis Vacuolar myelopathy

Ms affects males>females?

No females more

What are Bleulers 4 A's ?

Affect Autism Ambivalence Associations He presumed that these four A's were at the core of the schizophrenia and were fundamental aspects of the disorder.

Sporadic CJD

Affect older people *Dementia, ataxia/myoclonus, triphasic EEG* Protein 14-3-3

Emotion

Affective state of consciousness, during which we experience states like joy, fear, hatred, etc.; emotions are visible A response to a competent trigger (object or situation), processing is conscious or unconsious (auto-pilot) rooted in the reward/punishing mechanism

PTSD - neuronal changes

Abnormal amygdala reactivity, increased cingular cortex. decreased mPFC

Qualitiative memory disorders (paramnesia)

Abnormalities in accuracy and in a sense of security (inaccurate, distorted and false memories) Examples: Deja vu Pseudologia phantastica - fanasty -> "real memory" Ecmnesia - false location of experience in time Prosopagnosia - inability to recognize known faces Confabulation - false memory traces Pseudoreminiscence - filling in gaps in memory (witnesses in crime), frontal lobe damage Hallucination of memory Reduplicative paramnesia - duplication of places, faces

Polio

Affinity to spinal motor neurons in anterior horn --> *muscular atrophy*

Underlying mechanism of drug addiction

Abuse of the natural function of brain reward system

How many people suffer from tinnitus in CR?

According the pp 10-15 % (8%) *800 000 people* - (There is appx 10million people in CR) --> SYKT MANG!! (50 000 is wrong!!)

Name a drug improving the utilization of acetylcholine?

Acetyl-L-Karnitin

What type of drug is Galanatmine?

Acetylcholine esterase inhibitor

What is Biperiden?

Acetylcholine receptor antagonist (like Benzatropine!!)

Amygdala role in emotions

Activated during positive and negative emotions, acquiring new emotional experiences

Explain mechanism of drug addiction?

Acute effect - increased DA in mesolimbic system, changes in cell signalization and genes Transition to addiction - Role of DA and glutamatergic system - Neuroplastic changes - still reversible Addiction - irreversible or long-lasting changes - main changes in glutamate system (Prefrontal cortex --> ncl. accumbens)

Stress in aldulthood

Acute stress increases GR and improves retrieval of emotional salient information, but worsens retrieval of neutral information Chronic stress causes decreased function and size of hippocampus, seen in depression, PTSD

Where does the effects of nicotine come from?

Acutely increased DA in mesolimbic circuits.

How is Biperiden administered?

Administration routes: i. I.m.20 minutes to effect. ii. I.v. if urgent, life-threatening dystonia. d. NOTE: If patient has oculogyric crisis NOT responding to anticholinergics give CLONAZEPAM i.v.

Lesion of posterior part of parietal lobe can cause?

Agnosia - inability to process sensory information Dysgrafia Dyscalkulia (Gerstmann syndrome) (wiki - apraxia and hempispatial neglect) RW: Fundamentals of Neuro: Parietal lobe syndrome manifests itself in somatosensory deficits and a variety of neuropsychological abnormalities: The most prominent sign is usually a* hemisensory deficit.* Lesions of the language-dominant hemisphere (usually left) can cause left/right confusion, *finger agnosia, acalculia, and agraphia (These 4 = Gerstmann syndrome), and/or astereognosia.* Lesions of the nondominant hemisphere (usually right) can cause anosognosia (see above). With regard to motor function, there are often poorly coordinated, ataxic hand and foot movements on the side opposite the lesion. With regard to somatic sensation, there may be neglect for the contralateral half of the body (so-called extinction phenomenon: raw sensation is intact blaterally, but if the examiner touches the patient simultaneously and equally intensely at mirror-image sites on the two sides, the patient will report having felt something on one side only). Deep-seated lesions may produce contralateral ho- monymous lower quadrantanopsia or hemianopsia, or else only visual neglect for the contralateral hemifield.

26.Basal ganglia function 1. Voluntary motor regulation 2. procedural learning relating to routine behaviour or habits 3. eye movements 4. cognitive emotional functions

All are correct. Internet: the basal ganglia are responsible for *voluntary motor control*, *procedural learning*, *eye movement*, and *cognitive, emotional functions*.

a) IPSP occurs by GABA binding to the postsynaptic potentials b) generalized tonic-clonic seizures are characterized by loss of awareness c) wave spike rhythm on EEG is typical in some leads for absent type of epilepsy d) absence seizures are characterized by loss of consciousness

All are correct. a) IPSP occurs by GABA binding to the postsynaptic potentials (Binding to post synaptic potential? Typing error? should be receptor.) b) generalized tonic-clonic seizures are characterized by loss of awareness c) wave spike rhythm on EEG is typical in some leads for absent type of epilepsy d) absence seizures are characterized by loss of consciousness

1) tetrazepam is among benzodiazepine 2) The typical side effects with muscle relaxants centrally fatigue, sedation, sometimes increase instability 3) alteplase or treatment must be initiated within 3 hours of the onset symptoms of acute ischemia stroke 4) is Clopidogrel among the antiplatelet medicines, ADP receptor blocker and phospholipase C

All correct 1. True 2. True 3) True (3-6hours) 4. True

Act of nicotinic receptors in brain 1. enhance cognitive function 2. induce neuronal development learning and memory formation 3. Has a role in reward

All correct? Research: nAChR are involved in several important aspects of *cognitive function* including attention, *learning and memory*. Chronic nicotine use induces neuroadaptations in the brain's *reward system* that result in the development of nicotine dependence. Nicotine directly enhances dopamine levels in the mesolimbic system by interacting with nAChRs on the dopaminergic neurons and causing them to release more of the neurotransmitter.

What is alexia?

Also called *word-blindness* and refers to the loss of the ability to read and comprehend meaning of written words. Patient can often write, but not read often due to infarct of post.cerebral artery

Groups of neurodegenerative diseases

Alzheimer disease and aging Tauopathies - Alzheimer Synucleinopathies - Parkinson Trinucleotide repeat disorders (Huntington) Prion disorders - CJD Motor neuron disorders - ALS Frontotemporal degeneration Other

Braak staging is used in which disease?

Alzheimers and Parkinson's disease (not Lewy body dementia)

Galantamin is used for which disease?

Alzheimers disease Central AChEsterase inhibitor and binds to ACh receptors

What is Amantadine?

Amantadines primary function is: - It enchances the RELEASE of DOPAMIN - It also inhibit reuptake of dopamin - Glutamatantagonist som virker over NMDA-receptorene - It is a dopaminergic, noradrenergic, seratonergic, blocks MAO-A and NMDA receptors.

What does folic acid deficiency lead to?

Anemia Birth defects Fatigue, grey hair, brittle nails, mouth sores, tongue swelling, growth problems

What are the six basic emotions?

Anger Disgust Fear Happiness Sadness Surprise

Where is the lesion if you have nominal aphasia?

Angular gyrus (a region of the brain in the parietal lobe) Parietal lobe Temporal lobe Anomia is caused by damage to various parts of the *parietal lobe* or the *temporal lobe* of the brain. According to powerpoint: Nominal dysphasia o Cant name objects o Lesion in *angular gyrus* o "That thing that I write with" in stead of pen

Signs of epidural bleeding

Anisocoric pupils, mydriasis on affected side

Alcohol affects dopamine indirectly by agonism or antagonism of NMDA receptors?

Antagonist of NMDA Remember: alcohol is a depressant (and NMDA is positive) Alcohol: depress NMDA or enhance GABA --> either way you always depress.

what kind of drug is Betahistine?

Anti-vertigo drug Works as an antagonist on H3 receptor - increasing the levels of neurotransmitters histamine, acetylcholine, norepinephrine, serotonin, and GABA released from the nerve endings. Works as an weak agonist of H1 receptor - vasodilation of blood vessels in inner ear

What is Distigmine?

Anticholinesterase inhibitor.

What is Carbamazepine normally used for?

Anticonvulsant and mood-stabilizing drug used primarily in the treatment of epilepsy and bipolar disorder, as well as trigeminal neuralgia.

Herpes zoster oticus is treated with ?

Antivirotics(Acyclovir)

Primary progressive aphasia

Aphasia *without* structural background, gradually decline of mental abilities After 2 years it progress to dementia Types: - Primary non-fluent aphasia - Logopenic progressive aphasia - Semantic dementia

How long must the phase of mania last, to call it mania?

At least 1 week, and it must disrupt normal functioning daily life.

Mechanism of occlusion

Atherosclerosis Embolism Thrombosis (hypercoaguble state, vasculitis, trauma) Venous thrombosis (intracranial inflammations)

Consequence of internal carotid artery occlusion

Hemispheric infarct

Traumatic neuropathies

Axonal degeneration Traumatic neuroma (benign, painful growth)

63. Hallucination is: A. Disturbance of thinking B. Disturbance of perception C. Disturbance of consciousness D. Obsolete term for illusions

B. Disturbance of perception

a) metastatic tumors make up 50-60% of all brain tumors b) pilocytic astrocytoma is a typical histological form of astrocytoma in children c) meningioma is a typical brain tumor in children d) Lisch nodules are characteristic for Sturge-Weber sy.

B - Metastatic brain tumors: 20-50% (25-30% according to most recent test. TH) - Lisch nodules are found in NF1 - Meningiomas are typically benign & in adults B is correct but has been marked wrong on the most recent test correction TH

a) hypacusis without seizures can only be caused by the middle ear b) amaurosis fugax occurs during the insufficient blood flow into the ophthalmic a. c) bleeding into the upper brainstem can cause ipsilat. lesion of the 3rd cranial nerve with contralat hemiparesis d) drop of soft palate on one side is the result of chorda tympani disfunction

B C- was marked as false, but I think it is true Synne agrees with C) --> this is also known as Webers syndrome (superior alternating hemiplegia). maybe it is false because it says "hemiPARESIS" and not hemiPLEGIA?!

Which of the following are correct a) Attention is controlled by two frontoparietal networks: the ventral system is created by the frontal eye field (FEF) and the intraparietal sulcus (IPS) b) Memory is the ability to store and recall information, learn new skills and adapt the behavior based on past experience c) Vigilance is the ability to switch the attention and focus to multitask for longer periods d) A common feature of classical and operant conditioning is the association between conditioned and unconditioned stimulus

B Vigilance: the ability to maintain attention and alertness over prolonged periods of time.

a) CADASIL is AR hereditary disorder of small vessels that leads to prosessive vascular dementia b) secondary vessel spasms are a relatively common complication of subarachnoid bleeding c) rupture of an aneurysm of brain vessel wall leads to characteristic subarachnoid bleeding d) rupture of aneurysm of brain vessel wall leads to characteristic epidural bleeding

B - correct (you can use Ca2+ for prophylaxis) C - correct Meningeal arteries are usually ruptured in epidural bleedings.

Which of the following is right a) amygdala is implicated in aversive conditioning in the neurobiology of dependency b) Buprenorphine is a partial agonist of the opioid receptors c) main mechanism of action of stimulants is washing out of acetylcholine and adenosine out of glutamatergic neuron d) Alcohol increases the leaching of dopamine in the nucleus accumbens primarily indirectly, partly passed through the reward system

B - marked as false.. But it is *True* True: d) Alcohol increases the leaching of dopamine in the nucleus accumbens primarily indirectly, partly passed through the reward system

a) delusion is a sign of panic anxiety disorder b) during delusion formation doperminergic mesolimbic system takes part c) The patient insists that he is being followed by secret police. If the other criteria of delusions are fulfilled.... d) rumination delusion means that that patient doesn't see the point in living anymore

B is correct! If C) says something about non-bizzare delusions it should be correct. - Rumination: dwelling on negative thoughts and feelings.

22. Chlorpromazine: A. Is a nootropic B. 1st generation antipsychotic C. Neuroleptic D. Typical antipsychotic

B. 1st generation antipsychotic C. Neuroleptic D. Typical antipsychotic

61. Chief complaint in psychiatric examination refers to: A. Acute somatic conditions B. Acute mental conditions C. Mental conditions currently and in the past D. The time schedule of the hospitalisation

B. Acute mental conditions

56. Mirtazapine is: A. Beta blocker B. Antidepressant C. Antipsychotic D. Statin E. None of the above

B. Antidepressant

43. Affective (mood) Disorders: A. Typically occur only in the elderly B. Are characterised by pathological mood associated with somatic and psychomotor symptoms C. Never include psychotic symptoms D. Can lead to suicide

B. Are characterised by pathological mood associated with somatic and psychomotor symptoms D. Can lead to suicide

33. Opioid Addicts: A. Do not use other drugs B. Are often concomitantly addicted to benzodiazepines C. Do not use I.V drugs D. Have destroyed septi nasi E. Do not steal

B. Are often concomitantly addicted to benzodiazepines

67. What would you do if a patient is verbally hostile and abusive towards you? A. Swear at them too B. Ask them kindly to speak properly C. Leave the situation D. Immediately give sedating injection

B. Ask them kindly to speak properly

59. Antichollinegic effects of psychotropic drugs: A. Are not relevant in clinical practice B. Can be central C. Can be peripheral D. Pose a risk, especially in elderly patients

B. Can be central C. Can be peripheral D. Pose a risk, especially in elderly patients

6. Symptoms of panic disorder do not include: A. Panic attacks B. Delusions C. Anticipatory anxiety D. Palpitations, sweating, sensations of shortness of breath

B. Delusions

69. For personality disorders is typical: A. Abrupt onset B. Development from early childhood and adolescence C. Higher prevalence in females D. Treatment response to specific psychotropic drugs

B. Development from early childhood and adolescence

62. A 17 year old patient brought to the mental hospital by the parents. What should you do? A. First speak with the patient and send parents home B. First speak with patient and ask his permission to obtain information from parents C. At first invite all the family into your office D. Speak with the parents first, then the parent and send parents home

B. First speak with patient and ask his permission to obtain information from parents

9. Disorientation to time is: A. Frequent symptom of schizophrenia B. Frequent symptom of delirium C. Part of delusion D. An intrusive thought (an obsession)

B. Frequent symptom of delirium

10. Patient with Obsessive Compulsive Syndrome: A. Has delusions and hallucinations B. Has intrusive thoughts resulting in anxiety that provoke repetitive behaviour C. Has disturbances of memory D. Cannot be treated

B. Has intrusive thoughts resulting in anxiety that provoke repetitive behaviour

39. Vascular dementia: A. Has usually an insidious onset with slow gradual deterioration B. Has usually an abrupt onset with stepwise deterioration C. The risk factors for vascular dementia are: history of stroke, high blood pressure, diabetes D. There is no way to prevent vascular dementia

B. Has usually an abrupt onset with stepwise deterioration C. The risk factors for vascular dementia are: history of stroke, high blood pressure, diabetes

5. Electroconvulsive therapy: A. Is not used anymore B. Is effective in the treatment of severe depression C. Is effective in the treatment of catatonic schizophrenia D. Is allowed only with presence of anaesthesiologist

B. Is effective in the treatment of severe depression C. Is effective in the treatment of catatonic schizophrenia D. Is allowed only with presence of anaesthesiologist

40. There are several characteristic neuropathological changes found in the brain in Alzheimer's Disease: A. Microinfarcts B. Numerical atrophy of neurons C. Neuritic plaques D. Neurofibrillary tangles

B. Numerical atrophy of neurons C. Neuritic plaques D. Neurofibrillary tangles

35. Which substance does not belong to Acetylcholinesterase inhibitors: A. Donepezil hydrochloride B. Olanzapine C. Rivastigmine D. Galantamine

B. Olanzapine (zyprexa) - dopamine antagonist - schizophrenia and bipolar disease

49. Neurotrophic factor: A. This term is not used B. Regulate neuronal growth and differentiation during development C. Regulate plasticity and survival of neurons and glial cells D. One of them is brain derived neurotrophic factor (BDNF)

B. Regulate neuronal growth and differentiation during development C. Regulate plasticity and survival of neurons and glial cells D. One of them is brain derived neurotrophic factor (BDNF)

44. Gyrus cinguli: A. Is irrelevant from the point of view of affective disorders B. Represents a crossroad of emotions processing cognitive functions C. Integrates visceral and affective information with attention mechanisms D. None of the above

B. Represents a crossroad of emotions processing cognitive functions C. Integrates visceral and affective information with attention mechanisms

11. Patient complains of insomnia, GP should: A. Prescribe hypnotics immediately B. Search for symptoms of depression C. Make referral to psychiatrist D. Not trust the patient and ask husband or wife

B. Search for symptoms of depression

2. Neuroleptics: A. This name is not used anymore B. This name is used in clinical practice C. 1st generation antipsychotics is a synonym for neuroleptics D. Typical antipsychotics is a synonym for neuroleptics

B. This name is used in clinical practice C. 1st generation antipsychotics is a synonym for neuroleptics D. Typical antipsychotics is a synonym for neuroleptics

Non-declarative (implicit) memory - behaviorism

Behavior = only conditioned response to a stimulus, including speech and social behavior Accidental response -> reinforcement -> deliberative response

Attention

Behavioral and cognitive process of selective concentration on a discrete aspect of information, while ignoring other perceivable information.

What are the compulsions?

Behaviors repeated in order to reduce the anxiety or neutralize the obsession They are considered pointless and annoying, but resisting them makes the anxiety worse.

Meningiomas

Benign (grade I), adults, on the brain surface extracranially, slow growth progression Rare atypical (II-III) or anaplastic variant (IV)

What is Diazepam?

Benzodiazepine. - It is a long-acting BZP, which basically means Worse Hangover effect. - Not indicated for long-term treatment. (R. Tachyphylaxis) - Not used for anxiety, short-acting BZPs are preferred!

What is pseudobulbar syndrome?

Bilateral lesion of cortico-bulbar tract

What is rapid cycling?

Bipolar patient with more than 4 episodes of changing mood per year (mania-depression-mania-depression)

What is a bizarre delusion?

Bizarre delusion: Delusions are deemed bizarre if they are clearly implausible and not understandable to same-culture peers and do not derive from ordinary life experiences. An example named by the DSM-5 is a belief that someone replaced all of one's internal organs with someone else's without leaving a scar.

What is the function of atypical antipsychotics?

Block dopamine pathways Simen: D2 + 5-HT2A receptor antagonism (Less EPS and sedation, more weight gain and cardiovascular) RR: atypical antipsychotics 5-HT >>D2

Duchenne muscular dystrophy vs Becker muscular dystrophy

Both are XR disordes of the dystrophin gene Duchenne - no dystrophin Becker - abnormal dystrophin, less severe

What does an occusion of vertebral or basilar artery often result in?

Brain stem infarction

a) with bulbar syndrome is present central paresis n. XII b) by paresis n. VI patient has divergent strabismus with diplopia c) unilateral hyposmia or anosmia may indicate a tumor at the base of the frontal lobe d) Paresis of n.III causes lagophthalmus on the affected side

C

a) by paresis n. III seriously affected the movement of the eyeball nasally, upward and downward b) positive reflexes labial, suction and gripping are physiologically at any age, even in old age c) When damaged side mixed system (which apparently is CN IX, X, and XI) on one side is homolateral bunk dropped from shoulder d) peripheral paresis n. VII include hemihypestesia of the face

C A - false, because the n.IV innervates superior oblique muscle, which can cause the movement of the eye downward and nasally. K: I would say a) is correct: both inf. rectus and med. rectus is deinnervated, and you will have severe trouble with looking to your nose and down as well as up, and have divergent strabismus -- RR agrees

a) enlargement of lat ventricles in schizophrenia correlates with good treatment response b) absence of gliosis supports the hypothesis about neurodegenerative cause of schizophrenia c) people that develop schizophrenia show higher occurence of birth complications d) positive symptoms of schizophrenia are connected to the change of blood circulation in the parahippocampal gyrus

C- correct D - correct ( https://en.wikipedia.org/wiki/Mechanisms_of_schizophrenia ) Absence of gliosis is a supporting factor for a neurodevelopmental disorder, not a neurodegenerative disorder! (OMG - the details!)

Visceral pain (from organs) are conducted via which type of pain fibers?

C-fibers

31. Stimulants are: A. Heroin, psilocin B. LSD, muscarine C. Amphetamine, cocaine, caffeine D. Skopoloamine, THC, atropine E. None of the above

C. Amphetamine, cocaine, caffeine

72. All antipsychotic drugs: A. Have high abuse potential B. Are tolerated without side effects C. Block D2 receptors D. Have very similar side effect profiles

C. Block D2 receptors

20. Panic anxiety disorder does not include: A. Sudden panic attack B. Anticipatory anxiety C. Hallucinations D. palpitations

C. Hallucinations

68. The word "anhedonia" means: A. Inability to communicate in general B. Disturbance of locomotion C. Inability to feel joy D. Lack of proper emotions

C. Inability to feel joy

18. Fluoxetine: A. Is a 2nd generation antipsychotic B. Has an extremely short half life C. Is an antidepressant D. Is also used in the treatment of anxiety disorders

C. Is an antidepressant D. Is also used in the treatment of anxiety disorders SSRI, antidepressant drug, with long T1/2

13. Asking the patient about suicidal thoughts: A. Is dangerous as the patient might perceive it as a suggestion and a possible solution for his problems B. Is possible, but only in certain psychiatric diagnosis's C. Is necessary in all psychiatric diagnosis's D. Is necessary only in depression

C. Is necessary in all psychiatric diagnosis's

42. Delerium tremens: A. Is not treated, it is let go of B. Is treated with amitriptyline C. Is treated with clomethiazol D. Is treated by bed rest

C. Is treated with clomethiazol Acts like a sedative, hypnotic, muscle relaxant and anticonvulsant. Enhance GABA-A function Delirium tremens (DTs) is a rapid onset of confusion usually caused by withdrawal from alcohol. When it occurs, it is often three days into the withdrawal symptoms and lasts for two to three days. People may also see or hear things other people do not. Physical effects may include shaking, shivering, irregular heart rate, and sweating. Occasionally, a very high body temperature or seizures may result in death. Alcohol is one of the most dangerous drugs to withdraw from.

52. In the treatment of mania can be used: A. Antidepressants B. Acamprosate C. Lithium D. Atypical (2nd generation) antipsychotic

C. Lithium D. Atypical (2nd generation) antipsychotic Acamprosate-used in alcohol dependence Antidepressants are CONTRAINDICATED!

17. Treatment of schizophrenia is based: A. Only on pharmacotherapy B. Only on non pharmacological interventions C. On combination of pharmacological and non pharmacological interventions D. Only on hospitalisation

C. On combination of pharmacological and non pharmacological interventions

55. Advantages of SSRIs are: A. Good antipsychotic efficacy B. Good antimanic efficacy C. Simple dosing D. Safety in overdose

C. Simple dosing D. Safety in overdose

64. A situation in the inpatient psychiatric department occurs. Patient is sitting under the table, his hands over his ears and refusing to answer your questions. What do you do? A. Ask staff to remove patient to their room B. Instruct staff to apply injection C. Try to communicate with them by calm talk D. Try to communicate by sharing the same pace with the patient

C. Try to communicate with them by calm talk

NOTCH 3 mutations on chromosome 19 causes risk of what?

CADASIL Cerebral autosomal dominant arteriopathy with subcortical infarcts and leukoencephalopathy

Examples of vascular dementia - microangiopathy

CADASIL Subcortical vascular dementia (Binswanger´s disease) Microinfarcts White matter lesions (leukoencephalopathy) Amyloid angiopathy

What are the adverse effects of Amantadine?

CNS effects: - Confusion(R.elderly). - Psychiatric effects: euphoria, anxiety and agitation. - Involuntary movements. Peripheral effects: - Nausea and vomiting.(R. Domperidone=dopamine antagonist acting only in the periphery! Dont confuse with risperidone!!!) - HYPOTENSION, cardiac arrhythmias. - Peripheral oedema. - Leukopenia. - Can cause somnolence and sudden sleep onset episodes. - Reduce dose in pts with moderate renal insufficiency.

What is a good mnemonics for CYP450 inducers?

CRAP GPS induce me to madness!! Carbemazepines Rifampicin Alcohol (chronic) Phenytoin Griseofulvin Phenobarbitone Sulphonylureas

Tumors from neurons (rare)

Can arise anywhere, in all ages, often mixed with glial component 1. Gangliocytoma/ganglioglioma - grade I 2. Dysembryoplastic neuroepithelial tumor - grade I, children, cortex 3. Neuroblastoma - grade IV, with Homer Wright rosettes

Folic acid deficiency leads to?

Can cause depression, anemia, birth defects, problems with tissue with fast regeneration

What are the 3 most common causes of death in the developed countries?

Cardiovascular diseases Malignant tumors Cerebrovascular diseases

Chronic ischemia

Caused by multiple stenosis and occlusions of small arterial branches Result: brain atrophy - thin cortex/gyri, internal hydrocephalus, vacou, cribose and lacunar status (BG and centum semiovale)

What does lesion of subthalamic nucleus cause?

Causes hemiballismus

Are the central muscle relaxants or peripheral muscle relaxants most beneficial in Huntington's disease?

Central muscle relaxants

What is CADASIL?

Cerebral Autosomal Dominant Arteriopathy with Subcortical Infarcts and Leukoencephalopathy - Infarct in subcortical areas and it affect the white matter, due to a vascular abnormality A hereditary type of vascular dementia (chr. 3)

Causes of pseduobulbar syndrome ? (Supranuclear)

Cerebrovascular disease, arteriosclerosis (multi-infarct conditions)

What is moria?

Childish euphoria Cheerful excitement Euphoric behavior, such as frivolity and the inability to act seriously.

Pediatric vs adult tumor location

Children: infratentorial, posterior cranial fossa Adults: supratentorial, telencephalon

CADASIL - mutation on which chromosome and gene? - Clinical signs? - How is it diagnosed?

Chromosome 19, notch 3 Migrains with aura, repeated strokes, subcortical predominant form of dementia Diagnosed by skin biopsy + immunohistochemistry or electrons microscopy Cerebral autosomal dominant arteriopathy with subcortical infarcts and leukoencephalopathy

Amnesia

Complete or patchy Retrograde - before lesion, gradual effects (trauma, concussions) Anterograde - after lesion (AD, P synthesis blockers, alcohol blacouts)

Hypomnesia

Coding deficit Retention deficit -> recognition intact, impaired recall

Cognitive behavioral model components

Cognitive appraisal of situation Physiological response, behavior and, facial expression Emotion

Nociception and memory

Common menchanism of synaptic plasticity Early treatment of acute pain helps to prevent chronicity Sudden amnesia leads to analgesia

Bacterial agents - examples

Common: Streptococcus, staphylococcus, E. coli, Neisseria meningitidis Rare: Treponema pallidum (syphilis), rickettsia • Newborns: E. coli, Streptococcus • Todlers: H. influenzae • Children/young adults: N. meningitidis (G-) - endotoxin -> bleeding (DIC), shock, sepsis

What might "help" the patients with his obsessions?

Compulsions

Parenchymal brain injuries

Concussion Contusion Laceration (penetrating/open trauma, fatal) Diffuse axonal damage

Lesions of VI causes what kind of strabismus?

Convergent strabismus

Antipsychotics can be used as antiemetics

Correct

a) capsaicin in initial treatment can induce burning sensation in place of application b) we use SNRI for example Venlafaxin for neuropathic pain c) in neuropathic pain we use pregabalin or venlafaxin d) capsaicin is a selective antagonist of TRPV1 receptor

Correction: A,B,C

a) vertical gaze palsy occurs in the lesion of the dorsal part of mesencephalon (tectum) b) deviation of both eyes to one side means bilateral paresis of 3rd cranial nerve c) in the central lesion of extraocular mm. innervation of combined movements of both eyes the patient doesnt have diplopia d) in occipital lobe defect on one side the patient has homonymous hemianopsia on the contralat. side

D Ida: a) should be correct as well RR agrees

Memory deficit in MCI

Declarative memory - acquisition and retention Episodic and semantic memory Greater impairment for newer events and facts Confabulations

a) patient with social phobia is not interested in others company b) panic attacks occur only within panic disorder c) untreated panic attack disappears within 30min d) all existing places and situations can be the cause of anxiety during agoraphobia

D - correct I also think C) should be correct. If treated it can last shorter (only minutes), but it usually lasts MAX 30min according to ppt. What do you think? K: I agree with c), not sure about d? ALL situations shouldn't be a place for anxiety -- if you get anxiety in your own bed for example, it shouldn't be agoraphobia??

28. Except for an interview with the patient, diagnosis of alcohol dependence is based on: A. Somatic examination B. Questionnaires that help validate suspicion of alcohol dependence C. Laboratory examination of blood and urine D. A and C are correct E. Nothing is correct

D. A and C are correct

57. Amitriptyline can cause: A. Urinary retention B. Orthostatic collapse C. Impaired heart conduction D. All of the above E. None of the above

D. All of the above

30. Buprenorphrin is: A. Anti alcoholic treatment B. Antidote for neuroleptic intoxication C. Partial agonist of opioid receptors D. Anxiolytic drug

D. Anxiolytic drug (?? wrong?) Olav: C) is correct (I find little info about anxiolytic) E: Can be used in treatment resistant OCD, but I agree, C should be the correct answer!

15. Benzodiazepines: A. Are not anti-anxiety agents B. Are not used in psychiatry C. Bind to specific sites on GABA A receptors D. Are sometimes used as an additional therapy for schizophrenia

D. Are sometimes used as an additional therapy for schizophrenia C(??): "Benzodiazepines enhance the effect of the neurotransmitter gamma-aminobutyric acid (GABA) at the GABAA receptor"

7. Symptoms of schizophrenia do not include: A. Problems with memory B. Auditory hallucinations C. Delusions D. Attention deficit hyperactivity

D. Attention deficit hyperactivity Is the same as ADHD!

29. Delirium tremens: A. Is rarely seen in clinical practice recently B. Is not dangerous C. Is developing only in small children D. Can lead to death

D. Can lead to death Delirium tremens (DTs) is a rapid onset of confusion usually caused by withdrawal from alcohol.

26. Acute reinforcement induced by cocaine is mediated via: A. Serotonergic system B. Glutamatergic system C. Cocaine doesn't induce reinforcement D. Dopaminergic system

D. Dopaminergic system

24. Following diagnosis does not belong to psychiatric classification associated with substance abuse: A. Acute intoxication B. Harmful use C. Dependence D. Pyromania E. Dementia

D. Pyromania (=obsessive desire to set fire to things)

Herpes simplex

DNA herpes virus Encephalitis with hemorrhage and necrosis Inclusions in nucleus of neurons and glial cells

What should Amitryptiline NOT be combined with?

DONT USE with MAOI!!!(leave 2-3 weeks apart)

Declarative memory deficit - causes

Damage to MTL and PFC stuctures (Alzheimer, lobectomy) Damage to basal telencephalon (cholinergic nucleus basalis Meynerti) Damage to diencephalic stuctures (mammillary bodies) - Korsakoff (thiamine deficiency) - confabulations, but IQ is preserved

Characterisitics of neurodegenerative diseases

Death and progressive loss of neurons in distinct areas of CNS

Ventromedial prefrontal cortex importance

Decision making Motivation Reward sensitivity

Is hippocampus most important for declarative or non-declarative memory?

Declarative

Habituation

Decreasing behavioral response to stimuli

Definition of bipolar 1 disorder?

Defined by 1. manic or mixed episodes that last at least seven days or by 2. manic symptoms that are so severe that the person needs immediate hospital care. Usually, depressive episodes occur as well, typically lasting at least 2 weeks.

Definition of bipolar 2 disorder?

Defined by a pattern of depressive episodes and hypomanic episodes, but no full-blown manic or mixed episodes.

What is the life threatening syndrome in alcohol withdrawal called?

Delirium tremens

What did Kraepelin call schizophrenia?

Dementia praecox

Pathologic features of MS

Demyelinated plaque in a discrete region in the white matter or junction to gray matter - firm, < 2 cm - will not stain with luxol fast blue - active and inactive plaques of different histological ages - perivascular lymphocytic infiltrate associated with macrophage infiltration and reactive astrocytosis

Postnatal stress

Depressed mother -> higher HPA axis reactivity Adolescence is the period where the stress consequences become visible. Adolescence with low SES have higher GR levels.

What is citalopram used for? Name other drugs in this class:

Depression Panic disorder GAD and PTSD Should not be combined with MAOI It is an SSRI Fluoxetine, sertraline, escitalopram..

What is dysthymia?

Depressive neurosis Mix of anxiety and depression

How does serotonin correlate with agression?

Destruction of serotonergic neurons in the forebrain facilitates aggressiveness. SSRI decrease agressive behaviour.

Lesion in CN III causes what? - Deviation outwards or inwards? - Other symptoms?

Deviation out and down Ptosis Divergent strabismus Mydriasis Absence of light reflex

What are the subthalamic nucelus?

Diencephalic grey matter part of the BG The only part of the ganglia that produces excitatory neurotransmitter (glutamate)

How does stress affect the memory?

Disruption of consolidation. Adrenoreceptors play a role in processing information acquired during learning. If blocked -> no transfer into the long-term memory PTSD - loss of hippocampus volume in veterans due to high CRH ?, dissociation of emotinal memory from episodic memory with damage to amygdala

What happens if you remove the hippocampus?

Disruption of declarative memory -> global anterograde amnesia (new events, names, facts) Procedural learning is preserved

What is Dissociative sensation? what is dissociated sensory loss?

Disruption or breakdowns of memory,awareness, identity and perception; can be seen in despersonalization disorder, dissociateive amnésia, dissociative fugue, dissociative identity disorder Dissociated sensory loss is a pattern of neurological damage caused by a lesion to a single tract in the spinal cord which involves selective loss of fine touch and proprioception without loss of pain and temperature, or vice versa.

What is Antabus?

Disulfiram - an inhibitor of aldehyde dehydrogenase Used to treat alcoholism, you get unwell after drinking alcohol when taking this drug.

When should you NOT give Diazepam?

Do NOT give if: - Cirrhosis or Alcoholic hepatitis. - Hepatic encephalopathy. - FLUMAZENIL if diazepam is controlling seizures. IT may lead to intractable seizures.

Name some Inhibitors of acetylcholine esterase ?

Donepezil Rivastigmine Galantamine

Which neurotransmitters are responsible for alcohol dependency?

Dopamin Glutamate GABA Opioid system

Bottom-up attention stream

Dorsal frontopariteal network - control eye movements, representation of the stimuli salience FEF - frontal eye field IPs - intraparietal sulcus

What will lesion of Gracilis and cuneate tract look like?

Dorsal tracts - loss of proprioception, vibration and fine touch below the lesion spinal ataxia *Cuneate* - T6 and above *Gracilis* - T7 and below

Theories of motivation

Drive "biological" theory - internal states of disequilibrium, internal tension, to achieve homeostasis/stability (desire to eat, drink, reproduce) Incentive theory - external factor that trigger and regular motivation, emphasises external stumili and reward (drive for achievement, status)

What are red neurons?

Dying neurons due to hypoxia

What does both bulbar and pseudobulbar palsy lead to?

Dysarthria and dysphagia (also weak facial muscles) *Bulbar palsy - LMN* - V, VII, X, XI, XII is affected - Affection from nuclei and distally Gag reflex - absent Tongue - wasted, fasciculations "wasted, wrinkled, thrown into folds and increasingly motionless". Palatal movement - absent. Jaw jerk - absent or normal Speech - nasal "indistinct (flaccid dysarthria), lacks modulation and has a nasal twang" Emotions - normal Other - signs of the underlying cause, e.g. limb fasciculations. *Pseudobulbar palsy - UMN* - Affection of corticobulbar tract innervating V, VII, XI, X, XII - problems with masticatory muscles and facial expression Gag reflex - increased or normal Tongue - spastic "it cannot be protruded, lies on the floor of the mouth and is small and tight". Palatal movement - absent. Jaw jerk - increased Speech - spastic: "a monotonous, slurred, high-pitched, 'Donald Duck' dysarthria" that "sounds as if the patient is trying to squeeze out words from tight lips". Emotions - labile Other - bilateral upper motor neuron (long tract) limb signs.

What is the impaired protein in Duchenne muscular dystrophy?

Dystrophin

27. Hyperthermia: A. Can lead to rhabdomyolysis B. Can cause kidney failure C. Can be fatal D. Can be induced by ecstasy E. A and D are correct F. Nothing is correct

E. A and D are correct May also be fatal?! What about kidney failure? Yes, it can be fatal! Wiki: Extreme temperature elevation can become a medical emergency requiring immediate treatment to prevent disability or *death*. If heat stroke, the decrease in blood pressure can then cause blood vessels to contract reflexly, resulting in a pale or bluish skin color in advanced cases. Young children, in particular, may have seizures. Eventually, *organ failure*, unconsciousness and death will result. Cocaine, crack, metamphetamine, MDMA (ectasy) RR: A,B,C,D should be correct

14. 1st rank schizophrenia symptoms according to Schneider do not include: A. Audible thoughts B. Voices arguing or discussing or both C. Voices commenting D. Somatic passivity experiences E. Depression

E. Depression

a) perceptice aphasia is mostly caused by tumors of corpus callosum b) flashbacks arise when you have malfunction in the temporal lobe c) in traumatic deviation of optic bulb, alexia happens d) optic agnosia belongs to cognitive diseases

Elise: True B,D K: enig med elise

Amygdala lesion

Emotional memory dysfunction

Mood

Emotional state, less specific and of longer duration than emotion

Chronic forms of subdural bleeding

Encapsulation of hematoma followed by osmotic expansion - formation of a subdural hygroma

Memory processes

Encoding (acquisition) = perception of information and storing it in memory - medial temporal lobe and hippocampus Consolidation = strengthening/modulation of the memory, synaptic plasticity, dependent on gene transcription Retrieval - consolidated memory is no longer dependent on hippocampus, important are frontal lobes, include retrieval, recall and recognition Reconsolidation - recalled information already stored is again stored in long-term memory with possible changes (can lead to formation of false memories)

Frontal lobe lesion

Episodic and working memory dysfunction

How does decortication look?

Extension of LE Flexion of UP lesion is above midbrain usually it is an extensive bilateral lesion of hemispheres or diencephalon

What is the function of Huntingtin?

Exact function - unknown Important for nerve cells - signaling, transport, binding, protecting against apoptosis ?

What are exclusion symptoms for generalized anxiety disorder?

Exclusion symptoms: must not have panic disorder, OCD, hypochondriac disorder, phobias

What is ibogain been used for?

Experimentally in treating addiction of alcohol, heroin and nicotine - eliminates withdrawal for a certain period (?)

A patient non-responding to pain (no eye opening, speech, nor a motor reaction) has the GCS score 0 points

False

Where are the proteins deposited?

Extracellular - amyloid, PrP Intracellular - Tau, synuclein, TDP-43 --> intranuclear or cytoplasmic, neuronal or glial

Neglect syndrome

Failure to respond or to focus attention on stimuli presented on the opposite side of the brain lesion. Often connected with non-dominant hemisphere lesion.

"Tinnitus maskers" are instruments covering the external auditory canal

False

- Metabolic alkalosis decreases brain activity

False

- Patients with chronic mm pain have increased blood pressure and pulse rate

False

- Polyneuropathy is a paraneoplastic syndrome associated with end stage cancer

False

- Stupor puts the patent at the risk of hyperventilation

False

A chronic subdural hygroma occurs as a result of chronic epidural haematoma resorption in about 1-3 weeks after the fomration of haematoma

False

A compression of the optic chiasm from below (e.g. by a pituitary adenoma) causes binasal hemianopia

False

A damage to an optic nerve right out of the eye bulb (anterior to the optic chiasm) manifests as a homonymous ipsilateral hemianopia

False

A damage to the dorsal roots of the spinal cord is accompanied by a spastic paralysis

False

A decerebrate rigidity is characterized by the flexion of upper limbs and the extension of lower limbs

False

A decreased sodium level is a typical finding in the cerebrospinal fluid of patients with a basilar meningitis

False

A delusion of self-reproach is the first rank symptom of schizophrenia

False

A lack of emotional reaction and increased sleepiness are not "somatic" symptoms of depression

False

A lesion to thalamus causes homolateral hemialgia

False

A loss of tactile sensitivity in the radicular area L5 testifies for the compression of the L5 anterior root

False

A nuclear positivity for oestrogen receptor is typically found in meningeoma

False

A pain of cervical spine irradiating to an arm results from a C3 root irritation

False

Typical changes in the sleep architecture during depression include both an increased activity in stages 3 and 4 of NREM sleep and a REM sleep reduction

False

Typical symptoms of a somatophorm disorder are the fear of a serious disease, an interpretation of somatic sensations as signs of such an illness, and a delusional character of such beliefs

False

Unwanted intrusive thoughts (obsessions) are typical of agoraphobia

False

Visual hallucinations are the most frequent perception disorders in schizophrenia

False

- Obsessions and phobias are formal thought disorders

False Content of thought

- PML demyelinisation is due to JCD virus in the immunocompromised people

False JC virus

Serotonergic theory of pscyhosis has root in experiements with cannabis

False LSD!

Amyotrophic lateral sclerosis is a demyelinating disease

False MA: From PP - It is a *DENERVATION ATROPHY*. Loss of *motor neurons* in the brain and spinal cord, specifically the *anterior horn cells of the spinal cord*, the *motor nuclei of the brainstem* (especially the hypoglossal nuclei), and the *upper motor neurons of the cerebral cortex*. The muscles innervated by injured spinal areas become atrophic

Acute suppurative meningoencephalitis of newborns is caused by gram- diplococci

False MA: I think after 1 month of age this would be true. Neisseria meningitides (G-) are more frequent then

Oxycodon and naloxol will cancel out each others function, so to use them together is pointless ?

False Oxycodon + naloxone - used for treatment of pain, ok to combine. - Does not produce constipation - Oxycodon has low first pass effect, naloxone has high first pass effect - Oxycodon goes out into circulation, and naloxone is almost compleltely removed - Good for pain relief without causing constipation

illusions are thought disturbances?

False They are perceptive disturbances

- Confabulation fills in memories in mania

False Confabulation - making up stories, very characteristic in Korsakoff psychosis) (i guess it's not in mania because our notes say; Confabulation false memory traces, fills out the individual memory defects, often in amnesia after CNS damage - filling in "gaps" in memory (witnesses of crimes) -

The Pick´s disease predominantly affects the subcortical white matter

False Pick's disease aka. FTD.

- Streptokinase preferred over alteplase for ischaemic stroke treatment

False RW Streptokinase: acts on both circulating plasminogen and fibrin-bound plasminogen. Therapeutic uses: AMI and stroke, acute pulmonary embolism, dvt, and reperfusion of occluded peripheral arteries. (not targeted specifically to fibrin clots) Alteplase: Activity localized to fibrin clots

- Mesocortical dopaminergic sensitization is believed to underlie pathophysiology of paranoid delusions

False Should be mesolimbic! Mesocortical is possibly linked to negative symptoms of psychosis.

The intracranial Schwannoma most often affects the optic nerve

False Vestibular division of the Vestibulocochlear nerve? -Radiopedia on Intracranial Schwannoma

The astrocytic plaque of alpha-synuclein is characteristic for the corticobasal degeneration

False RW: alpha synuclein parkinsonian disease / tau protein is for cb degeneration

During the "finger-to-nose" test, the patient with cerebellar syndrome does not carry out the whole movement but stops early in front of the target

False (RW - cerebellum intention tremor / parkinsons resting tremor - this one overshoots the target)

Dihydroergotoxin has only peripheral effects, it does not pass through the blood-brain barrier

False (RW - does not cross the BBB but is not just peripheral)

A nuclear positivity for eostrogen receptor is typically found in meningeoma

False (RW - it's progesterone, not estrogen)

A gross injury to the lumbar enlargement causes a central paraparesis or paraplegia

False (RW GRR - maybe because it's UML and LMN? i think it was supposed to have 'only' you can have peripheral at the level)

Benzydamine is one of the CCAIDs (common cytokine anti-inflammatory drugs)

False (rw: It selectively binds to inflamed tissues (Prostaglandin synthetase inhibitor) and is normally free of adverse systemic effects. Unlike other NSAIDs, it does not inhibit cyclooxygenase or lipooxygenase, and is not ulcerogenic)

Lamotrigine blocks Ca2+ channels with a subsequent stabilization of neuromuscular plate

False (rw: anti anxiety/epilepsy, gaba/na channels)

The so called sham rage is a behaviour observed in animal models and it is caused by the bilateral damage to amygdala

False (rw: cerebral cortex)

Furosemide can be used in the treatment of glaucoma

False (rw: fureosimde treats fluid buildup)

The origin of Parkinson´s disease lies in the dominance of dopaminergic component over the cholinergic one

False (should be cholinergic over dop rw)

A delusion is a perception disorder

False (thought content rw)

Spinal cord automatisms may occur in the cauda equina syndrome

False RW :SCA only in complete transversion

Clopidogrel is a thrombolytic drug

False RW: Clopidogrel acts by inhibiting the ADP receptor on platelet cell membranes -->Anti-platelet drug

Vigilance is the ability to switch attention and concentrate on several tasks at the same time over a longer period of time

False, it is to maintain concentrated attention on the same task over a prolonged period of time

Negri bodies are characteristic glial viral inclusions found in the rabies

False. Found in neurons, not glial cells

Affect

Feeling or emotion Psychiatry: expressed or observed emotion (e.g. restricted, flat)

Homeostatic regulation

Feelings, emotions, drives and motivation, pain and pleasure behaviors, immune responses, basic reflexes, metabolic regulation

Familial CJD

First degree relative with CJD *Neuropsychiatric illness* + specific mutation of PRNP

What should you do before you give a patient Alteplase?

First do a CT scan to exclude the presence of a hemorrhagic stroke!! (Also, there is a risk of hemorrhagic stroke from alteplase treating MI).

What is Hexylresorcinolum ?

Fixed combination of alcoholic & phenolic antiseptic drug Local anesthetic, antiseptic and amtihelmintic. Topical use and mouth wash.

What are Lisch nodules, when and where can you find them?

Found on iris in neurofibromatosis type 1. A Lisch nodule is a pigmented hamartomatous nodular aggregate of dendritic melanocytes

Metastatic tumors

From carcinomas, often from lung, breast, melanoma, kidney, GIT Well-circumscribed mass, often with surrounding edema Meningeal carcinosis - lung and breast

Stress impacts different brain structures in different stages of development - sensitive periods

Frontal cortex - 14-16 years Corpus callosum - 9-10 years Hippocampus - 3-5 years Same trauma/stress can have different consequences later in dependent on timing of events (e.g. depression (earlier) vs PTSD (later))

Focal atrophy in FTD

Frontotemporal atrophy predominate Rostral insula and gyrus cinguli Asymmetry can cause different symptoms (left = progressive aphasia, right = apathy, perservations)

Dementia before age 60

Frontotemporal dementia develops in younger population, around and earlier than age 60. Young onset of Alzheimers disease also affects people under the age of 65

What are the Adverse effects of Benzatropin?

Full stop of all flow (salvia, stool and urine) and cognitive changes and psychosis

What is the function of Clopidogrel?

Function: Blocks ADP receptor and phospholipase C.

32. Anti-craving therapy: A. Is not used B. Bupropione is used in the treatment of nicotine addiction C. Caffeine is used in the treatment of amphetamine addiction D. Ibogain is experimentally used in the treatment of opioid addiction E. Is casual F. B and C are correct

G. "G" is said to be correct - but there is no answer?! *Ibogain* - Used experimentally in the treatment of addiction on alcohol, heroin and nicotine "*Bupropion* is a medication primarily used as an antidepressant and smoking cessation aid." So, the answer should be B and D.

What are the neurotransmitters of anxiety?

GABA-A NA - Locus coeruleus Alpha 2 antagonist - induction of anxiety (Yomibin) 5HT agonist - Acute anxiogenic effect - Chronic Anxiolytic effect (SSRI)

ADHD differential diagnoses

GENERALIZED ATTENTION DISRORDER LEARNING DISABILITY OPOSITIONAL BEHAVIOR CONDUCT DISORDER EMOTIONAL AND BEHAVIORAL PROBLEMS

How can you administer Alteplase?

Given by: - Intra-venous route and within 3-4,5 hours if we have a patient with carotid ictus. - Intra-arterial route and within 6 hours if basilar a. or middle cerebral a. involved in the stroke. Summary, important: To be effective, it must be given within about 3 h of the thrombotic episode.

Differences in grey and white matter development

Grey (U-shape) - increase, max about 12 y.o., then declines gradually, variable volume changes in different regions White (linear) - increase, equal growth in all brain areas

Neurosyphilis

Gumma Atrophy of rubra corticis -> progressive paralysis Demyelination of spinal posterior fasicles (tabes dorsalis)

When can triptans not be used?

HTN May cause recurrent migraine Pregnancy

Basic emotions

Happiness, sadness, anger, disgust, fear, surprise

Viral meningitis

Has increased mononuclear cells (While acute purulent has increased polymorphonuclear and proteins?) Yes-RW

Lesions in cerebellum, what happens when the patient bends backwards?

He does not flex is knees, so he falls backward

Typical meningeal symptoms are?

Headache Vomiting Nausea Photophobia Neck stiffness? --> this is a sign, not a symptom

Sensory stimulation - first to amygdala for an emotional vignette, then to thalamus

Hearing and seeing goes to thalamus first, then to amygdala. Olfaction and tactile sense bypass thalamus and go directly to amygdala (Then it should be true for tactile and olfactory input? - Klara)

Mycobacteria infection

Hematogenous spread from lung TBC Forms: - Granulomatous meningitis (basilary in children) - Focal well circimscribed lesions (tuberculoma)

10. In alternating hemiplegia (unilateral brainstem lesion) : (look up answers) a) hemiparesis or hemiplegia occurs ipsilaterally b) cranial nerve palsy is contralateral to brainstem lesion c) cranial nerve palsy is peripheral (lower motor neuron lesion) d) cranial nerve palsy is central (upper motor neuron lesion)

Hemiparesis - contralat CN palsy - ipsilat If the CN lesion is peripheral or not depends if the lesion is above or below the nuclei of the CN.. So I'm not sure how to answer this. Most commonly lesions of brainstem causes *peripheral* CN lesions! Klara: c) must be correct! if it would be central, it would be contralateral and also wouldn't be alternating hemiplegia I think! (think brainstem lesions is almost always peripheral CN lesion with ipsilateral manifestation)

What is Weber syndrome?

Hemiplegia superior alternans Midbrain infarction 3rd nerve palsy (ipsilat) with contralateral hemiparesis

Prenatal stress

Higher HPA axis activity Lower birth weight --> ADHD, antisocial behavior, mood disorders

What memory is affected first in the aging process?

Higher cognitive functions - declarative memory

Declarative memory - important sturctures

Hippocampal formation Limbic system - Papez circuit Neocortical areas

lesion of occipital cortex leads to what type of visual loss?

Homogenous hemianopsia, contralateral to lesion RW: according to our book: homonymous hemianopsia- a lesion of optic tract, lateral geniculate body, optic radiation or visual cortex produces crossed hmonymous hemianopsia. for example a lesion of the left optic tract or following structures affects the fibers from the left half of each retina, hence the visual ield defect includes the right half of each visual feild. if only a partial / part of fibers: can give homonymous quadrantanopsia or homonymous scotoma

Non-declarative (implicit)

How? - without awareness, procedural memory (skills, habits)

What has psilocin been used as (experimentally)

Human and animal models of psychosis Therapy of PTSD Cluster headache

What is the most common cause of intracranial hemorrhage?

Hypertensive aneurysms and AV malformations On the other test it says: hypertension, aneurysm and AV malformations.

Constistent findings

Hypoactivity in networks linked with emotional regulation, including PFC Amygdala hyperactivity triggered by fearful and angy stimuli

Quantitiative memory disorders

Hypomnesia Amnesia Hypermnesia

Brain structures that mediate emotion?

Hypothalamus Limbic System Limbic cortex Amygdala Brainstem

Central conus leads to mydriasis on the opposite side?

Hæ ? No

Is perimyelo CT used for diagnosis of tearing of spine root?

Hæ? :P I don't think, that would be MRI --> De spør om det brukes for å diagnostisere ruptur av spinal roots, og det kan det i følge radiopedia =)

TBC spondylitis are cut of MRI in coronal and sagittal planes?

Høæ? Probably...never seen i MRI which would take scans only in one plane --> sagittal I think? Agreed, CT will often be in coronal. RW: In ppt for 'spine inflammation: * MRI is the most sensitive imaging modality; both planes presumably

Possible ADR of tryptans is recurrence of migraine, coronary spasms, serotonin syndrome ??

I think all could be correct.. ?

- Cerebellum connected to temporal lobe is a part of Papez circle

I think not, according to powerpoint.. The classical papez circuit does not include cerebellum, but new studies show that cerebellum may be the starting point of Papez

How to modify progress of MS ?

IFN beta

Herniation of lumbal disc leads to paraparesis

If lesion is in upper Lumbal region L1-L2 --> yes Under L1-L2 --> not full paraparesis. Seeing how spinal cord ends at L1-L2 then it would only affect the roots which would be asymmetrical (what is ment by this? --> I guess "not symmetrical"; not two legs?) Ida: Disc herniation is often laterally displacement, so in most cases only one extremity will be affected RW--> so hemiparesis only? of Lower Limb? --> NO

Background of ADHD

Impaired function of adrenergic and dopaminergic system (ARAS)

Spatial memory - navigation

Implicit = egocentric - following a known route, procedural learning (cerebellum, BG cautate ncl) Declarative = allocentric - finding the way in a new environment, memory for orientation cues and their relationships, a cognittive map (hippocampal formation) Test: hidden goal task

Fungi - examples

In IS patients and deep tissue mycosis, spreading to CNS as a late complication --> granulomatous meningitis, encephalitis (BG cavities) or vasculitis Candida albicans Aspergillus fumigatus Cryptococcus neoformans Mucor

Ecstasy-opioids: MDMA/ecstasy, today, in ecstasy located mainly opioids?

In ectasy we should find MDMA, but on todays market ectasy is filled with other types of drugs in (80%) of cases. (such as amphetamine, ketamine, PCP and others)

How does amantadine work?

Increased dopamine release Direct effect on dopamin receptors Block dopamine reuptake MAO-A inhibitor NMDA antagonist Anticholinergic effects

What is neurasthenic syndrome?

Increased fatigue +somatic symptoms after a relatively small mental effort. Rest and relaxation does not help

How do we get addicted to nictoine?

Increased number of Ach receptors in the brain

What is function of MDMA?

Increased release of serotonin, less NA and DA

Does THC increase or decreased heart rate?

Increases HR for around 3h BP is higher while sitting, lower when standing

Sensitization

Increasing behavioral response to stimuli

What are the indications for use of Atropine?

Indications: - Decrease the secretions perioperatively. - Increase the HR. - Treat poisoning, e.g. by organophospate insecticides or by „nerve gas agent" both of which are irreversible AchE inhibitors.

What is indications for use of Amantadine?

Indikasjoner: levodopainduserte dyskinesier. - It is both ANTIVIRAL and ANTIPARKINSONIAN DRUG - Limited role in treatment and prophylaxis of influenza (Which was its initial indication!!) - Fatigue in MS (moderate effect) - Organic compound

Causes of secondary dementia

Infections Metabolic and paraneoplastic Post-traumatic Toxic Intracranial expansions

WHat is the mechanism of action of cocaine?

Inhibit reuptake of DA, NA and serotonin

Properties of attention

Intensity (focused and selective) - level of concentration Vigilance (sustained attention) Capacity/range - nr of stimuli captured at one time Alternation - shifting focus Divided attention - multitasking

What is most common of intracranial and intraspinal tumors?

Intracranial

Where are the Lewy bodies in parkinsons disease?

Intracytoplasmic

Signs of lesion in medulla?

Ipsilatearl hypoglossal palsy and contralateral hemiparesis (*Jackson's sy* according to school, *Dejerine* according to the internet).

Signs of lesion in pons?

Ipsilatearl peripheral facial palsy Contralateral hemiparesis (Millard-Gubler synd.)

What is Benserazide?

Is a PERIPHERALLY-ACTING inhibitor of DOPA decarboxylase. It can not cross the blood brain barrier

Lambert-Eaton is a paraneoplastic syndrome; small lung cell carcinoma

Is a rare autoimmune disorder that is characterized by muscle weakness of the limbs. It is the result of an autoimmune reaction in which antibodies are formed against *presynaptic* voltage-gated calcium channels in the NM junction. Around 60% of those with LEMS have an underlying malignancy, most commonly small cell lung cancer; it is therefore regarded as a *paraneoplastic syndrome*.

Non-bizarre delusions

Is when a girls believes that secret police is after you and want to kill you "it is something ocurring in a person's life which is not completely impossible" For example, the person may believe their significant other is cheating on them, that someone close to them is about to die, a friend is really a government agent, etc. All of these situations could be true or possible, but the person suffering from this disorder knows them not to be

When does post-traumatic stress disorder develop?

It can have a latency period from weeks to 6 months. (probably even longer..)

How does Benzalconium work?

It denatures the cell membranes leading to the release of their intracellular components.

How does memantine work, and what is it used for?

It is an NMDA receptor antagonist It is used for moderate to severe alzheimers disease

What is Risperidone?

It is an antipsychotic drug mainly used to treat schizophrenia (including adolescent schizophrenia), schizoaffective disorder, the mixed and manic states of bipolar disorder, and irritability in people with autism. Risperidone is a second-generation atypical antipsychotic. It is a dopamine antagonist possessing anti-serotonergic, anti-adrenergic and anti-histaminergic properties.

Primidone is converted to phenobarbital and has identical effect?

It is converted, and they are structural analogues according to wiki.. So it is probably correct. Used as anti-epileptic drugs

What is penumbra stroke?

It is the area just around the ischemia Still viable tissue around the ischemic zone - tissue with critical metabolic reservce

When is Benserazide used?

It is used in the mangement of Parkinson´s disease in combination with L-DOPA as CO-BENELDOPA

How is Dentrolene used therapeutically?

It is used therapeutically: - To relieve muscle spasm in the rare condition of malignant hyperthermia!!!! - For MS and cerebral palsy, so for chronic muscle spasms as well. - Neuroleptic malignant syndrome. - It is a challenge in long-term use to maintain a balance between muscle spasm relief and muscle weakness and hypotonia. NOT for spasm after acute injury. R. Baclofen and clonazepam.

How does Benzydamini hydrochloridum work?

It selectively binds to inflamed tissues (*prostaglandin synthetase inhibitor*) and is normally free of adverse systemic effects. Unlike other NSAIDs, *it does not* inhibit cyclooxygenase or lipooxygenase, and is not ulcerogenic.

Progressive multifocal leukoencephalopathy (PML) - etiology

JC virus affect the white matter primarily, it infects oligodendrocytes and astrocytes. Oligodendrocytes die, while astrocytes show extreme pleomorphism (bizarre, may mimic neoplasia)

Dementia praecox is the same as schizophrenia according to Kraeplin or Bleuler?

Kreaplin - Dementia Praecox Bleuler - Schizophrenia

Main candidates for the neural substrates of learning and memory

LTP = long-term potentiation LTD = long-term depression

Spinal muscluar atrophy (SMA)

Large number of atrophic fibers, often involving an entire fascicle = panfascicular atrophy Scattered abnormally large fibers (compensation?)

Name a precursors of acetylcholine?

Lecithin

Which brain structures are involved in anxiety?

Limbic system Amygdala (ablation of amygdala --> Anxiety amelioration) Locus Coeruleus (NA --> Autonomic reaction) Frontal cortex Hyperperfusion of parahippocampal gyrus

In which lobe is amygdala?

Located deep within temporal lobe of the brain

What happens to your brain when you drink alcohol?

Loss of dendritic arborization + neuronal shrinkage Neuronal death and Wallerian degeneration of myelinated axons Reduced gray and white matter Increased cisterne volume Atrophy of mammillary bodies Decreased volume of corpus callosum Neuronal loss in - Dorsolateral prefrontal cortex - Hypothalamus - Cerebellum Reduced glial mass in hippocampus

Infantile amnesia (0-2 - 5 years)

Loss of memory for the first 4 years of life due to immature hippocampus

Risk factors for PTSD

Low IQ Smaller hippocampus level (increased vulnerability, war experience does not cause hippocampus atrophy) Genetic factors

Nucleus accumbens is responsible for cravings?

Low dopamine in VTA and NAc --> Cravings OBS! According to pp: prefrontal cortex and amygdala are responsible for craving. Addiction is VTA and ncl. accumbens What to answer? RW: NO? Notes say :* Rewarding effects of most drugs depends on increased DA in NcAcc • * For drug seeking and craving is imprtant DA in prefrontal cortex and amygdala

Verbal abuse in children

Lower FA in arcuate fasiculus, cingulus, and fornix Correlations: FA and VIQ, inversed FA with depression, somatization and anxiety

What can be the consequence of repeated sexual abuse in early childhood/adolescence?

Lower hippocampus/PFC volume

What are symptoms of lumbar enlargement?

Lumbar enlargement is not a pathology, it is the widened area of the spinal cord that gives attachment to the nerves which supply the lower limbs. The questions should be what are symptoms of lumbar enlargement lesion? Mono- or paraparesis of the legs. Peripheral paresis at the level of segmental cord damage (and upper motor neuron lesion below the lesion, detecting level of cord damage). Divided into an upper part (L1-L4) and an lower part (L5-S2), and they have different symptoms.

Viral infections forms

Lymphocytary meningitis or encephalitis with viral inclusions and lymphocytic perivascular infiltrates

Arthropod borne viral encephalitis

Lymphocytary meningitis or meningoencephalitis Mosquito and tick vectors

What are entactogens?

MDMA - ecstasy and many others (MDEA, MDA, MBDB)

What will a transversee lesion of spinal cord cause?

MOTOR 1st phase - spinal shock --> flaccid paresis 2nd phase - spastic UMN below level of lesion SENSORY Hypersthesia or hyperalgesia at level of lesion Impairment of all modalities below lesion BLADDER First urinary retention Then autonomic neurogenic bladder

Name some drugs with "resin" in them?

Marihuana Hashish

Diabetic neuropathy

Marked loss of myelinated fibers Thickening of endoneurial vessel wall - ischemic neuropathy

what is MUS?

Medically unexplained symptoms

Rhombencephalon evolve into what?

Medulla Pons Cerebellum

Poorly differentiated tumors

Medulloblastoma - grade IV - children, cerebellum - desmoplastic, neuroblastic or large cell Atypical teratoid/rhabdoid tumor - grade IV - children

Name an inhibitor of NMDA glutamate receptor?

Memantine

Memory vs learning

Memory is the ability of organisms to save (encoding), organize, store (storage) and recall information Learning is the process of storing information (learning, encoding, acquisition) into memory.

Feeling

Mental representation of physiological changes; private, not visible to others

CADASIL clinical manifestation

Migraines with aura Repeated strokes Subcortical predominant form of dementia

What is MCI?

Mild cognitive impairment - presymptomatic state of dementia

How does cannabis work in the brain?

Modulates brain dopamine levels, can induce psychosis

What are mood congruent delusions?

Mood-congruent delusion: Any delusion with content consistent with either a depressive or manic state, e.g., a depressed person believes that news anchors on television highly disapprove of him, or a person in a manic state might believe she is a powerful deity.

What are mood-incongruent delusions?

Mood-incongruent psychotic features are delusions or hallucinations that either contradict or are inconsistent with the prevailing emotions, such as delusions of persecution or of thought insertion in either a manic or a depressive episode. An example is a depressed person who believes that thoughts are being inserted into his or her mind from some outside force, person, or group of people, and these thoughts are not recognized as the person's own thoughts (called "thought insertion").

what is a mood neutral delusion?

Mood-neutral delusion: A delusion that does not relate to the sufferer's emotional state; for example, a belief that an extra limb is growing out of the back of one's head is neutral to either depression or mania

Contusion

More severe trauma with morphological changes of brain Unconsciousness *Edema* *Always accompanied with subarachnoid and parenchymatous bleeding* Coup with or without contracoup LIFE THREATENING -> ICU, consider surgery

Cerebellar lesion

Motor learning dysfunction

What is the mechanism of Benzatropin?

Muscarinic Ach receptor antagonist.

How can we diagnose neuromuscular disorders?

Muscle and nerve biopsy

What happens in cases in which reinnervation of a skeletal muscle does not occur?

Muscle fibers progressively become atrophic and are ultimately replaced by fibroadipose tissue resulting in muscle weakness

On which receptor does Ketamine and PCP work?

NMDA antagonists

Echolalia is a typical sign of cataplexy? What is Echolalia and Cataplexy ?

NO! OBSOBS! Echolalia is typical of catatonic schizophrenia! Echolalia: meaningless repetition of another person's spoken words as a symptom of psychiatric disorder. Cataplexy: temporary loss of muscle tone and weakness precipitated by emotional state. A strong emotion or laughter causes a person to suffer *sudden* physical collapse though remaining conscious.

Hypoprosexia is lack of attraction?

NO, it is loss of attention! Hypoprosexia is the reduced ability to focus attention and to concentrate on an object or task.

Bipolar disease: The depressive episodes are less frequent and last shorter than the manic episodes?

No

NOTE ON ALL ANTIPSYCHOTICS:

NOTE ON ALL ANTIPSYCHOTICS: i. When given by intramuscular injection they require lower dose. j. ATYPICAL antipsychotics are better than conventional antipsychotics for the NEGATIVE symptoms of schizophrenia. So: i. DIFFERENT THERAPEUTIC PROFILE of Atypical vs. Typical antipsychotics. (in test) k. Conventional antipsychotics are completely contraindicated in PD. Only Atypical antipsychotics can be used!!!! l. Antipsychotics work as Dopamine antagonists since Increase dopamine in the Mesolimbic pathway leads to positive symptoms!!!!!! R. Glutamatergic theory better..both positive and negative symptoms battled.

Benzydamini hydrochloride: 1. Spermicidic effect 2. bactericide + fungicide + viricide eff. 3. NSAID

NSAID Anesthetic, analgesic, anti-inflammatory for mouth and throat Has antibacterial effect

The drug most commonly used for migraine is?

NSAIDs OBS! Triptans are effective in 80 % of patients and are "first line drug" BUT only cca 20 % of patients are treated by triptans.. Teit!

Is the therapeutic window for lithium narrow or wide?

Narrow

When MS is acutely worse, which drug to use?

Natalizumab (also known as Tysabri)

Can Benzodiazepines induce psychosis?

No

Markers of AD

Neuritic plaques (amyloid) - extracellular Neurofibrillar tangles (tau protein) - intracellular inclusions caused by hyperphosphorylation of microfibrils

2 causes of primary dementia

Neurodegeneration Ischemic-vascular

What medication is used for Huntingtons disease ?

Neuroleptics (1st generation antipsych Haloperiodol) Benzodiazapines (Diazepam) Tetrabenezines Pharma pp notes 1. Reserpine, Meserpine --> depletion of dopamine 2. Tetrabenzine -> Depletion of dopamine 3. Baclofen --> Central muscle relaxants 4. Typical antipsychotics 5. Drugs decreasing excitotoxicity

What is Korsakoff psychosis?

Neurological disorder caused by a lack of thiamine (vitamin B1) in the brain. Its onset is linked to chronic alcohol abuse and/or severe malnutrition.

Subarachnoideal (periencephalic) bleeding is characterized by blood in the ventricles?

No (Not characterized, but blood can be present in ventricles after both subarachnoid bleeding and intracerebral bleeding!) (I think it is "often" seen in intracerebral btw)

In intracerebral bleeding is often connected with diastasis of sutures and damage to meningeal arteries?

No (This should be epidural bleeding)

Tremor is always a manifestation of parkinson's disease?

No (this question has a double meaning, but both are wrong)

Traumatic deviation of eye ball can lead to alexia?

No Alexia is due to a lesion in cerebrum, nothing to do with the periphery!

Anosognosia is typical just for patients with schizophrenia?

No Also for those with bipolar disorder Anosognosia is *a deficit of self-awareness*, a condition in which a person who suffers some disability seems unaware of the existence of his or her disability.

Frontal dementia leads to anterograde amnesia

No Anterograde amnesia is typical for lesions in hippocampus and subcortical regions.. I don't think the FTdementia typically present with anteograde amensia, but I read some research saying it can.. But I don't know. I frontotemporal dementia pp fra skolen: no problems with memory test? from wiki: However, the following abilities in the person with FTD are preserved:[3] Perception Spatial Skills Memory --> preserved memory! Praxis MA: (from PP) medial temporal lobe and hippocampus (anterogradeamnesia)

Bipolar disease has the same prevalence as depression?

No BD: 1-2% Depression: 20%

Can 6th cranial enrve palsy lead to divergent strabismus?

No Cause convergent strabismus due to failure to abduct horizontally to affected side

Confabulations are perception disorders?

No Confabulations are *memory disturbance*, defined as the production of fabricated, distorted or misinterpreted memories about oneself or the world, without the conscious intention to deceive usually seen in *alzheimers, dementia, Wernicke Korsakoff syndrome*. Confabulation is distinguished from lying as there is no intent to deceive and the person is unaware the information is false.

Metastasis to vertebrae leads to syringomyelic dissociation?

No Definition: loss of pain and temperature sensation with relative retention of tactile sensation, related to a cavity in the central portion of the cord interrupting the decussation of nerve fibers.

In depression, is stage 3 and 4 of non-REM sleep increased, and REM sleep decreased?

No Depressed people show abnormal sleep EEG 1. Reduced REM latency (lasts shorter) 2. reduction of 3rd and 4th stage of NREM 3. prolongation of REM in the 1st half of the night 4. more frequent alteration of NREM phases

The risk of suffering from bipolar disease is the same as with depression?

No Depression 20% risk BPD 1% (but there his a higher suicide risk in those with BPD)

Incidence of BAD and depression is the same?

No Depression: 20% BAD (bipolar affective disorder): 1-2%

Lesions in wernickes area is called expressive aphasia?

No Expressive aphasia is the same as Broca's aphasia Receptive aphasia is the same as Wernickes aphasia

Variant CJD incidence

No Gives a statement about occurrence in 1-2/million people or smth like that and then says ''that so naturally we have 5-8 incidents per year in CR''. This is wrong since the epidemiology is described in the ppt for cows and not for people. a. Hmm, in my documents (from the pp) it says occurrence of *sporadic* CJD is 1-2/1million.. According pp we have around 220 cases of vCJD.. So it is not 1-2 per million people at least..

During hemiplegia alternans inferior we have circular rotatory nystagmus?

No Hemiplegia alternans *inferior*= occurs with hypoglossal palsy - lesion of medulla oblonganta. Nystagmus occurs with vestibulocerebellar lesions, which is in lesions of pons = Hemiplegia alternans *media*

- Concomitant use of naloxone with oxycodon removes the analgesic effect

No If naloxone is given i.v --> YES If naloxone is given as tablets --> NO (high 1st pass effect)

Is Risperidon used for cognitive enhancement? What kind of drug is it?

No It is an antipsychotic (atypical) for schizophrenia, bipolar, mixed and maniac and autism

Olfactory and tactile sensory signals go to thalamus, and then to the amygdala ?

No Olfactory sense and tactile is connected *directly* to amygdala, and bypasses the thalamus. --> Smells and pain, therefore, often evoke stronger memories or feelings than do sights or sounds, which have to go through thalamus first!

Does a lesion of optic nerve cause homonymous hemianopia?

No Optic nerve lesion --> mono-ocular blindness Lesion posterior to optic chiasma causes homonymous hemianopsia RW: lesion of the right optic tract, lateral geniculate body, optic radiation, or visual cortex produces a left homonymous hemianop- sia, while a lesion of any of these structures on the left produces a right homonymous hemianopsia

Paracetamol and Metoclopramide is much better than triptans?

No Paracetamol 1g + Metoclopramide 10mg has *no significant difference* in pain relief compared to sumatriptan 100 mg after 2 hours. There is no difference between the two drugs in photophobia and phonophobia, but there is more adverse drug reactions after sumatriptan.

The plaque in Alzheimers disease is realted to tau proteins?

No Plaques -> Beta amyloid Tangles -> Tau protein

(Level of) Vigility does not influence memory ?

No Quantity: level of consciousness - vigility (normal) vs. somnolence, stupor, coma Assess the patients' ability to interact with his environment

The difference between a recurrent depressive episode and depressive episodes in BAD is very easy?

No Recurrent depressive episode: Is multiple depressive episodes, without any episodes of mania or hypomania BAD: At least one depressive episode, with atleast one manic or hypomanic episode Often BAD is diagnosed as depression first.

Lumbar disc herniation leading to cauda equina syndrome can be operated within a week?

No The patient has to be acutely operated, disc prolapse must be removed within 24-48 hours

For surgical therapy is used ventral stereotaktical operation of radix anterior?

No Therapy of what? :P https://en.wikipedia.org/wiki/Stereotactic_surgery Test from this years winter semester: Stereotactic surgery of thalamus is the best method for the treatment of neuropathic pain - TRUE

Triptans are contraindicated in hypotension?

No They are contraindicated in untreated arterial hypertension

Lesion in superior medulla oblongata will cause contralateral hemiparesis, ipsilateral lesion to CN III ?

No This is a description of a superior *midbrain* lesion (Weber syndrome) In medulla we find the lower cranial nerves (IX-XII)

Effect of triptans is severely stronger when combined with METO (etc) ... (???)

No Triptans should no be combined with MAOinhibitors!

Preparations containing Ginkgo can be used for migraine patients?

No (Alzheimers) I think it can.. atleast the internet thinks so! https://migraine.com/migraine-treatment/natural-remedies/gingko-biloba/ Klara: Agree that it can be used for prophylaxis :)

In dermatomyositis the infiltrate is mostly CD8+ ?

No, CD4+

In addiction treatment cocaine is connected with lacrimation?

No (That is opioids)

Are there imputation of ideas in mania?

No (Thought insertion) (RW: thought insertion doesnt equal imputation of ideas?)

Compulsions is a perception disorder?

No (anxiety disorder)

TDP-43 in pathological states are found in the nucleus?

No (physiologically found in the nucleus)

The incidence of MS is 1:10000 ?

No 1:1000

Hypascusis on one side can be only from the middle ear?

No Can also be from the external ear hypascusis=partial loss of hearing

Causalgia is the same as phantomic limb pain?

No Causalgia - occurs after large mixed nerve injury with partial denervation. Involves burnin paing, prominent in the hand or foot within 24 h of injury Sensory stimulation worsens the pain it is not the same. Causalgia is pain of an limb that is actually existing, while phantom limb is pain in a limb that is not existing

Parkinson's disease is somehow related to cholinergic receptors?

No Dopaminergic receptors But OBS! You have increased acetylcholine and decreased dopamin in Parkinsons, but the cholinergic receptors are not "affected". --- so it really depends what they mean by "related" RR

In frontal type of dementia the function mostly affected is speech?

No I think behavior and personality is first.. Changes in personality Deterioration of language These two are present *before* memory disturbance.. Behavior changes are often the first noticeable symptoms in bvFTD, the most common form of FTD.

The thinnest cranial nerve is number XI ?

No IV nerve is the thinnest

Tickborn encephalitis is JUST leukoencephalitis?

No It has 3 variants (Unsure what they mean here..)) Types of tick born encephalitis viruses: 1. European or Western tick-borne encephalitis virus 2. Siberian tick-borne encephalitis virus 3. Far eastern Tick-borne encephalitis virus ( Russian Spring Summer encephalitis virus) Can affect: 1. meninges 2. brain 3. both

MS changes in white matter happens mostly in the ncl. Caudatus?

No Mostly in optic nerve chiasma and periventricular white matter Ncl. Caudatus is grey matter (A part of Basal Ganglia) RW: characteristics include depletion of myelin-producing oligodendrocytes, with multiple focal areas of demyelination (plaques) that are irregularly scattered in the *brain and spinal cord; the optic nerve, brain stem, and paraventricular areas* are favored sites; helper CD4+ and cytotoxic CD8+ T lymphocytes and macrophages infiltrate plaques; reactive gliosis occurs later.

Lesions of optic nerve before the chiasma leads to homonymous hemianopsia?

No Scotoma or monocular blindness

Patient with chronic muscle pain will have increased HR and BP?

No The chronic pain does not seem to activate the sympathetic system

The most common localization of intracerebral hemorrhage is in the mesencephalon due to the lenticulostriate vessels?

No Nikolas - ppt: usually bg and capsula interna especially avm formation - a lenticulostriate vessel (charcot's hemorragic artery) - but it aint the mesencephalon From most common to least: 1. basal ganglia haemorrhage (especially the putamen) 2. thalamic haemorrhage 3. pontine haemorrhage 4. cerebellar haemorrhage Mesencephalon is made from: tectum inferior colliculi superior colliculi cerebral peduncle midbrain tegmentum crus cerebri substantia nigra Lenticulostriate vessels come from the a. cerebri media And supply the basal ganglia and much of the internal capsule.

Is carbamazepine safe in pregnancy?

No Like other anticonvulsants, intrauterine exposure is associated with spina bifida and neurodevelopmental problems.

Osteochondrosis is lysis of vertebra?

No (?) They are characterized by interruption of the blood supply of a bone, in particular to the epiphysis, followed by localized bony necrosis, and later, regrowth of the bone. Ida: It will cause necrosis, so maybe lysis is correct? RW: I say No. From Neurobook: Osteochondrosis leads to reactive spondylotic changes and, therefore, increased stress on the intervertebral joints. This, in turn, causes faulty posture, reflex functional disturbances of the muscula- ture, and, therefore, pain.

Increased passivity in lesion of cerebellum is due to apathy?

No (?) (RW agree) Passivity (hypotonia) during passive movements: • Resistance to passive movements is diminished • The joints may be passively hyperextended • Pendulousness is increased Due to loss of the *deep nuclei in cerebellum* supplying the cerebellocortical tract.

Phantomic limb - due to increased amines (?)

No (?????) Tricyclic antidepressant are one of the most effective treatment against neuropathic pain. And these are reuptake inhibitors of serotonin and NA. (Just info depending on how the question is asked)

Can triptans be used in pregnancy?

No (Asked the teacher!) Eva: No (triptans are used as abortive medication, it is non-recommended to use them in pregnancy)

Vasodilatation decreases intracranial hypertension?

No (RW agree) I think vasodilation is the way to increased the ICP I agreeeee Nikolas

Is the prefrontal cortex responsible for creating space memory ?

No (RW) Powerpoint: Memory formation happens in hippocampus, amygdala, striatum and mammillary body Wiki article: The medial prefrontal cortex processes egocentric spatial information. It participates in the processing of short-term spatial memory used to guide planned search behavior and is believed to join spatial information with its motivational significance. So what do we answer here? - no according to school!

Increased dopamine in nucleus accumbens leads to craving?

No (increase in mesocortical) During drug abuse the amount of dopamine increases, so when you stop taking the drug, the body is not satisfied with the "normal" amount, and wants more --> Craving! -Lack of dopamine in NAc gives craving

If there is a pathology of cerebellopontine angle, is the *central* facial nerve affected?

No (not central) But the peripheral nerve can be affected! "Central facial palsy can be caused by a lacunar infarct affecting fibers in the internal capsule going to the nucleus" Central: From cortex --> Nucleus in brainstem

Niemann Pick is a neurodegenerative disease ?

No - metabolic neuropathy A group of inherited severe metabolic disorders in which sphingomyelin accumulates in lysosomes in cells.

Concussions

No detectable morphological change Unconsciousness of variable duration *Amnesia is always present* Desorientation and anxiety is common

What does the removal of both hemispheres and hypothalamus cause (speaking about rage)

No rage

Can dissociated sensory loos be caused by anterior spinal artery occlusion?

No(??) (old test answer) RW-Yes, of course it can be. Check the causes on the bottom of this wiki-article https://en.wikipedia.org/wiki/Dissociated_sensory_loss --> Wikipedia --> YES it can be a cause...! Dissociated sensory loss: a pattern of neurological damage caused by a lesion to *a single tract* in the spinal cord which involves selective loss of fine touch and proprioception without loss of pain and temperature, or vice versa RR: since it affects the spinothalamic tract --> affect pain + temperature - the other sensory modalities are preserved - also affects corticospinal tract --> motor deficit ==>>> agreee with RW

Affection of a peripheral nerve or of a spinal root manifests as tactile sensation disorder, deep and thermic sensation is normal ?

No, all modalities are impaired Skjønner ikke denne? Lesion of peripheral nerve, a plexus or a root leads to motor and sensory disturbance of all modalities, alt går jo sammen ut fra spinal cord i samme nerve - klara)

Somnolence is the highest stages in quantitative consciousness?

No, awake and aware is higher? - Awake and aware (alert) - vigility - Somnolence - Stupor - Coma

Glaucoma is caused by increased ICP (intracranial pressure) ?

No, caused by increased intraocular pressure High ICP can cause papilledema

Does oculomotor nerve lesion leads to convergent strabismus ?

No, divergent strabismus (VI takes over) (ptosis, absent accomodation, mydriasis, no light reflex)

Phenytoin is an anxiolytic drug?

No, epileptic drug

Ergotamines - a disadvantage is the accompanying hypotension

No, hypertension They have vasoconstrictor effects Treatment of migraine

Antipsychotic medication occupying dopamine receptors affect mesolimbic and nigrostriatal pathway, but not tuberoinfundibular?

No, it affects all three a. Mesolimbic and mesocortical system: antipsychotic effects b. Nigrostriatal system: extrapyramidal effects c. Tuberoinfundibular system: endocrine effects

Suicide is common with ppl suffering from tinnitus ?

No, it happens but it is not very common

Benzalconium increases the viability of sperms?

No, it is a spermicide

Uremic encephalopathy is related to liver failure

No, it is related to renal failure

Tuning fork examination are obliterated today?

No, it is still a fundamental part of examining CN VIII

Triptans will cause constriction of veins in the brain?

No, it should be arteries (???) https://en.wikipedia.org/wiki/Sumatriptan "The specific receptor subtypes it activates are present on the cranial arteries and veins."

lesion of the back portion of parietal lobe will lead to ataxia ?

No, it would be apraxia and hemispatial neglect *Apraxia* is a motor disorder caused by damage to the brain (specifically the posterior parietal cortex), in which someone has difficulty with the motor planning to perform tasks or movements when asked. A person with apraxia cannot move his or her lips or tongue to the right place to say sounds correctly because, even though the muscles are not weak, the messages from the brain to the mouth are disrupted.

A lesion to CN3 can lead to problems closing the affected eye?

No, leads to problem opening the eye. A problem with closing the eye is seen in bell's palsy (facial nerve)

Rabies get to the brain via the middle ear?

No, via peripheral nerves from muscles after animal bit

Can you reason with a patients delusions?

No, you can't change what he thinks, because he is sure that it is real.

Glove sleeve = diffuse sensory impairment?

No... ? Stocking and glove (strømpe og hanske) typical for peripheral neuropathy eg. DM. Where tingling and paresthesia starts on feets then hands. (longest nerves) Nikolas-yes definitely (Several nerves Polyneuritis or polyneuropathy - diffuse sensory impairment is predominantly on the distal parts of the limbs - „glove and stocking anesthesia")

In subarachnoidal bleeding the meningeal signs are present at once?

No? Symptoms of meningeal irritation, including nuchal rigidity and pain, back pain, and bilateral leg pain, occur in as many as 80% of patients with SAH but *may take several hours to manifest*.

What is a non-bizarre delusion?

Non-bizarre delusion: A delusion that, though false, is at least possible, e.g., the affected person mistakenly believes that he is under constant police surveillance.

a) pyritinol is a drug used for encephalopathy in children b) therapy with a dopamine precursor causes strong dopaminergic side effects c) newborn seizures in acute phase can be successfully supressed by BZD d) Primidone partially changes to phenobarbital in the body and has similar pharmacological action to it

None are marked as correct. Correction: D+B (Unsure!?!) Pyritinol - used in cognitive disorders and learning disorders in children. Used for dementia and cognitive decline in elderly, little evidence of function.. Ida: c) should be correct. RW: i think they treat kids with phenobarbital BCD is correct according to newest test, articles about A exist saying it is used for encephalopathy in babies so perhaps true as well. TH

Oculovestibular reflex is tested when you irrigate the inner ear with water, what is the normal response?

Normal response: *Warm water*: Both eyes turn away from the ear with hot water + nystagmus toward the ear with hot water *Cold water*: Eyes turn towards the cold ear, nystagmus away from the cold ear.

Histology of normal innervated muscle vs reinnervated muscle

Normal: checkerboard distrubution of type 1 and 2 muscle fibers Reinnervated: fiber types are grouped together "type grouping"

How is Benzalconium normally used?

Normally used as Benzalkonium chloride. Benzalkonium chloride is used as an active ingredient in many consumer products: - pharmaceutical products such as eye, ear and nasal drops or sprays, as a preservative - Personal care products such as hand sanitizers, hygienic towelettes and wet wipes - Aftershave, deodorants, hair products and cosmetics

Benzalkonium is an antidote for TCA? Benzalokonium is used for artificial tearss?

Not antidote Yes (It is a preservative used in artifical tear solutions, but due to damaging effects it is today on its way out of the market!)

1) Typical modifications of sleep architecture depression belongs multiplication stages 3 and 4 NREM sleep and REM sleep reduction 2) Typical symptoms psychotiske in Manila include inserting and withdrawing thoughts 3) Increased sleepiness (hypersomnia) was in depression occur 4) The most common co-morbid mental disorders in the patient and bipolar disorders are anxiety disorders and addictions.

Nothing is marked Correction: C - depressed people have either insomnia or hypersomnia D - true according our pp! Thought insertion and withdrawal are typical for schizophrenia, not for mania

What is obsessive compulsive disorder?

Obsessions or compulsions that take up a considerable amount of time in one's days, and causes impairment in that person's normal day, social, academic, or occupational functioning. Individual is aware that the obsessions/compulsions are causing impairments in daily functioning.

The narcolepsy may manifest by: a. attacks of imperative falling asleep b. sleep paralysis c. sleepiness during the whole day d. sleep apnoea syndrome e. attacks of cataplexy

Old test: a. attacks of imperative falling asleep e. attacks of cataplexy Correction: a, b, c,e Agreed --> "tetrad of narcolepsy, are cataplexy, sleep paralysis, hypnagogic hallucinations, and excessive daytime sleepiness."

65. For depressive mood is typical A. Sad and depressive feelings with sleep and appetite disturbance B. Sad and depressive feelings typically without sleep and appetite disturbance C. Sad and depressive feelings with simultaneous energy disturbance D. Sad and depressive feelings with disorientation in time

Old test: A Correction: A+C A. Sad and depressive feelings with sleep and appetite disturbance C. Sad and depressive feelings with simultaneous energy disturbance

a) patient in stupor is at risk for hyperventilation b) panic attack can last for days c) in pathophys OCD enforced by basal ganglia d) delusion are false sensoric perceptions that occur without outer stimulus

Old test: A + C Correction: C OBS! On the test viewing from February, a) is marked as FALSE. Positron emission tomography and functional magnetic resonance imaging have shown increased brain activity in the frontal lobes, basal ganglia and cingulum in OCD patients as compared to controls

38. Withdrawal following opioid abuse is characterised by: A. Dysphoric mood B. Nausea C. Muscle pain D. Lacrimation E. None of the above mentioned F. A and D are correct

Old test: A+D Correction: A,B,C,D Withdrawal (pp) o Tearing, rhinitis, stomach pain, muscle pain or even convulsions, pilo-erection, sweating, increased heart frequency, increased blood pressure, nausea and vomiting, insomnia, mydriasis and strong craving

a) locus coeruleus plays a role in worry stimulus b) agoraphobia belongs among main signs of fear in situations where the worst opportunity of effect than attainment of help c) panic disorder differs from generalized anxiety disorder by vegetative signs d) acute stress reaction starts to *subside* within 48h

Old test: A, B Correction: A+D, unsure about meaning of B! *Agoraphobia*: anxiety about being in situations from which escape might be difficult or embarrassing D: Acute stress reaction: symptoms disappear within 48 hours.

Which of the following match a) Peripheral neuropathy can develop with deficiency of vit B12 b) During the anterior root damage of the spinal cord there's electric irritation of the muscles innervated by those roots c) During the anterior root damage of the spinal cord there's spasms of the muscles innervated by those roots d) Apraxia is damage of the reflective motor skills

Old test: A, C Correction: A C) should be false: LMN signs, meaning flaccid instead of spastic. For C it could be talking about fasciulations/fibrillations which does happen in LMN

Which is the right statement(S) a) Disorder of association can present as rhyming or playing with the words, and the patient connects the words accodring to the phonetic similarity b) In the presence of confusion state we ask the question: Is anything special or unusual happening around? c) Perseveration is a persistent response to the past stimulus even though the new stimulus has been present d) The adequate question in order to figure out whether the person has imperative hallucinations is: How did you sleep last night?

Old test: A,B,C Correction: A,C A: CLANG ASSOCIATION B - these are question for hallcuinations, in confusion state we should ask for name, place, date and time..

a) BZD are addictive b) BZD can cause anterograde amnesia c) diazepam is a BZD with short time of action d) adverse effects of BZD include orthostatic hypotension and tachycardia

Old test: A,B,D Correction: A,B

From the following choose the correct statements a) Nicergolin is characterized by cognitive rather than general nootropic effect b) The mechanism of action of gabapentin is to reduce excitation of GABA c) Donepezil in Alzheimers disease positively affects serotonin neuronal pathways d) Lamotrigine blocks Ca channels with subsequent stabilization of the neuromuscular junction

Old test: All are wrong Correction: A is correct in the newest test Nicergoline is used in senile dementia Donepezil - acetylcholine Lamotrigine - antiepileptic, Na channels

a) compulsion and obsession are signs of panic disorder b) pick vision (optic illusion of bent or walk through walls) is a sign of organic affect on the brain stem c) delusion belongs to formal thought disorder d) delirium doesnt belong to consciousness disorders

Old test: B + C Correction: B (??) False: C) delusion belongs to formal thought disorderder - It is a disorder of content of thought! D is correct in the newest version of the test

Which of the following is true a) In the neuropathic pain treatment the treatment of choice is SSRIs and SNRIs b) Triptans stimulate 5-HT1D receptors c) IN neuropathic pain treatment SSRIs (ex: escitalopram) is used as an antidepressive d) Triptans effect is stronger than the combination of paracetamol + metoclopramid

Old test: B+C Correction: B a) In ppt from school it says that TCA's are drug of choice. c) IN neuropathic pain treatment SSRI is used as an antidepressive (FALSE?, Its used as an analgesic, not antidepressant)

Histological findings in the previous disease? (JC polyoma)

Oligodendrocytes with intranuclear inclusions and astrocytes with pleiomorphism

Choose: a) 'Red neuron' is a morphological image of acute intracerebral...? b) Red encephalomalacia is the same as traumatic intracranial hemorrhage c) Cerebral amyloid angiopathy is most often associated with amyloid B peptide d) The most common place for intracerebral bleeding is in the mesencephalon idue to a common rupture lenticulostriata a.

Old test: C B could be correct if it didn't say "traumatic", and it should be intracerebral! :) *Red softening* occurs in the event of a hemorrhage, usually of the middle cerebral artery. *White softening* occurs in areas of the brain that experience little or no blood flow and therefore come to contain dead neural tissue. *Yellow softening* is a result of athersclerotic plaque building up in the arteries of the brain combined with yellow lymph.

a) irreversible mm weakness is characteristic for myasthenia gravis b) patient with absolute expressive aphasia understands words and text c) titubation (staggering) can be a sign of cerebellar dysfunction d) parkinson sy. can be provoked by dopaminergic receptor antagonist

Old test: C+D Correctino: B+C+D C - titubation is typical for cerebellar lesions D - parkinsoniam syndrome

14. which ones of the following is correct: a) Visceral pain is mechanical stimulation of the gray matter of the brain (eg. During operation) b) the most successful neurosurgical practices in intractable pain belongs to extralamniscal myelotomy c) Symptoms of pain may be the presence of skeletal muscle contraction d) The most successful neurosurgical practice for intractable pain belongs stereotactic operations on ventral roots of the spinal cord

Old test: C+D Correction: C+B Ventral roots of spinal cord are involved in motor function.. (Visceral pain is pain that results from the activation of nociceptors of the thoracic, pelvic, or abdominal viscera (organs)

70. Clinical signs of delirium does not include: A. Agitation B. Consciousness deterioration C. Aggravation D. Increased or excessive suggestibility

Old test: C. Aggravation Correction (Elise+Klara): D. Increased or excessive suggestibility

a) Cannabis doesnt cause orthostatic hypotension as an acute effect b) cannabis can typically provoke euphoria, hysterical laughter, decelerated perception of time, decreased appetite and sprayed conjunctiva and diarrhea c) alcohol has only excitatory effects d) biological factors dont play any role in the development of addiction

Old test: No correct answers "Discussion" B -False Diarrhea and decreased appetite is not effects of cannabis. Typical is hunger! A - False! Cannabis -->Lower BP while standing (according to pp) C - False! Alcohol has depressant effect D - False! Biological factors can influence addiction development Synne agrees with all being false :-)

Amyloid angiopathy is due to AL amyloid?

Old test: Yes Correction: No It is characterised by the deposition of ß-amyloid (Aß-40) in the media and adventitia of small and medium sized arteries of the cerebral cortex and leptomeninges Cerebral amyloid angiopati (CAA) er en vanlig småkarssykdom i hjernen, karakteriseres av progressiv avleiring av amyloid beta peptid i små og mellomstore kar i hjerne og leptomeninger. RR: should be A-beta (AS - senile) amyloid according to German Pathology book -->> so wrong

Epidural bleeding can lead to meningeal symptoms?

Old test: Yes Correction: No Meningeal symptoms are associated with subarachnoid bleeding. Cant find any association with epidural. K: I would guess it can cause compression --> mening. sy. ?

Side effects of prostaglandin analogues are? (Probably meant as a SE of glaucoma treatment!)

On eye lashes: you'll get ticker and more eyelashes Discoloration of iris: • Mostly irreversible • Very typical for blue eye patient o Blue eyes --> brown o Brown eyes --> even darker eyes!

The difference between suicide and "self kill" is ?

Only orientated and healthy people commit suicide, if you hear voices telling you to kill yourself this is different

Optic manifestation of pituitary tumor

Optic chiasma compression --> visual field defect Ocular motility defect Diplopia Nystagmus (optic atrophy, pituitary apoplexy)

How does Chlorhexidine work?

Osmotic destabilisation (loss of K+ and PO42-); at higher concentrations precipitation in cytoplasma

What is a red neuron? And is it related to ischemia?

Pathological finding, indication of acute neuronal injury, followed by apoptosis or necrosis Found in the first 12-24 h after stroke Red color due to pyknosis or degradation of the nucleus and loss of nissl bodies (which are basophilic), leaving only proteins, which stain eosinophilic

How does hemiparetic walking look like?

Patient does circumduction with his paretic leg (throws it around) GAIT, in general: RW: * upper motor neuron function - hemiplegia: decreased arm swing on affected side; circumduction of extanded leg, pronation of arms when held outstretched with palms up, flexion of arm when walking on sides of feet * lower motor neuron function - foot drop, cannot walk on heels, e.g. peroneal palsy - a steppage gait with lifting his leg high with each step (it results from foot-drop) * vestibular syndrome: the patient is reeling and falls to one direction (peripheral) x to various directions (central type) * lumbar disc: inability to walk on heels or toes on one

Which patients often have somatization?

Patients with alexithymia (inability to describe feelings with words)

Classical conditioning (associative learning)

Pavlovs dog, eye conditioning - unconditioned stimulus (US) is paired with conditioned stimulus (CS), which is then able to induce a behavioral response separately Does not require free will Cerebellum connects the US and CS

What is dysthymia?

Persistent depressive disorder with less severe but longer lasting symptoms than major depressive disorder; Present for atleast 2 years More common in women

What is Alexithymia?

Personality construct characterized by the sub-clinical ability to indentify and describe emotions in the self There is a dysfunction in emotional awareness, social attachment, and interpersonal relating

Drug of choice of choice for neuropathic pain?

Pharma pp: antidepressants, anti epileptics (anticonvul.), opiods and locally acting drugs. Carbamazepine - drug of choice in trigeminal neuralgia Tricyclic antidepressant - Amitriptylin is drug of choice in post-herpetic neuralgia and central pain.

3 components of working memory

Phonological loop Visuo-spatial sketchpad Central executive unit

Top 6 antidepressants

Physical activity - D and 5-HT signals, anti-aging Omgea 3 Sunlight Helthy sleep Anti-ruminative activity (avoid dwelling, let negative thoughts out) Socail connection

Name some agonists of cholinergic receptors?

Pilokarpin

Causes of bulbar syndrome? (nuclear)

Polyradiculoneuritis Guillan-Barre sy Brainstem lesion Tumors, meningoencephalitis, motor neuron disease

Other tumors

Primary lymphomas - B cell, high grade NHL, associated with EBV - Perivascular infiltration, multifocal - Common in IS patient Pinealoma, pinealoblastoma, seminoma Neural sheet tumors - Schwannoma (Antoni A/B, Verocay bodies), MPNST - neurofibroma

Anterograde amnesia

Problem with encoding memory

What is parietal syndrome?

Problems in spatial orientation are produced by *damage to the parietal lobe*, typically in the right hemisphere. Spatial abilities are used when one forms a "mental map" of an area, or when one figures out how shapes will fit together. For example, in the acute phase (soon after injury) they get lost easily. What about neglect if it is in non-dominant hemisphere? Teuber test http://www.intropsych.com/ch02_human_nervous_system/parietal_syndrome.html

Basal ganglia lesion

Procedural learning dysfunction

Bulbar syndrome - which way does the tongue deviate?

Protruded --> toward lesion At rest --> away from lesion

Consequences of maltreatment

Psychiatric disorders, drug abuse, risky sexual behavior, deliquency In childhood - increased amygdala volume In adulthood - smaller hippocampus (not if it occured in childhood)

What are the Adverse Effects of Baclofen?

Psychiatric reactions - unpredictable. CNS adverse effects: a. Drowsiness. b. Confusion. c. Fatigue. d. Convulsions Generalized hypotonia. Hypotension. Reduce dose in patients with moderate renal insufficiency Withdrawl from baclofen can manifest as withdrawl syndrome similar to benzo-withdrawl syndrome

Variant CJD

Pulvinar sign Florid plaques

Bacterial infections forms

Purulent meningitis Purulent encephalitis Brain abscess (acute and chronic)

What are Schneider's first-rank symptoms:

RW *Schneider's first-rank symptoms:* A memory device that is frequently used to remember the first rank symptoms is *ABCD*: *A*uditory hallucinations, *B*roadcasting of thought, *C*ontrolled thought (delusions of control), *D*elusional perception. - Auditory hallucinations - Voices heard arguing or giving instructions (2nd person) - Voices heard commenting on one's actions (running commentary, usually describing the patient in the 3rd person) - Somatic/thought passivity experiences (delusions of control / of being controlled) - Thought echo (thoughts being heard aloud) - Thought withdrawal - Thought insertion (thoughts are ascribed to other people who are intruding into the patient's mind) - Thought broadcasting (also called thought diffusion) - Delusional perception (linking a normal sensory perception to a bizarre conclusion, e.g. seeing an aeroplane means the patient is the president) - Made feelings - Made impulses and drives - Made volitional acts A memory device that is frequently used to remember the first rank symptoms is *ABCD*: *A*uditory hallucinations, *B*roadcasting of thought, *C*ontrolled thought (delusions of control), *D*elusional perception.

What are some primary neurotropic viruses?

Rabies VZV

Lyssa

Rabies virus from animal bite Intraneuronal transport into CNS Formation of *Negri bodies* in cytoplasm

What does lateral hypothalamic stimulation lead to ?

Rage, attack

What is Devic disease?

Reaction against aquaporins 4. *Neuromyelitis optica*, is an autoimmune, inflammatory disorder in which a person's own immune system attacks the optic nerves and spinal cord. This produces an inflammation of the optic nerve (optic neuritis) and the spinal cord (myelitis). Although inflammation may also affect the brain, the lesions are different from those observed in the related condition, multiple sclerosis. Spinal cord lesions lead to varying degrees of weakness or paralysis in the legs or arms, loss of sensation (including blindness), and/or bladder and bowel dysfunction. RW: simultaneous appearance of symptomatic foci in the optic nerve and spinal cord; mainly affects young women; elevated CSF protein; no oligoclonal bands in CSF; inflammatory axonal damage

Reward circuit

Reinforces behaviors when activated. The circuit includes the dopamine-containing neurons of the ventral tegmental area, the nucleus accumbens, and part of the prefrontal cortex.

Implicit learning

Relatively unaffected by amnesia No effect of aging Not affected by IQ

Medial parabrachial nucleus function?

Relay information from the taste area of the solitary nucleus to the ventral posteromedial nucleus of thalamus

What are obsessions?

Repeated intrusive thoughts, images, impulses or doubts entering the patients minds repeatedly. They are perceived as the patients own, and they are *unwanted*

what is somatization?

Reporting somatic symptoms in response to a psychosocial stressor, and seeking medical help for the somatic problem.

Hypotheses to explain phantom limb pain:

Researchers have advanced a number of hypotheses to explain phantom limb pain. 1. Maladaptive changes in the primary sensory cortex after amputation (maladaptive plasticity) 2. A conflict between the signals received from the amputated limb (proprioception) and the information provided by vision that serves to send motor commands to the missing limb 3. Vivid limb position memories that emerge after amputation

What are the adverse effects of Diazepam?

Respiratory distress. CNS effects: - Ataxia. - Confusion. - Drowsiness. - Paradoxical aggression. • Dependency. Also possible in fetus, avoid in pregnancy.

What are adverse effects of Cloazepam use?

Respiratory distress. CNS effects: a. Ataxia. b. Confusion. c. Drowsiness. d. Paradoxical aggression!!! e. Dependency. f. FLUMAZENIL: BZP antagonist, dont use if BZP are treating seizures.

What are the adverse effects of Tolcapone?

Risk of hepatotoxicity

What is the difference between Risperidone and Domperidone?

Risperidone is used centrally (CNS), while Domperidone dont cross the BBB and is therefor only working in the periphery.

Which part of the spinal cord is affected in conus lesion?

S3-S5 (saddle anesthesia) But remember that conus medullaris is located at L1-L2! MA: according to the book: conus medullaris is S3-S5. Epic onus medullaris is L5-S2.

What are the Adverse effects of Amitryptiline?

SEVERE ANTICHOLINERGIC Effects (avoid in elderly people): - Blurred vision. - Dry mouth. - Constipation. - Difficulty with micturition. CARDIAC TOXICITY, increased risk of arrhythmias if given with other drugs that prolong the QT interval. (R. Antipsychotic drugs!!) • Hypotension in 20% of pts. • Lowers the seizure threshold! • SIADH syndrome and hyponatraemia consequently (R. Central pontine myelinolysis=Osmotic demyelination syndrome if corrected too quickly!!) • Serotonin syndrome.

indications for atypical antipsychotic?

Schizophrenia Bipolar disease Agitation with dementia Anxiety Disorder OCD

Name 3 groups with atypical antipsychotics?

Second-generation („Atypical") antipsychotics i. *MARTA* (multi acting receptor target angents) - clozapine, olanzapine ii. *SDA* (serotonin dopamin antagonist) - risperidone iii. *Other* e.g. sulpiride, quetiapine, aripiprazole

What is status verminosus?

Seen in chronic ischemia Multiple small foci of gliosis subsequent to focal necrosis of ganglionic cells

What happens if Carbamazepin is stopped abbruptly?

Seizure

Definition of demyelinating diseases

Selective loss of myelin in CNS and PNS with relative sparing of axons. However, there is limited capacity to regenerate normal myelin -> secondary axondal damage

2 types of declarative (explicit) memory

Semantic - facts (capital of France is Paris), independent of spatial and temporal context Episodic - events (mental time travel)

Classification of memory

Sensory/perceptual/immediate memory (iconic, echoic) Short-term/working memory (visual, verbal) Long-term memory (facts, events, skills)

What is suggested biochemical explanation for panic disorder?

Serotonin deficiency in limbic system --> Chronic hyper-activation of the limbic system

What does the removal of cerebral hemispheres lead to (speaking about rage) ?

Sham rage

What is sham rage?

Sham rage is a controversial term used to describe behavior such as *biting, clawing, hissing, arching the back* and *"violent alternating limb movements"* produced in animal experiments by REMOVING THE CEREBRAL CORTEX, which are claimed to occur in the absence of any sort of inner experience of rage. These behavioral changes are *reversed* with small lesions in hypothalamus *Bard* concluded that the hypothalamus is critical for the expression of aggressive responses and the cortex is responsible for inhibiting and directing those responses.

What are side effects of Metoclopramide?

Side effects. d. Metabolized by the liver_be careful when liver failure. e. Contraindication: i. GIT obstruction. ii. PD. iii. Phaeochromocytoma. f. Indications: i. Nausea caused by GIT, liver or biliary disease. ii. Chemotherapy high doses-emesis(replaced by 5HT3 receptor antagonists) g. INEFFECTIVE in motion sickness, give antihistamines.

Operant conditioning

Skinner rat - learning to perform some action to obtain reward or avoid punishment Appetitive motivation (reward) - increased reponse rate Aversive motivation (punishment) - reduction in response rate

When to use zopiklone?

Sleeping pill - cause depression of CNS Treating insomnia

Indication of Baclofen?

Spasms It is a muscle relaxant (MS, stroke, spinal cord tumors)

which anxiety disorder is the most common?

Specific phobias

Lesion between pontine center for urination and sacral center for urination will affect the bladder in which way?

Spinal cord injury above the lumbosacral level eliminates voluntary and supraspinal control of voiding, leading initially to an *areflexic bladde*r and complete *urinary retention*, followed by a slow development of *automatic micturition* and neurogenic detrusor overactivity (NDO) that is mediated by spinal reflex pathways. However, voiding is commonly inefficient owing to simultaneous contractions of the bladder and the urethral sphincter (detrusor-sphincter dyssynergia) The sacral micturition center is located at the S2-S4 levels and is responsible for bladder contraction. The pontine micturition center acts as a central relay and may play a role in the coordination of external sphincter relaxation with bladder contraction. The cerebral cortex plays an inhibitory role in relation to the sacral micturition center.

What is SCIWORA?

Spinal cord injury without radiographic abnormality - Spinal injuries (in cervical region) without any identifiable bony or ligamentous injury on x-ray or CT. - Injuries are found on MRI - injury of spinal cord, ligaments or vertebral body end plate SCIWORA should be suspected in patients subjected to blunt trauma who report early (immediate) or transient symptoms of neurologic deficit or who have existing findings upon initial assessment.

What is Sciwora?

Spinal cord injury without radiological abnormality

What is Dithiaden?

Stron selective H1 - H1-antihistamine. - Hypnotic, antiadrenergic and very weak anticholinergic and antiserotonergic effects.

Phineas Gage

Survived penetrating brain injury through prefrontal cortex

Measles

Subacute sclerosing panencephalitis

How does the EEG normally look on an absence seizure?

Sudden onset of 3-Hz generalized symmetrical *spike-and-wave* or multiple spike-and-slow wave complexes. Diffuse spike-wave

How does benzalkonium work?

Surface active ammonic base - destabilisation & enhanced permeability of cell wall

Lesions of chiasma due to tumors of hypophysis leads to what kind of visual loss?

Temporal heteronymous hemianopsia

Thomayer

Tests hypomobility and hypermobility of the spine. Some czech guy invented it.. (SHOCKER) You stand in upright position, and bend forward.

What does the lesion of ncl. centralis and lateralis lead to?

The inability to learn and express fear

Symptoms of lower part lesion?

The lower part of lumbar enlargement - epiconus - L5 - S2 segmental lesion: i. Paresis resembles radicular lesions of L5 + S1 - Easily made the wrong diagnosis! ii. Weakness of extensors predominates - perineal muscles, gluteal muscles, flexors of the crus iii. Deep tendon reflexes L2 - S2 are absent iv. Sensory impairment (on the dorsal part of the leg and from the knee distally) v. Automatic bladder (reflex neurogenic bladder) vi. Sexual dysfunction vii. Investigation - MRI must be focused to the T12-L1 vertebra level! viii. Cord compression is not caused by disc prolapse, but often by a tumour!

Symptoms of upper lumbar segment lesion?

The upper part - lumbar segments L1 - L4: i. Paresis of the thigh muscles - weakness of flexors and adductors of the thigh and of extensors of the shin (crus) ii. Deep tendon reflexes L2-4 are absent iii. Sensory loss in areae radiculares iv. Urinary incontinence (sympathetic lesion) - rare v. Cremasteric reflex is absent in L1-2 segmental lesion

Where can you find Hirano bodies?

They are intracellular aggregates of actin and actin-associated proteins first observed in neurons. Alzheimers disease and Creutzfeldt Jakob disease (test asks for ALS, but I cant find anything about ALS) Ida E: I´ve seen several articles about ALS + hirano bodies too, so think this should be included as well. Synne agrees! I have also seen it when I googled it.

Prion disease form vacuoles in white matter in brain?

They form in gray, and also in white (I think) https://www.ncbi.nlm.nih.gov/pubmed/21073844 Ida E: From robbins it seems like it´s only grey matter though. RW: only grey

WHat is dangerous about opioids?

They supress breathing center

LSD works best on type 2 receptors?

They work best on Serotonin 1A receptors or Dopamine 3 receptors LSD works as an agonist of: (from pp) 1. DA receptors (D3 > D4 > D2) 2. 5-HTR (1a >2a >1d >2c >7)

Thomayer test - bad enligsh -->

Thomayerova distance Thomayerova distance, which is sometimes referred to as a simple crouch test. Display and evaluate non-specific mobility throughout the spine. It consists bent conducted from a standing position, when in extreme position, measure the distance of the third finger from the pad. Under normal result is considered touch of your fingertips, yet we tolerate distance of 10 cm, 30 cm above, it is the clear pathology. During the test we have to be careful about certain distortion - movement can be compensated by moving the hips or vice versa is restricted due to shortening of the knee flexors (patient shrugs her knees and feel pain in the knees). Except hypomobility test can investigate and significant hypermobility, when the patient touches the floor the entire hand or forearm (it is considered a very important disorder of connective tissue). Increased ligament laxity and thus the positive examination hypermobility is more frequent in women.

- Stereotactic surgery of thalamus is the best method for the treatment of neuropathic pain

Trie

- Basal ganglia participate in the obsessive-compulsive disorder pathophysiology

True

- Experimantal autoimmune encephalitis is induced in the mouse using myelin basic protein

True

- In supranuclear disturbances of eye movements, conjugate gaze movements fail and the patient has no diplopia

True

- Metabolic acidosis decreases brain activity

True

- Pseudobulbar palsy: increased masseter reflex and incontinence of affect (uncontrollable crying and laughing)

True

- Psychosocial stress causes progression of Alzheimer's dementia

True

- Right posterior parietal cortex damage causes loss of visual objects detection

True

- Subdural hematomas can cause dementia

True

- Uremic neuropathy is usually distal axonal sensomotoric polyneuropathy

True

20´s gustatory aura, brief periods of unresponsiveness -> diagnosis is complete partial epiletic seizure (I didn´t write down the whole statement)

True

A 19-year-old man with the first psychotic episode describes the week at home prior to hospital admission: "I..I watched TV, but the newspaper didn´t come, I..David is at school, too. Sometimes it´s better to be alone, you know, to save for a rainy day." It is an example of derailment (loosening of associations)

True

A central palsy may develop in the Brown-Sequard syndrome in the muscles innervated by motoneurons situated below the lesion

True

A central paraparesis (of lower extremities) without sensory impairment can be due to a parasagittal lesion (meningioma)

True

A combination of acetylosalicylic acid with dipyridamol is used for the secondary prevention of ischaemic stroke recurrence

True

A conversion is a defense mechanism in which a mental problem is replaced by a somatic problem

True

A damage to the posterior columns of spinal cord leads to the impairment of tactile sensitivity

True

A decorticate rigidity is due to lesions above mesencephalon (cortical or diencephalic lesion)

True

A delay of visual evoked potentials can be expected in a young paraparetic woman, diagnosed with multiple sclerosis

True

A delirium is one of the most severe forms of alcohol and benzodiazepine withdrawal state

True

A dental drill is an example of the potentiation of startle reaction with a conditioned stimulus

True

A finding of papilloedema on the eye fundus is most commonly due to an increase in intracranial pressure

True

A homonymous hemianopia may arise from a lesion of the occipital cortex

True

A homonymous hemianopia may arise from a lesion of the optic tract

True

A lesion to epiconus presents similarly to the radicular lesions to L5 and S1

True

A mixed paresis is caused by a dual lesion, when upper and lower neurons are damaged at the same time

True

A patient with a central hemiparesis of capsular type walks with a circumduction of the affected leg

True

A patient with a visceral pain has a higher BP and pulse frequency than normal

True

A patient with peroneal palsy has a steppage gait, lifting his leg high with each step due to a foot-drop

True

A renal failure may cause polyneuropathy

True

A spinal cord lesion below the cervical enlargement may cause paresis or plegia of the legs only

True

A statoacoustic neurinoma affects mainly medium and high frequencies

True

A subdural hematoma is due to a venous bleeding

True

A typical morphological feature of progressive supranuclear palsy is an astrocytic cytoplasmic clump of tau protein

True

A typical morphological features of Parkinson´s disease is a Lewy neurite

True

A typical symptom of frontal-type dementia is a speech dysfunction

True

A unilateral hyposmia or anosmia can be a manifestation of a tumour on the basis of the frontal lobe

True

A weight change is not a characteristic symptom of mania

True

Acute intracranial hypertension: you can reduce pressure by hypothermia and hyperventilation

True

Adverse effects of some antipsychotics may include agranulocytosis

True

Amaurosis fugax occurs with the insufficient blood supply from the ophthalmic artery

True

Amaurosis fugax: insuff. blood supply of ophthalmic artery

True

An increasing tension that precedes an impulsive act is typical for impulse control disorders

True

Anhedonia is the loss of pleasure

True

Antidote to paracetamol is N-acetylcysteine

True

Apraxia develops as a result of disconnection between brain regions

True

Atypical antipsychotics are better tolerated than typical antipsychotics

True

Benzodiazepines have a myorelaxant effect

True

Benzodiazepines have a specific antagonist

True

Benzydamine acts antiseptically

True

Bilateral amygdala lesions result in fear reduction

True

Bipolar disease -> migraine is frequent

True

CADASIL can be diagnosed by the presence of notch-3-protein deposits in the blood vessels walls on skin biopsy

True

CADASIL is an autosomal dominantly inherited disease of small blood vessels leading to a progressive vascular dementia

True

Caloric testing is used to determine the reactivity of the vesticular system

True

Central hemiparesis presents with low abdominal skin reflexes or absent on same side

True

Chlorpromazine blocks histamine receptors

True

Cognitive ACh esterase inhibitors have side effects: nausea, vomiting and diarrhea

True

Crack: synonym for free base of cocaine

True

Crosstalk refers to the perception in contralateral ear when hearing is tested

True

Damage of apraxia is referred to tight frontal and parietal cortices

True

Damage to mesencephalon: contralateral spastic hemiparesis with hemiataxia + ipsilateral peripheral palsy of oculomotor nerve

True

Dextramethorphan (DXM), which is contained in anti-cough drops, behaves as NMDA antagonist in high doses

True

Eating disorders are disturbances of instincts

True

Epileptic seizures are NOT necessarily accompanied by disturbances of consciousness

True

Fear is rather a phasic phenomenon, while anxiety is tonic

True

Gama motoneurons are located in ventral spinal horns

True

Gentamycin can be used for the treatment of chronic tinnitus

True

Granule cells: present during resorption of cerebral ischemia

True

Hemiparesis develops as a consequence of damage to the contralateral capsula interna

True

Hexylresorcinolum is bacteriorstatic at 1% and bacteriocidic at >1%

True

In the Huntington´s disease, cholinergic and GABA neurons are predominantly affected by the neurodegeneration and the dopaminergic activity is increased

True

In the acute transversal thoracic spinal cord lesion, a pseudo-flaccid paralysis occurs with anaesthesia up to the level of the lesion

True

In the multiple sclerosis, the autoimmune targets are antigens of myelin sheaths surrounding CNS neural fibres

True

Interferon beta or other immunomodulators can prolong the remission in some forms of the multiple sclerosis

True

Interruption of peripheral motor nerve -> development of atonia in innervated muscle

True

Intracranial bleeding can cause ischemia?

True

Ipsilateral optic nerve lesion causes loss of vision on the same side as the lesion?

True

Ischaemia may lead to epileptiform seizures

True

Language prosodia is influenced by right parietal lobe

True

Levodopa (L-DOPA) crosses the blood brain barrier

True

Lithium: prevention of manic periods in bipolar disease

True

MRI is an essential imaging method if SCIWORA is suspected

True

MTT (mean transit time) is a suitable parameter to evaluate a brain perfusion CT scan

True

Marchiafava-Bignami disease is a progressive neurological disease of chronic alcoholism characterized by corpus callosum demyelination and necrosis and subsequent atrophy.

True

Memantine lowers an excessive calcium influx into neurons through the partial inhibition of NMDA glutamate receptor and thus it prevents neurodegeneration

True

Metaanalysis of fMRI-brain activation is used during perception of auditory hall in schizophrenia (don´t remember the full statement)

True

Methylphenidate could be used as an anxiolytic

True

Midazolam is used for a conscious (procedural) sedation

True

Montreal cognitive test: short and comprehensive, assesses cognitive functions

True

Mood-congruent delusion is present in mood disorders?

True

Most freq. obsessions are: contamination, catastrophe, blame, accuracy, precision and symmetry

True

Neuroblastoma does not rank among the tumours of glial origin

True

Neuromuscular junction is a part of muscle unit - true or false?

True

Neuropathological feature of Parkinson´s -> degradation of dopamine pathway to neostriatum

True

Oligoclonal bands specific for MS in CSF is used for diagnosis of MS?

True

On CT-perimyelography, a contrast medium is administered intrathecally by lumbar or suboccipital puncture

True

On a deeper retroflexion of the trunk, the patient with paleocerebellar syndrome does not flex his knees and falls backward

True

Optic agnosia is cognitive disturbance

True

Paraesthesia is a spontaneous feelings (without an external stimulus)

True

Paranoid delusions include persecutory delusions, delusions of reference and control, delusions of grandeur is macromanic delusions

True

Parkinsons syndrome manifests with micrographia, increased salivation and hypomimia

True

Patients with somatic illness more commonly also have anxiety?

True

Pegaptanib is used to treat macular degeneration

True

Pick´s visions are signs of organic brainstem disorder

True

Poliomyelitis anterior acuta -> caused by polioviruses

True

Positive symptoms of schizophrenia are associated with an altered blood flow in the parahippocampal gyrus

True

Syringomyelic dissociation - loss of pain and temperature sensation with relative retention of tactile sensation, related to a cavity in the central portion of the cord interrupting the decussation of nerve fibers.

True

Syringomylia: dissociated sensory loss may be seen

True

The Huntington´s chorea is a disease with trinucleotide repeat expansions

True

The Parkinson syndrome presents with micrographia, increased salivation and hypomimia

True

The TDP-34 peptide takes part in the pathophysiology of neurodegeneration

True

The alien hand syndrome is characteristic for a corticobasal degeneration

True

The anxiety syndrome includes various forms of fear with non-specific (vegetative) symptoms

True

The ascending reticular formation activated vigilance of the forebrain

True

The autoimmune response in multiple sclerosis is directed primarily against the myelin basic protein of oligodendroglial cells

True

The blockade of muscarinic receptors by tricyclic antidepressants results in the retention of urine

True

The central pontine myelinolysis is a toxonutritive demyelination which affects diabetic patients in a diabetic coma

True

The cerebral amyloid angiopathy is frequently associated with amyloid-beta-peptide

True

The corneal reflex is functional only with the intact trigemino-facial circuit

True

The dopaminergic mesolimbic system is involved in the formation of delusions

True

The epidural bleeding is life-threatening intracranial bleeding often associated with a traumatic fissure of the skull and a damage to meningeal arteries

True

The glial fibrillary acidic protein (GFAP) is used as a diagnostic marker of astrocytic tumours

True

The history of Raynaud´s disease and of ischaemic stroke is a contraindications to the use of triptans

True

The injury to frontal horns of the spinal cord results in the atonia of innervated muscles

True

The locus coeruleus is involved in a phobic reaction

True

The main mechanism of action of amphetamine is the dopamine release from the presynaptic ending

True

The main symptoms of Parkinsons syndrome are hypertonia and hypokinesia

True

The metaanalysis of fMRI studies focused on the brain activation during perception of auditory hallucinations in schizphrenia shows a higher activity in Broca´s center, insula and superior temporal gyrus

True

The most likely clinical consequence of right posterior parietal cortex damage is an impaired detection of visual stimuli

True

The multiple sclerosis is a demyelinating disease

True

The multiple sclerosis may cause axonal damage

True

The multiple system atrophy falls among atypical Parkinsonian syndromes

True

The muscle weakness in lower limbs, perianal sensory deficit and sphincter problems are typical signs of cauda equina syndrome

True

The new variant of Creutzfeldt-Jakob disease presents with a typical hyperintensity on MRI scan located in the pulvinar thalami

True

The optic agnosia ranks among cognitive disturbances

True

The palsy of the oculomotor nerve produces the inability to move the eye inwards, upwards and downwards

True

The peripheral vestibular syndrome results in tonic deviations toward the affected side

True

The peripheral vestibular syndrome results in tonic deviations towards the affected side

True

The progressive multifucal leukoencephalopathy frequently occurs as a lethal complication of immune system diseases

True

The rapheal system of reticular formation is serotoninergic

True

The retrograde amnesia in trauma or concussion has a gradual character, as a newly obtained information is sensitive to damage due to insufficient transfer or consolidation into the long-term memory

True

The rupture of aneurysmatic walls of cerebral blood vessels typically results in a subarachnoideal bleeding

True

The symptoms of peripheral paresis may accompany the damage to cranial nerves

True

The therapy by pure dopamine precursors exhibits strong dopaminergic adverse effects

True

The time course of disease with attacks and remissions is typical for the multiple sclerosis

True

The treatment of multiple sclerosis is focused on a timely management of individual attacks and a prolongation of remissions

True

The tremor in a patient with cerebellar disorder is more pronounced at the end of movement

True

The upgaze palsy is not a characteristic symptom of Parkinson´s syndrome

True

Vasoactive drugs are used in treatment of tinnitus

True

Vasodilating agents can be used to treat tinnitus

True

Verbigeration is a meaningless repetition of certain words or phrases

True

We can use NSAIDs in acute migraine attack with a low intensity of pain

True

When peripheral motor nerve fibers are interrupted, an areflexia develops in the innervated muscles

True

When peripheral motor nerve fibers are interrupted, the electrical excitability of the innervated muscles changes

True

Working memory (a short-term retention and manipulation with information) is mainly associated with the activity of prefrontal cortex

True

Zolpidem is a fast-acting non-benzodiazepine hypnotic drug

True

- Donepezil and rivastigmin are competitive and reversible AchE Inhibitors

True (but this is not correct..) RW: they are both competitive adn rev ache inhib - check wiki

Damage to dorsal roots -> ataxia

True Ataxia = loss of full control of body movements

In microzoopsia you can see delirium tremens

True Delirium tremens (DTs) is a rapid onset of confusion usually caused by withdrawal from alcohol.

Myotonic dystrophy: AD disorder

True MA: From PP: 3) Myotonic Dystrophy (chloride transport abnormality) *dominantly-inherited* , progressive disease with myotonia and progressive weakness. Associated abnormalities include: cataracts, premature balding, cardiac arrhythmias, endocrine disturbances. Early atrophy and hypertrophy of selected fibers and increased central nuclei.

Panic disorder increases cardiovascular morbidity, true or false?

True HTN, MI, stroke

- Dementia praecox is historical predecessor of schizophrenia

True Krepelin

Frontal lesions would lead to eyes deviating to the same side of the lesion?

True (+ contralateral hemiparesis) (Pontine lesion is the opposite)

Telencephalon has four lobes

True (5 including insular lobe, and 6 according to wiki)

Zolpidem binds to GABA A receptors

True (RW: ambien zzzzzs (and binge eating at night without remembering))

The Marchiafava-Bignami disease is characterized by demyelination of the rostral part of the corpus callosum and septum pellucidum

True (RW: occurs in alcoholism)

Dystonia may result from a lesion to basal ganglia

True (Rw: dystonia - like a tic, sustained or repetitive muscle contractions result in twisting and repetitive movements or abnormal fixed postures. The movements may resemble a tremor. Dystonia is often intensified or exacerbated by physical activity, and symptoms may progress into adjacent muscles

A prevailing static tremor occurs primarily in the essential tremor

True (static tremor > essential in regards to non-parkinson RW)

a) TBC spondylitis usually affects C1-C2 area b) during CT perimyelography we apply contrast intrathecaly lumbally or through suboccipital puncture c) osteochondrosis on X-ray is characterized by decrease on intravertebral space d) to confirm osteochondrosis radiculography is used

True : C+D B - false (why ? ) (Think it is because its injected in the dural sac, which extends from conus medullaris down to s2, but not sure) A - false (TBC usually affect thoracic and lumbar area)

Oxycodon and naloxone - Oxycodo is a very strong painkiller related to morphine Naloxone can be taken simultaneously to counter the effects of oxykodon in the GI tract, since naloxone acts there primarily

True ? "Naloxone and oxycodone is a combination medicine used for around-the-clock treatment of severe pain that is not controlled by other medicines. Naloxone is included in this medicine to prevent the misuse of the narcotic ingredient" Oxycodon + naloxone - used for treatment of pain, ok to combine. - Prevents constipation - Oxycodon has low first pass effect, naloxone has high first pass effect - Oxycodon goes out into circulation, and naloxone is almost completely removed - Good for pain relief without causing constipation But Naloxone given i.v counteracts the opioids, and is used for overdoses RW i would say false? naloxone is opiod antagonist -to ccounter from respiratory depression but http://www.netdoctor.co.uk/medicines/aches-and-pains/a8707/targinact-oxycodone-and-naloxone/

Choose the right statement a) Homonymous hemianopsia can be caused by lesions of optic nerve b) Dissociative sensation disorder may arise during occlusion of medial cerebral artery c) In syringomyelia occurs dissociation of sensory loss d) In a young paraparetic woman with multiple sclerosis reduced concentration of protein in the CSF will be expected

True C Generalized sensation disorder MS lady - higher CSF proteins

Vigabatrin is suitable for the treatment of partial seizures in symptomatic epilepsy

True RW (Vigabatrin, also known as gamma-vinyl-GABA, is an antiepileptic drug that inhibits the breakdown of γ-aminobutyric acid (GABA) by acting as a suicide inhibitor of GABA transaminase)

Around 15% of those with depression commit suicide?

True RW: this is true if they mean 'attempt' to commit suicide "ppt says -' 15% of patients with depression attempts to commit suicide, mortality rate increased

- Do you ever experience your actions or thoughts being physically controlled by an outside force - inquires for delusions passivity experiences

True!

a) Citalopram is an SSRI antidepressive b) Lithium is suitable for prevention of manic phase of bipolar disorder c) Clozapine is an SSRI antidepressant d) Chlorpromazin blocks histamine receptors

True: A,B, D Clozapine - MARTA - atypical antipsychotic

Choose the right statements a) Levodopa crosses the BBB b) Inhibitory dopa-decarboxylase combined with levodopa does not cross BBB c) Unlike carbamazepine, valproate can be given in the absence seizures d) Topiramate is among the new AEDs

True: ACD Levodopa crosses BBB But the dopa-decarboxylase does *not* cross the BBB.. So depending how the question is in "proper" english, I think this (B) could be correct.

Which are correct: a) Multiple sclerosis is cerebrospinal and more common in men b) experimental autoimmune encephalitis is a mouse model of multiple sclerosis, conditioned immunization of the experimental animal of glial fibril acidic protein c) Devons optic neuromyelitis is directed at glial fibrilar acidic protein d) multiple sclerosis is an autoimmune disease

True: D MS is more common women The mouse model was injected with myelin basic protein Optic neuromyelitis is against aquaporin 4

Tumors from ependymal cells

Tumors are close to cavities (children 4th ventricle, adults spinal cord (NF type 2)), CSF metastasis is common and hydrocephalus due to obstruction Ependymoma (children) - grade II, pseudorosettes with central vessel and rosettes with central cavity Adults - in spinal cord

4 systems for CNS tumors

Typing - what tumor? - glioma, neuroma, meningioma... Grading - I-IV, WHO system Staging Rating

Types of attention

Unintentional (exogenous) - automatic, driven from outside, no effort Intentional (engogenous) - maintained by conscious will, you need to pay attention to something, effort is necessary

1. The urgent surgery for lumbar intervertebral disc prolapse should be recommended for: (look up answers) a) painful lumbar radicular syndrome with a sudden paresis of the lower extremity and a proved disc prolapse b) urine retention and prostate hypertrophy associated with _____ L2-S2 c) severe lumbar pain after heavy work in bent forward position and no available CT d) flaccid quadruparesis with pronounced cervical osteofytes

Unsure what they ask for here, but it is the cauda equina syndrome that require immediate surgery. I would guess a) maybe.. Cauda Equina syndrome: - Problems with bowel and bladder function (usually inability to pass urine). - Numbness in the saddle area around the back passage (anus). - Weakness in one or both legs Simen: a) is the same as radicular syndrome, lumbar stenosis, or typically sciatica - laminectomy to relieve pressure - not immediate surgery needed b) I think they mean any compressive lesion somewhere in L2-S2 segment, which will cause *cauda equina syndrome*, and this is considered a medical emergency RR: a+b

Pseudomembranes are produced after hemorrhagic strokes?

Unsure? (RW: ppt says ischemic - so i would say no) Pathophys pp says that pseudocysts are produced after ischemic strokes. Hemorrhagic strokes are not mentioned..

How long can adjustments disorders last?

Up to 6 months (Prolonged depressive reaction can last up to 2 years)

What has ketamine been used for in research?

Used as experimental models of psychoses, ketamine in the treatment of depression

When do we use Amitryptiline?

Used in treatment of: - Depression. - Panic disorder. (Anxiety) - Neuropathic pain.(R. Also SNRI=venlafaxine) - Migraine prophylaxis. - also used in lesser extent in psychosis and bipolar disease

What is Citalopram used to treat?

Used to treat: - Depression. - Panic disorder. (with paroxetine) - GAD and PTSD. - Do NOT use with MAOI. - Do NOT give if there is increased risk of bleeding, especially of GIT bleeding if combined with NSAIDs. SOS Can SSRIs be used in bipolar disorders? - NO, becasue they have Stimulant effect!

In Korsakoff psychosis, do we have loss of retrograde or anterograde memory?

Usually anterograde, but can be both

What is NOTCH3 associated with?

Vascular dementia and CADASIL (AD inheritance)

Which of these are used to treat tinnitus? 1. Beta blockers 2. vasodilators 3. beta-histine

Vasodilators Beta-histine

Top-down attention stream

Ventral frontoparietal network - reorientation and detection of new and behavioral relevant stimuli TPJ - temporo-parietal junction VFC - ventral frontal cortex

Which brain structures are involved in reward system and reinforcement ?

Ventral tegmental area Nucleus accumbens Amygdala

Which drug is used for macular degeneration?

Verteporfin (SCN inhibitor) - For use in the photodynamic treatment of subfoveal choroidal neovascularisation associated with macular degeneration or with pathological myopia. I.v. infusion, activation by non-thermal red light => cytotoxic derivates

What are Zoopsie hallucinations?

Visual hallucination of animals (It is not an hallucination of little animals talking to you)

AE's of Risperidon?

Weight gain DM Tardive dyskinesia Neuroleptic malignant syndrome

Declarative (explicit)

What? - requires attention, episodic (events) and semantic (facts) memory

What is group atrophy?

When a group of one type of muscle fibers becomes deinnervated

When does an adjustment disorder develop?

Within 1 month after a stressful life event.

When is the onset of acute stress reaction?

Within 1h

Hypermnesia

Without consciousness disturbance - threatening states, anxiety With consciousness disturbance - fever, hypnosis, LSD After electrical stimulation of temporal lobe - retention of forgotten memories

Clopidogrel is a fibrinolytic drug?

Wrong, antiplatelet drug

What type of inheritance does Duchenne have?

X-linked

Muscular dystrophies

X-linked: Duchenne and Becker Autosomal: Myotonic dystrophy

- In neuropathic pain, we can use SSRI - - In neuropathic pain, we can use SNRI - - In neuropathic pain, we can use TCA -

YES x3

For diagnosis of bipolar affective disorder is important to have mania and at least one episode of depression

Yees RW: unclear - dsm 4 or 5? http://apps.who.int/classifications/apps/icd/icd10online2003/fr-icd.htm?gf30.htm+ RR: according to (https://www.ncbi.nlm.nih.gov/pmc/articles/PMC3637882/) depression is NOT key for diagnosing BAD (1 in this case) -- they use DSM-4 - also the lecture notes contradict themselves any ideas????

Addiction is connected to dopamine, nucleus accumbens and the frontal cortex?

Yes

Alzheimers can be diagnosed genetically?

Yes

Astroglial cells are macroglial cells?

Yes

Audiometry should be performed in a soundproof room?

Yes

Biperiden and Benzatropin can be used in parkinsons as anticholinergic drugs ?

Yes

Both in bacterial and viral meningitis can lead to meningeal signs?

Yes

Schizophrenia is associated with a defect in communication or connection between temporal lobe seen by functional imaging

Yes http://www.ncbi.nlm.nih.gov/pmc/articles/PMC3181763/ "To date, numerous other structural brain differences between chronic patients with schizophrenia and controls have been reported from computed tomography (CT) and magnetic resonance imaging (MRI) studies. These include 1. non-localizcd reduced gray-matter and white-matter changes 2. temporal lobe volume reductions, 3. PARTICULARY anomalies of the superior temporal gyrus and TEMPORAL and frontal lobe white-matter connections, ic, arcuate, uncinate, and fornix"

Subarachnoidal hemorrhage can cause vascular spasm and can lead to ischemia ?

Yes This type of vasospasm increases the risk of an ischemic stroke. Vasospasm typically occurs 4-10 days after subarachnoid hemorrhage

Is supranuclear palsy among the Tauopathies?

Yes (Atypical parkinsonism)

In bulbar syndrome and pseudobulbar syndrome you have dysphagia and dysphonia?

Yes (Dysphonia, Dysarthria and dysphagia is present!) RW bulbar: bilaterla affection of causal/cranial nerves 9-12 or nuclei in brainstem: dysarthria, atrophy of tongue with fasciulations, dysphagia, drop of soft palate and diminshed/abscent gag reflex pseduobulbar: due to bilateral lesion of corticobulbar tract. clinical picture is similar to bulbar syndrome but tongue atrophy and fasciculations are absent, because it is a central lesion. Peripheral motor neurons are intact, jaw (masseter) rflex increased

BAD type II - usually depressive episodes with at least on hypomanic episode

Yes (No manic episodes,one ore more hypomaniac, one or more major depressive episodes)

Imputace and amputace of ideas are signs of first level of schizophrenia?

Yes (What is imputace and amputace?) Blunted affect and insertion of thoughts maybe? Synne: I think it is the same as thought insertion and thought withdrawal in first rank symptoms (imput and amput).. RW: translated from CZ: amputace - removal (of thoughts) / imputace (insert) - so agree with synne

Cervical olisthesis by 1 cm can lead to quadruplegia?

Yes *Olisthesis* means slipping/sliding of the vertebra The term spondylolisthesis refers to anterior displacement (forward slip) of vertebral body relative to one below. The term retrolisthesis refers to posterior displacement (backward slip) of a vertebral body relative to one below.

Can you use triptans with prokinetics ?

Yes *Prokinetic*: a type of drug which enhances gastrointestinal motility by increasing the frequency of contractions in the small intestine or making them stronger, but without disrupting their rhythm.

Are there glomeruloid formations in glioblastoma multiforme? What are glomeruloid formations?

Yes - Microvascular proliferation and glomeruloid vascular structures are important histopathological features of glioblastoma multiforme (GBM)

Amaurosis fugax can cause hypoperfusion of a.centralis retinae ?

Yes Amaurosis fugax is loss of vision in one eye due to a temporary lack of blood flow to the retina. it may be a sign of an impending stroke RR: Amaurosis Fugax can be caused by hypoperfusion of retinal artery. but it doesn't cause the hypo-perfusion

Tizanidin causes hypotension ?

Yes Because of the possibility of tizanidine *lowering blood pressure*, patients should be warned about the risk of clinically significant orthostatic hypotension that is lightheadedness on arising It is a central myorelaxant!

Alzheimers disease leads to anteograde amnesia?

Yes Both anterograde and retrograde

When dorsal columns are affected in spine, does standing and walking with closed eyes lead to worsening the symptoms?

Yes But not in cerebellar lesions I think! Then it doesn't matter if you have them open or close, it is bad anyways...

In MS there is elevated amount of proteins in CSF?

Yes Elevated levels of IgG and oligoclonal band proteins

Early diagnosis of parkinsons can be due to findings of alpha-synuclein in olfactory nerve ?

Yes Er det en grunn til at det står akkurat olfactory nerve?--> Alpha-synuclein is highly expressed in the mitochondria in olfactory bulb, hippocampus, striatum and thalamus

In addiction treatment opiates are connected with lacrimation?

Yes How?RW- it's part of withdrawal symptoms- excessive lacrimation - Early Signs (8-12 hrs): Diaphoresis, nausea, yawning, lacrimation, tremor, rhinorrhea, irritability, dilated pupils, resp. rate, pulse>90

Benzalkonium has spermicidic effect as well as fungicidic, bactericidic and viricidic?

Yes Ida E: I think this should be wrong, benzalkonium is not effective against virus and bacterial spores RW: just sperms and bacteria

Reflexes in neonates can be "pathological" in one year

Yes If they mean that the reflexes are patholgical if they are present beyond 1 year of age it should probably be correct

Can a lesion of middle cerebral artery lead to hemiparesis without sensoric deficit?

Yes Marte: No (RW agree) Middle cerebral artery syndrome - Contralateral paralysis and sensory loss of the face and arm (anterior cerebral artery supplies the lower extremity homunculus) - Contralateral homonymous hemianopia - Deviated conjugated gaze toward the side of lesion (frontal eye field = area 8) NB! If there is only irritation, not destruction, the eyes moves to contralateral side - Dominant hemisphere - aphasia (usually left premotor area) - Non-dominant hemisphere - neglect syndrome

Spinal muscular atrophy is linked to SMN gene?

Yes Mutations in the telomeric copy of this gene are associated with spinal muscular atrophy; mutations in the centromeric copy do not lead to disease.

Pegaptanib is used for makular degeneration treatment ?

Yes Other drugs are - Ranibizumab - Atlibercept (VEGF)

Alien hand syndrome can be seen in corticobasillary degeneration?

Yes RW: A patient with alien hand syndrome (AHS) experiences their limbs act without the person having control over the action. ... Other areas of the brain that are associated with alien hand syndrome are the frontal, occipital, and parietal lobes.

Treatment of neuropathic pain is done with pregabalin or venlafaxin?

Yes RW: pregabalin: structural analogue of GABA. Function: Pregabalin and Gabapentine block Ca2+ influx decreasing Ca2+ intracellulary. NO release of transmitter, no GABA receptor affected. treatment for Neuropathic pain, epilepsy, gad venlafaxin: snri - antidepressant neuropathic pain

Selegilin can be used an antidepressant ?

Yes Selective, irreversible MAO B inhibitor - inhibits DA degradation Parkinsons disease and dermal patch in depression

Deja vu is due to lesions of temporal lobe?

Yes déjà vu has been linked to seizure activity in temporal lobe epilepsy, and clinical reports suggest that many patients experience the phenomenon as a manifestation of simple partial seizures.

MTT (mean transit time) - is a measure of cerebral perfusion?

Yes the ratio of cerebral blood volume to cerebral blood flow (CBV/CBF), is a valuable indicator of the cerebral circulation

Alpha-synuclein inclusions are found in oligodendrocytes in multiple system atrophy (MSA) ?

Yes "Papp Lantos inclusions"

Do we typically have ketoacidotic coma in DM1 ?

Yes (In DM2 it is more uncommon)

Ayahuasca (DMT) need harmalin for activation?

Yes - Ayahuasca contain: DMT, Harmine and Harmaline - DMT is generally not active orally unless it is combined with a monoamine oxidase inhibitor (MAOI) such as a reversible inhibitor of monoamine oxidase A (RIMA), for example, harmaline. Without an MAOI, the body quickly metabolizes orally administered DMT, and it therefore has no hallucinogenic effect unless the dose exceeds monoamine oxidase's metabolic capacity. - Hallucinogens agonists, 5-HT2 - Effective dose 1-20 mg, the duration of action of 20 minutes to 1 hour - DMT and 5-MEO-DMT administered by smoking, orally effective (only with MAOIs)

- Contraindications of tryptans - hypertension untreated and variant disease - Tryptans should be given during aura

Yes No (should be given after the headache starts)

Nucleus basalis Meynerti is cholinergic?

Yes RW: NB -ALZ - Generalized cerebral atrophy; neurofibrillary tangles, neuritic (senile) plaques, granulovacuolar degeneration, Hirano bodies; decreased number of neurons in the nucleus basalis of Meynert

Schizophrenia has onset in third decade?

Yes 15-30 years

Anxiety is the most prevalent mental disorder?

Yes >15% of population have clinically significant anxiety

Anxiety can be difficult to exclude from affective disorder, since they often overlap?

Yes >60% of depressed patients also have anxiety

Huntington - Can it cause dementia? - Inheritance? - Typical clinical sign?

Yes AD Chorea

Tysabri (monocloncal Ab) can be used in MS?

Yes Another name for Natalizumab Treatment of relapsing form of MS

A lesion of facial nucleus in brain stem produces peripheral facial palsy?

Yes Before nuclei --> Central From nuclei and peripherally --> Peripheral

Atypical parkinsons syndrome can present in MSA patient? (Multiple system atrophy )

Yes But, with no response to levodopa therapy Impairment of autonomic functions, ataxia and parkinsonism, disease develops in adulthood. Atrophy of striatum and pons

Negri bodies are connected with Rabies?

Yes Eosinophilic inclusion bodies found in cytoplasm of nerve cells containing rabies virus Typically in Ammons horn in hippocampus

Astrocytomas are connected with some glial astrocytal protein?

Yes Glial fibrillary acidic protein (GFAP) can be used for staining astrocytomas, even the anaplastic variants stain positive.. Unsure if this is what they are asking for though.

Lesion of nervus peroneus causes stepping, walking like a rooster ?

Yes It causes the inability to lift foot at the ankle, causes foot drop walking The roosters kind of "drag their feet" after them haha. RW in class notes says "walking like a stork" so..a bird. * cerebellum: unsteady posture, broad-based gait, unsteadiness when turning around, in tandem walking, independent of eye control * sensation (posterior column syndrome): increased swaying when eyes are closed (positive Romberg test) * muscle disease: difficulty with deep knee bend, waddling gait * basal ganglia: abnormal postures and movements (e.g. parkinsonism)

Lewy bodies contain alpha-synuclein and they can be seen in degenerative disorders

Yes Lewy bodies are found in parkinson's disease, lewy body dementia and some others

Spinal muscular atrophy is related to SMA gene?

Yes NO-smn 1 gene-nikolas

Metastasis are the most common intracranial tumors

Yes Nikolas-No way(only 15-20% are metastatic)

In stimulation therapy of depression you only stimulate a part of cingulum ?

Yes No ???? Psych at a glance: Deep brain stimulation of cingulate cortex or nucleus accumbens has anti-depressive effects.

Zolpidem is a hypnotic drug?

Yes Used for insomnia and some brain disorders. it is a short acting NON-BZD that potentiates GABA function works within 15 min, half life 2-3h

Lesion of radix anterior leads to hypotony?

Yes Ventral root - spinal nerve

Frontal cortex lesions causes problems with eyes?

Yes You are unable to look away from the affected side, and the eyes are deviated towards the lesion (gaze disorder) horizontal gaze: brainstem and cortical (Frontal eye field - frontal lobe)

For depression more inflammatory marker?

Yes http://www.medscape.com/viewarticle/584794

- Use of TCA can be accompanied by cognitive disorder - The specific antidote for TCA is flumazenil

Yes no (it's for BZD)

triptans - contraindicated in HTN ? - Contraindicated in renal and liver failure? - Can be used during pregnancy?

Yes yes no

Conus lesion will lead to sensation disturbance and pain in the saddle area?

Yes • Sensory disturbance - saddle anesthesia over the "saddle area" • Pain over the "saddle area" • Sphincter paralysis: • Disturbance of urination (denervated autonomous bladder - retention) and fecal incontinence • Sexual dysfunction - impotence

Subjective tinnitus linked with hallucinations and delusions about its origin?

Yes (?) Subjective tinnitus is tinnitus only you can hear. This is the most common type of tinnitus. It can be caused by ear problems in your outer, middle or inner ear. It also can be caused by problems with the hearing (auditory) nerves or the part of your brain that interprets nerve signals as sound (auditory pathways) Objective tinnitus is a tinnitus that the doctor can hear as well.

Is there fibrinoid necrosis in small arteries in brain when someone suffers from hypertension

Yes (NB: must be malignant HTN) --> so not in normal HTN?

Catathymia is a change of motion linked to mood

Yes (RW - NO! this is cataplexy - agree with elise) Elise: No I think this is the defintion of cataplexy.. Any good definition of catathymia ? Mental process is under the influence of emotions. RW: is this change of "emotion" not motion? catathymic crisis an unexpected explosive outburst of impulsive often destructive behavior understandable only in terms of unconscious motivation..... Such a fixed idea, along with a mounting pressure to act, is characteristic of the (chronic) catathymic process, in which a subject, without apparent motive, resorts to extreme violence directed at someone close to him.

In alternans hemiplegia there is deviation of the palate and lowered shoulder?

Yes (RW NO!) It only says "inferior" on the old test.. So I guess the statement would be correct if it said Inferior hemiplegia alternans, because the lower cranial nerves would be affected Ida: think only hypoglossus is affected? - No! All the cranial nerves in medulla are affected, but typically we see clinical signs from hypoglossus, because it's easy to investigate! RW- if inferior - it's 12th nerve n only tongue (scapula is accesory) From wiki scriptAlternating hemiplegia is a type of central paralysis , caused by damage to the brain stem . It manifests with contralateral hemiplegia (caused by a disorder of the corticospinal tract before crossing) and ipsilateral renal any of the cranial nerves. Individual hemiplegia divided according to the site of injury. alternans hemiplegia inferior (dejerine syndrome): Caused by lesions of the medulla oblongata. Manifested paresis n. Hypoglossal (XII N..) , Which is ipsilateral. Symptoms of the movement disorder of language, atrophy and fibrillation language. Further, the contralateral hemiplegia with impaired sensitivity.

In MS you can find oligodendrocytes in the inactive plaques?

Yes (RW-Agree) In the inactive plaques you can see gliosis and minimal to no inflammation. *Gliosis*: Proliferation of glial cells (astrocytes, oligodendrocytes, microglia) Ida E: think it´s wrong, cause we find astrocytes, not oligodendrocytes as far as I know. RW: http://neuropathology-web.org/chapter6/chapter6aMs.html

Is meningitis in children < 1 year most commonly caused by gram negative diplococci?

Yes (RW: in another test - it's marked no - i think they want GBS) MA: But then its marked wrong because it says *newborns*. Here this is right because they are referring to neisseria - Immediately after birth, group B Streptococcus (G+), Escherichia coli (G-), and Listeria (G+) species are the most common. - After approximately 1 month of age, Streptococcus pneumoniae (G+), Haemophilus influenzae type B (G-) (Hib), and Neisseria meningitides (G-) are more frequent. - G-negative diplococci: neiserria meningitides, haemophilus influenza type B (OBS! We are vaccinated against this one)

Sensitive ataxia is due to a lesion in the back of medulla?

Yes (RW: wrong if its the sensory ataxia - then it is peripheran nerves, if they are talking about spinal column it is spinal ataxia) Can also be a lesion in cerebellum, thalamus and parietal lobes Look at this link and see where the sensory tracts go (posterior) https://www.google.cz/search?q=tracts+in+medulla&client=firefox-b&tbm=isch&imgil=kIAi6s5tIbhcvM%253A%253BYQmeAiEHkhdNGM%253Bhttps%25253A%25252F%25252Fwww.pinterest.com%25252Fpin%25252F177258935303607213%25252F&source=iu&pf=m&fir=kIAi6s5tIbhcvM%253A%252CYQmeAiEHkhdNGM%252C_&usg=__S-V8QyUzxD2axWJTJmcGg7S-H9o%3D&biw=1366&bih=635&ved=0ahUKEwjV46DrhY7NAhXC1hQKHdLpA34QyjcIKQ&ei=m51SV5WqJ8KtU9LTj_AH#imgrc=yE_Tag60wODUcM%3A

Post-encephalomalatic pseudocyst is formed after ischemia of the brain, when the liquefactive necrosis develop into a cyst?

Yes (according to pathophys)

Cranial nerve XI (accessory) can lead to dysarthria?

Yes (also dysphonia and dysphagia) (RW- disagree - accessory lesion SCM and Traps) Eva: CN XI palsy er ikke assosiert med dysarthria, dysphonia og dysphagia...? Marte: enig med Eva --> the shoulder is dropped and the patient cannot turn the head to the healthy side Synni e enig med både Eva og Marte! Dette må isåfall være snakk om XII?? Ida: steike beiken (Synne: hahahaahah) Lesion in the region is usually accompanied by dysarthria, but a lesion isolated in cranial nerve XI should not cause it.

When liver is failing does it induce making of pathological mediators in brain?

Yes (ammonia)

Tuberculous spondylitis is the same as Pott's disease, is most commonly located in the thoracic part of vertebral column?

Yes (and in lumbal) *NOT* cervical as they try and trick us with.

Bulbospinal atrophy is a triplet repeat disease?

Yes (same as *Kennedy's disease*) The genetic basis of the disease involves an expanded *repeat of the CAG trinucleotide* in the proximal portion of the q arm of the X chromosome. Characterized by degeneration of both motor and sensory neurons. It involves loss of lower motor neurons supplying the limb and bulbar musculature. Extraocular muscles are spared, possibly because of reduced numbers of androgen receptors in these muscles RW: Trinucleotide repeat disorders * Huntington´s Disease * Spinocerebellar Ataxias * Friedreich´s Ataxia * Kennedy´s Disease (SBMA) * Dentorubral-pallidoluysian Atrofi

Do you loos consciousness in an "absence" epileptic seizure?

Yes (usually) The person has a brief loss of consciousness (an absence) for a few seconds. They do not fall, but may pause in what they are doing. Their face often looks pale with a blank expression. They may look dazed, the eyes stare, and the eyelids may flutter a little. Person is unaware of this absence

Neuropathic pain can be treated by both SNRI DNRI ?? :)

Yes =) SNRI - venlafaxin DNRI - bupropion

Dysbindin and Neuregulin are forming synapses?

Yes ??? What is this?! Dysbindin is a schizophrenia susceptibility gene required for the development of dendritic spines. RW - YES: in ppt it says these two are genes for '"synaptic functions" *Dysbindin*: strong association was found between expression of a particular dysbindin allele and a clinical expression of schizophrenia Dysbindin, (dystrobrevin-binding protein 1) is a protein constituent of the dystrophin-associated protein complex (DPC) of skeletal muscle cells.( ...). dysbindin is found in neural tissue of the brain, particularly in axon bundles and especially in certain axon terminals, notably mossy fiber synaptic terminals in the cerebellum and hippocampus *Neuregulin*: family of four structurally related proteins that are part of the EGF family of proteins( ....)Schwann cell and oligodendrocyte differentiation, some aspects of neuronal development, as well as the formation of neuromuscular synapses. - related/support to glutamate hypothesis of schizo

In lesions of cauda of spinal cord always includes problems with the sphincters?

Yes and no is found on tests.. I hate the word "always". I would think so.. It says on pp from school: Cauda equina lesion: sphincter disorders (acute retention of urine and constipation) RR: YES

Ethosuximid is used for cramps in children?

Yes as long it doesn't say children under 3 years old. Ethosuximide is used alone or in combination with other medications to *treat absence seizures* (also called "petit mal" seizures) in *adults and children*. Ethosuximide should not be given to a child younger than 3 years old. Klara: Should be wrong? if its used for absence seizures, it shouldnt be used for cramps? Why can't you use it in both?

Meniers disease is with increase pressure in the ear? It is hydrops?

Yes! Meniers disease probably develops due to fluid buildup in inner ear, which causes increased pressure. (vertigo, pressure, hearing loss, tinnitus) Ménière disease is a disorder of the inner ear that is also known as idiopathic endolymphatic hydrops. Endolymphatic hydrops refers to a condition of increased hydraulic pressure within the inner ear endolymphatic system

Anti-paracetamole is acetylcysteine

Yes! Acetylcysteine, also known as N-acetylcysteine or N-acetyl-L-cysteine (NAC), is a medication used to treat paracetamol (acetaminophen)

Schizotypal disease belongs to psychosis?

Yes, according to ICD-10 it is a psychotic disorder! (under personality disorders) NB: can have illusions, but not hallucinations

Amantadine was made as antivirotic drug in the first place?

Yes, also antiparkinson drug

Buspiron is anxiolytic?

Yes, but can also be used as antidepressant Serotonin agonist

Can surgery be used in epilepsy?

Yes, but pharmacotherapy is first line therapy

Is MDMA neurotoxic?

Yes, for serotonergic nerve terminals

Alteplase is a fibrinolytic drug?

Yes, it is a tPA drug (Tissue plasminogen activator) must be administered within 3h after start of stroke! Obs, be sure it's not hemorrhagic

Central pontine demyelination is related to alcoholism?

Yes, leads to locked-in-syndrome Locked-in?! Really? =O

Anosmia or dysosmia in one nostrill can be due to a tumor in the frontal lobe

Yes, meningioma in olfactory groove typically causes unilateral anosmia -Anosmia is the inability to perceive odor or a lack of functioning olfaction—the loss of the sense of smell.

Guillan Barre is an autoimmune disease?

Yes, onset is from CMV

Devic's disease has Ab against aquaporin 4 ?

Yes, present on astrocytic processes

Rexed zones divide gray part of the medulla spinalis?

Yes, starts at the end of medulla/the beginning of spinal cord (rexed zones - rexed laminae)

TCA have anticholinergic effect and are cardiotoxic?

Yes. Act as potent antihistamines and *anticholinergics* Act as sodium channel blockers and calcium channel blockers, respectively. The former property is responsible for the high mortality rate upon overdose seen with the TCAs via *cardiotoxicity*.

Lesion of epiconus L5-S2 is the same as L5-S1 ?

Yes? I would say no - but they do resemble each other. RW-i would say they are not the same Epiconus lesion: L5-S2 Often resembles the radicular lesion of L5-S1 But in the radicular lesions the weakness of extensors predominates - peroneal muscles, gluteal muscles, flexors of the crus --> so you mean no then? S1 affects the hips and groin area S2 affects the back of the thighs

Choose the right statement a) Brain edema may develop as a consequence of hyponatremia b) Swelling of the brain develops as a result of plasma hypoosmolality c) For patient with acute intracranial hypertension, vasodilation can reduce intracranial pressure d) Hyperglycemic coma can cause brain swelling

a) Brain edema may develop as a consequence of hyponatremia b) Swelling of the brain develops as a result of plasma hypoosmolality TRUE (C IS OPPOSITE!)

Carpal tunnel syndrome: a) Caused by overloading of upper extremity b) Caused by lesion of radial nerve c) Typical finding in glass cutters d) Usually manifests during walking

a) Caused by overloading of upper extremity c) Typical finding in glass cutters Medial nerve?

Choose the correct a) In inflammatory lymphocytic infiltrate dermatomyositis CD4+ T lymphocytes are predominant b) Myotonic dystrophy is an autosomal recessive disease c) Lambert-Eaton syndrome is a paraneoplastic neuropathy d) Duchene muscular dystrophy is an AD disease

a) In inflammatory lymphocytic infiltrate dermatomyositis CD4+ T lymphocytes are predominant TRUE c) Lambert-Eaton syndrome is a paraneoplastic neuropathy TRUE Myotonic dystrophy - AD inheritance Muscular dystrophy - Xlinked Gesine: Robbins S.830 - LES characteristically develops as a paraneoplastic process...most commonly in setting of small-cell lung ca (60%) and it is a neuropathy due to the fact that AB´s attack presynaptic Ca-channels--> C is TRUE

choose the right statement a) Memantin is inhibitory of glutamate NMDA receptors b) To behavioral disorders in demented patients (such as agresivity, psychotic symptoms) risperidone is appropriate therapy c) Peripheral myorelaxation can positively influence the symptoms of Huntington disease - d) Psychosocial stress may be the reason of progression of Alzheimer`s dementia -

a) Memantin is inhibitory of glutamate NMDA receptors b) To behavioral disorders in demented patients (such as agresivity, psychotic symptoms) risperidone is appropriate therapy- TRUE d) Psychosocial stress may be the reason of progression of Alzheimer`s dementia - TRUE on SASHA's test last semester! (c-it's central, usually, i think - RW)

2. Which of the following are true? a) Nimodipin blocks Ca channels preferentially in the CNS b) Baclofen acts as an antagonist on presynaptic GABA-B receptors c) Tolperison is used in spastic skeletal muscle after a stroke, encephalitis , in multiple sclerosis d) Nimodipin is a betablocker

a) Nimodipin blocks Ca channels preferentially in the CNS - TRUE (Given after stroke to reduce the damage since Ca is good for cells) c) Tolperison is used in spastic skeletal muscle after a stroke, encephalitis , in multiple sclerosis - TRUE RW (baclofen is an agonist!) (nimodipin -ca channel blocker for sub arachnoid hemorrage vasospasm)

Which of the following applies best a) Pain and sensory disturbances are in cauda equine syndrome (perianal and perigenital) b) In posterior column syndrome has patient has impaired balance when standing and walking with eyes closed c) Dissociated syringomyelia syndrome may be due to vertebral metastases d) Paresthesia occurs without external stimulus, in dysaesthesia patient perceives the stimulus differently

a) Pain and sensory disturbances are in cauda equine syndrome (perianal and perigenital) - TRUE B - False --> Should be correct? D - False --> Should be correct? Paresthesia: „a pins and needles sensation ", „a burning feeling "- spontaneous (no stimuli) Dystesthesia is a qualitative change of sensation. Clothing can feel painful on the skin A,B,D definitely true, C (syringomyelia syndrome = loss of pain and temperature sensation) can be due to tumors but not sure if particularly vertebral metastases. TH

What is Brown Sequard syndrome?

a) Pain and temp loss - contralateral - below the lesion - spinothalamic tract lesion b) Position and vibration - ipsilateral- below lesion-posterior column lesion c) UMN paralysis - ipsilateral - below the lesion - corticospinal lesion

which of the following choices apply a) Paralysis, atony, areflexia and progressive muscular atrophy are the signs of destruction of peripheral motor nerve fibers b) Myasthenia gravis is characterized by increased muscle fatigue during the day c) When damaged anterior spinal horn arises paresis of innervated muscles d) In anterior acute poliomyelitis is typically affected both upper and lower motor neurons

a) Paralysis, atony, areflexia and progressive muscular atrophy are the signs of destruction of peripheral motor nerve fibers TRUE b) Mysthenia is characterized by increased muscle fatigue during the dayTRUE c) When damaged anterior spinal horn, arises paresis of innervated muscles TRUE

2. Myasthenia gravis may be present by: a) Ptosis (or diplopia) b) paraparesis or hemiparesis with hyperreflexia c) sphincter disturbances d) lagophtalmus

a) Ptosis (or diplopia) The most frequent m. symptoms occur in the eye. Then neck, face etc. depending on which m. group the AB attach

Choose the right statements a) Remission of Schizophrenia is periods with no symptoms b) Positive symptoms of schizophrenia include blunted affect, depressed mood, and suicidal thoughts c) Bleuler`s 4 A`s denotes : association, autism, blunted affect and ambivalence d) Schizophrenia starts typically at the age of 30-40 years

a) Remission of Schizophrenia is periods with no symptoms TRUE c) Bleuler`s 4 A`s denotes : association, autism, blunted affect and ambivalence - TRUE

From the following choose the right statements a) The cerebellar symptoms include scanned speech, nystagmus and macrographia b) Asynergy is a manifestation of neocerebellum and is manifested as rebound phenomenon c) During the bigger back tilt of the standing patient with paleocerebellar syndrome doesn't bend his legs and falls d) Lesion of cerebellum can manifest as rest or static tremor

a) The cerebellar symptoms include scanned speech, nystagmus and macrographia TRUE c - was marked as false.. But according to the NEURO pp about cerebellum: "Dyssynergia"- lesion of paleocerebellum --> leads to retroflexion of the trunk, and wihtout flexion of the knees the patient falls backward Cerebellum --> intention tremor! Rebound phenomenon is typical is dysmetria

a) Typical morphological substrate in MSA is neuronal cytoplasmic deposit of alpha synuclein b) the protein that is pathologically conformationally changed in Huntingtons is huntingtin c) typical morphological substrate in parkinsons is lewy body d) oligodendroglial inclusions are found characteristically in MSA and corticobasal degeneration

a) Typical morphological substrate in MSA is neuronal cytoplasmic deposit of alpha synuclein (marked as FALSE on the newest version of the test. TH) b) the protein that is pathologically conformationally changed in Huntingtons is huntingtin (true) c) typical morphological substrate in parkinsons is lewy body (true)

a) arrhythmia is one of the side effects of TCA b) SSRIs can cause serotonin syndrome in combination with MAO inhibitors c) fluoxetine is an TCA d) amytriptiline is an TCA

a) arrhythmia is one of the side effects of TCA b) SSRIs can cause serotonin syndrome in combination with MAO inhibitors d) amytriptiline is an TCA

Narcolepsy : a) attacks of imperfect sleep b) sleep paralysis c) sleepiness during whole day d) sleep apnoea syndrome e) attacks of cataplexy

a) attacks of imperfect sleep b) sleep paralysis d) sleep apnoea syndrome It should be: b,c,e (a?) Synne agrees with these (also a)! (RW: everything should be correct except sleep apneo) "The classic symptoms of the disorder, often referred to as the "tetrad of narcolepsy," are cataplexy, sleep paralysis, hypnagogic hallucinations, and excessive daytime sleepiness."

5. Parkinsonian syndrome manifests by : a) bradykinesia b) intention tremor c) rigidity d) paradoxical kinesia e) postural hypotension f) mask-like face

a) bradykinesia c) rigidity e) postural hypotension f) mask-like face Tremor er jo typisk, men ikke intention? --> Jepp, det er resting tremor!

9. Early signs of myasthenia gravis : a) diplopia b) dysphagia c) dysphasia d) dyspnoe

a) diplopia b) dysphagia (dysphonia is very rare) (dyspnoe is one of the later signs - affection of trunk muscles comes late)

12. Chronic metallic mercury poisoning : a) follows inhalation of mercury vapours b) follows ingestion of mercury metals c) manifests by erethism (irritability, restlessness) and intention tremor d) manifests by abdominal colic pain e) causes peripheral neuropathy with wrist drop

a) follows inhalation of mercury vapours b) follows ingestion of mercury metals c) manifests by erethism (irritability, restlessness) and intention tremor e) causes peripheral neuropathy with wrist drop

3. Epileptic seizure called grand mal is : a) generalised tonic-clonic seizure b) seizure in which epileptic activity is bilateral c) has focal origin d) can be connected with cyanosis

a) generalised tonic-clonic seizure b) seizure in which epileptic activity is bilateral - Affecting whole brain (both hemispheres) - Tonic-clonic = grand mal (earlier)

a) in meningioma we commonly find cytoplasm positivity of progesterone receptors b) glioblastoma multiforme doesnt belong to glial cell tumors c) GFAB is a diagnostic marker for oligodendroglial tumors d) neuroblastoma doesnt belong to glial cell tumors

a) in meningioma we commonly find cytoplasm positivity of progesterone receptors d) neuroblastoma doesnt belong to glial cell tumors

Right statement a) lesions in anterior root of spinal cord leads mostly to small scale paresis and without sensory disturbances b) in a central paraparesis legs with normal movement of the arms may arise from the upper cervical spinal cord lesions of cervical intumescence c) In acute spinal cord lesion is always present from the beginning significant distinctive spasticity

a) lesions in anterior root of spinal cord leads mostly to small scale paresis and without sensory disturbances

a) neologisms is phrases or words made up by the patients or use of conventional phrase independent of their normal meaning b) enlargement of the lateral ventricles is a morphological finding that occurs only in schizophrenia c) loosening of associations can be a sign of schizophrenia d) dopaminergic sensitization in mesocortical pathway is the primary theory explaining the pathophys of paranoid delusions in schizophrenia

a) neologisms is phrases or words made up by the patients or use of conventional phrase independent of their normal meaning c) loosening of associations can be a sign of schizophrenia d) dopaminergic sensitization in mesocortical pathway is the primary theory explaining the pathophys of paranoid delusions in schizophrenia I think D is wrong! Mesocortical is known for *negative* symptoms, mesolimbic for *positive* ! Synne agrees =)

6. Secondary Parkinsonism may be caused by : a) severe brain injury b) cerebrovascular disease c) epileptic seizures d) severe carbon monoxide poisoning

a) severe brain injury b) cerebrovascular disease d) severe carbon monoxide poisoning http://www.merckmanuals.com/professional/neurologic-disorders/movement-and-cerebellar-disorders/secondary-and-atypical-parkinsonism

2. Lesions of post. columns leads to : a) spinal ataxia b) pain & temp sensation are impaired c) deep tendon reflexes are absent d) spastic paresis occurs

a) spinal ataxia c) deep tendon reflexes are absent What is spinal ataxia ??? Same as sensory ataxia and it is due to a lesion in the spinal cord (a cerebellar ataxia is caused by a lesion of the cerebellum, vestibular ataxia = dysfunction of vestibular system) *Sensory ataxia* is both a symptom and a sign in neurology. It is a form of ataxia (loss of coordination) caused not by cerebellar dysfunction but by loss of sensory input into the control of movement dorsal column: gracile fasciculus and the cuneate fasciculus and itself is part of the posterior funiculus. It is part of an ascending pathway that is important for well-localized fine touch and conscious proprioception called the posterior column-medial lemniscus pathway.

Which of the following is true a) thalamus is a part of diencephalon b) the pyramidal tract has 2 neurons c) the mesencephalic nucleus of the 5th nerve has pseudounipolar cells for proprioception d) the indentation of the spinal cord is deeper on the dorsal side

a) thalamus is a part of diencephalon b) the pyramidal tract has 2 neurons c) the mesencephalic nucleus of the 5th nerve has pseudounipolar cells for proprioception

a) tinitus is an accompanying symptom of menieres disease b) temporomandibular joint dysfunction cannot be the cause of tinnitus c) tinitus can be caused by drugs d) tinitometry is measuring of tinitus via logarithmic rule

a) tinitus is an accompanying symptom of menieres disease c) tinitus can be caused by drugs

a) a typical finding in the cerebrospinal fluid during basilar meningitis is lowered Na+ concentration b) acute purulent meningoencephalitis can occur from spreading from infection from sinuses c) causative agent of rabies belongs among primary neurotropic viruses d) herpetic meningoencephalitis is characterized by the presence of intracytoplasmic inclusion in glial cells

a,b,c D - false, because it says intracytoplasmic!!! Pathology powerpoint: Herpes simplex - encephalitis with hemorrhage and necrosis, intranuclear inclusions in neurons and glial cells

a) 3rd ventricle is unpaired b) gamma motorneurons are in ant. spinal horns c) cortical auditory area is in the frontal lobe d) ncl. basalis meynerti is cholinergic

a,b,d

106. Zolpidem:

a. 3rd generation hypnotic. b. POSTIVE ☺ : i. Zolpidem(and zaleplone) do NOT affect the respiratory syndrome so they can be used even if there is APNOEA!! ii. They(zalelpone as well) do NOT affect memory!! c. There is still chance of addiction. d. Short-acting drugs! e. As safe as BZPs, they have a HIGH therapeutic index! THINK: From BZPs, zolpidem/zaleplone and Chlorpromazine only the 2 latter are useful in AD, since BZPs affect memory!

65. N-acetylcystein:

a. Acetylcysteine or methionine can substitute for cysteine, increasing GSH availability and reducing mortality in patients with paracetamol poisoning.

77. Pilocarpine:

a. Ach receptor agonist b. Treatment of glaucoma and xerostomia.

85. Rivastigmine:

a. AchE inhibitor. b. R. DRG initials for treatment of mild-moderate AD.

Inducers of enzymes in the liver.CYP.

a. Alcohol, rifampicin, phenytoinCYP3A4, phenobarbital. b. Pomelo, grapefruit juice, some oranges, mojito-lime. c. Clarithromycin. d. SSRIs-CYP2D6. e. Amiodarone.

101. Valproate:

a. Anticonvulsant.

What is Topiramate?

a. Anticonvulsant.

103. Vigabatrin:

a. Anticonvulsant. b. Irreversible inhibitor of GABA transaminase!(required for breakdown of GABA)

107. Cannabidiol:

a. Antispastic! b. THC is a psychotropic, so difficult to isolate and prevent SE!

Which of the following are true? a. Atrophy of the tongue is found in bulbar syndrome (bulbar palsy) b. When affected the optic nerve behind the eye bulb (before the chiasma) , the patient will have homonymous hemianopsia on the same side c. In peripheral paresis of n.VII , the patient has weakness on half of the face and lagophthalmos d. Pseudobulbar syndrome is caused by impairment of cranial nerve nuclei

a. Atrophy of the tongue is found in bulbar syndrome (bulbar palsy) c. In peripheral paresis of n.VII , the patient has weakness on half of the face and lagophthalmos (pseudobulabar syndrome caused by bilateral lesion of corticobulbar tract - page 49)

94. Tiaprid:

a. Atypical antipsychotic. b. NOT for long-term treatment!! c. It is used only in the beginning of treatment until the effects of the long term treatment take place!!!

What is Risperidone?

a. Atypical antipsychotic. b. SDA= Serotonin Dopamine antagonist.

93. Tetrazepam:

a. BZP!! b. GABA-A agonist!

70. Nimodipin:

a. Ca2+ blocker, praticularly acting in the CNS. b. It has neuroprotective effects and acts against vessel spasms. c. For treatment of intracerebral hemorrhage!!

98. Tolperison:

a. Central muscle relaxant.

61. Methylprednisolone:

a. Corticosteroid for acute attacks of MS. b. Given i.v

57. Lithium:

a. Dose must be adjusted according to plasma concentration. It has a very narrow therapeutic window! b. In rapid cycling bipolar disorder the pts are usually unresponsive or with decreased response to lithium.

Suspicion of myestenia gravis can be confirmed with: a. EMG examination b. muscle biopsy c. cheast CT d. CSF examination e. ACHe-inhibitors f. LAB

a. EMG examination c. cheast CT (to look for thymoma, hyperplasias) e. ACHe-inhibitors (to look for improvement after administration) f. LAB (Anti Ach receptor antibodies)

68. Natalizumab:

a. Humanized monoclonal anitbody. b. Neutralizes VLA-4{very late antigen} on lymphocytes. c. Given subcutaneoulsy or by i.v. once a month, not orally since it is protein! d. Given for active form of MS (used in severe MS) and only after interferons and immunoglobulins have failed!

97. Tolcapone:

a. INHIBITOR OF COMT. b. It has central effects(cf to entacapone which only has peripheral). c. Adjuvant combination with levodopa in LATER phases of PD. d. Allows more levodopa to pass through the BBB as it inhibits COMT in blood and peripheral tissues.(cf with MAOI which slow down breakdown of levodopa in BRAIN)

82. Pyritinol:

a. In the same group as piracetam=ampakine b. Increases expression of BDNF!!(ampakine, check Rang and Dales) c. Pyritinol, piracetam and the brain vasodilators: dihydroergotamine and flunarizine, are all NOOTROPICS(ginkgo biloba is as well)! i. Piracetam is used only after brain damage!! ii. In general nootropics are not used for dementia per se but post stroke! d. NOOTROPICS increase the functioning of the neuronal cells.

What is not a clinical manifestation of epilepsy?

a. headache ? People with epilepsy have a higher tendency to develop headaches compared to others, but I'm not sure if it is a clinical manifestation.. think not..

90. Sorafenib:

a. Inhibits tyrosine kinases...?!

80. Primidone:

a. It acts by being metabolised to phenobarbital. b. Na+ channel blocker.

91. Streptokinase:

a. Its action is blocked by antibodies, which appear 4 days or more after the initial dose: its use should not be repeated after this time has elapsed.

58. Memantine:

a. Non competitive NMDA receptor antagonist. b. Used for moderate to severe AD! i. Combination of memantine and anticholinesterases, or the isolated use of memantine if antichoinesterases are not working anymore!

66. Naloxone:

a. Opioid antagonist. b. It has a shorter biological half-life than most opioid agonists. c. In the treatment of opioid overdose, it must be given repeatedly to avoid the respiratory depressant effect of the agonist reoccurring once the naloxone has been eliminated. d. Naltrexone has a longer biological half-life.(given intravenously)

78. Povidone:

a. Povidone iodine_Betadine.

74. Paracetamol: (acetaminophen)

a. R. Paracetamol and metoclopramide are better and cheaper than sumatriptan!(in test) b. Paracetamol is metabolized by the liver-conjugation 95% and the remaining 5% is mtb by CYP2E1 to NAPQI!!!!!. NAPQI is hepatotoxic and alcohol increases CYP2E1!! c. Lethal dose: >7,5g d. DEFINTITE DEATH if >13,5 g.

102. Venlafaxin:

a. SNRI

89. Sertralin:

a. SSRI. b. Check citalopram for more!!

76. Phenobarbital:

a. The clinical uses of phenobarbital are virtually the same as those of phenytoin, although phenytoin is preferred because of the absence of sedation. b. Na+ channel blocker and Ca2+ channel blocker.

73. Combination of naloxone and oxycodone: (internet..)

a. To relieve moderate to severe persistent pain when other forms of treatment have not been effective and naloxone will help prevent and treat opioid-induced constipation.

75. Pegaptanib:

a. VEGF inhibitor. b. Macular degeneration treatment.

83. Ranibizumab:

a. VEGF inhibitor. b. Macular degeneration treatment. c. NOTE: >pegaptanib. Ranibizumab and pegaptanib are given by intraviteal injection so most common SideEffects are associated with the route of administration and include: i. Increase in intraocular pressure within 1 hour of injection. ii. Endopthalmitis. iii. Intraocular inflammation. iv. Retinal detachment, tear v. Iatrogenic traumatic cataract.

105. Ziconotide:

a. a synthetic analogue of the N-type calcium-channel blocking peptide ω-conotoxin MVIIA. b. is effective when administered by the intrathecal route. c. It is used in patients whose pain does not respond to other analgesic agents

104. Warfarin:

a. a vitamin K antagonist! b. Oral anticoagulant. c. Absorbed rapidly and completely from gut. d. Used for prolonged treatment! R. The effect of warfarin takes several days to develop because of the time taken for degradation of preformed carboxylated clotting factors. e. Frequent blood tests to individualise dose are required, as they have a low margin of safety. f. R. INR!!

What is true about endometritis? a. acute endometritis could be a complication of delivery and abortus b. acute endometritis is usually of purulent type c. is never caused by chlamydia d. the presence of plasmacytes besides lymphocytes is necessary for the diagnosis of chronic endometritis

a. acute endometritis could be a complication of delivery and abortus d. the presence of plasmacytes besides lymphocytes is necessary for the diagnosis of chronic endometritis

Abnormal uterine bleeding might be caused by? a. anovulatory cycles b. endometrial hyperplasia c. some ovarian tumors d. tubal pregnancy

a. anovulatory cycles b. endometrial hyperplasia c. some ovarian tumors d. tubal pregnancy

95. Timolol:

a. beta-blocker. b. glaucoma treatment, eye drops.

Carpal tunnel syndrome a. can be caused by overloading of the upper extremities b. is caused by the lesion of the radial nerve c. is a typical finding in the glass cutters (grinders) d. it usually manifests during walking e. can be associated with the history of exposure to extreme hand-arm vibration f. its degree can be assessed by electromyography

a. can be caused by overloading of the upper extremities c. is a typical finding in the glass cutters (grinders) e. can be associated with the history of exposure to extreme hand-arm vibration f. its degree can be assessed by electromyography

The most frequent causes of an intracranial hemorrhage are according to the incidence rate a. hypertensive diseases, aneurysm, arterio-venous malformations (AVM) b. venous infarction, AVM, hypertensive diseases c. aneurysm, hypertensive disease, AVM

a. hypertensive diseases, aneurysm, arterio-venous malformations (AVM)

What is *NOT* true about chondyloma acuminata a. involve usually vulva, perineal region, portio vaginalis and vagina b. is caused by HPV 6 and 11 c. is not sexually transmitted d. koilocytes are usually present

a. involve usually vulva, perineal region, portio vaginalis and vagina (tror denne er rettet feil c. is not sexually transmitted RW: I think A is not a correct answer for this question. It does involve vulva, etc. This benign squamous cell papilloma is caused by human papillomavirus (HPV), most frequently the low-risk types 6 and 11. Transmission is by sexual contact. Clinical manifestations include multiple wart-like lesions, venereal warts, in the *vulvo vaginal and perianal regions and sometimes on the cervix.* Histologic characteristics include koilocytes, or expanded epithelial cells with perinuclear clearing.

Hemolytic disease of the newborn a. is caused by incompitability in the ABO system between mother and fetus b. the result is fetal microcytic anemia c. develops in Rh+ fetus and Rh- mother d. develops in Rh- fetus and Rh+ mother

a. is caused by incompitability in the ABO system between mother and fetus c. develops in Rh+ fetus and Rh- mother

Eclampsia a. is characterized by cramps and nephrotic syndrome b. is characterized by cramps, hypertention and proteinuria c. the basic pathogenetic mechanism is in incompitability in the ABO system d. the manifestation is due to organ changes in DIC

a. is characterized by cramps and nephrotic syndrome b. is characterized by cramps, hypertention and proteinuria

Lichen sclerosis et atrophicus a. is most common in young girls b. might be associated with hyperplasia of squamous epithelium c. leads to carcinoma in all cases d. there is usually atrophy of squamous epithelium, sclerosis, and lymphocytic infiltrate in dermis

a. is most common in young girls d. there is usually atrophy of squamous epithelium, sclerosis, and lymphocytic infiltrate in dermis RW: Lichen sclerosus is an inflammatory disease characterized by pruritus and dyspareunia It grossly manifests with white plaques and atrophic skin with a "parchment paper" quality. . Microscopically there is hyperkeratosis, epithelial thinning with flattening of the rete pegs, and a band of chronic inflammatory cells. Squamous cell carcinoma develops in a minority of patients ( ̃15%). RW: bimodal age distribution (wiki)/pathoma-postmenopausal

59. Mesalazine=5-aminosalicylic acid:

a. is used to treat inflammatory bowel disease.

Cervical intraepithelial neoplasia (CIN) a. is usually associated with HPV infections b. CIN III might lead to distant metastases c. flat high grade CIN usually contains high risk HPV d. CIN II never progress into carcinoma

a. is usually associated with HPV infections The squamocolumnar junction is most frequently involved. 2. There is a major association with HPV infection types 16, 18, 31, or 33. 3. Disordered epithelial growth manifested by loss of polarity and nuclear hyperchromasia, beginning at the basal layer and extending outward, is characteristic. 4. Dysplasia can progress through mild, moderate, and severe forms to carcinoma in situ and is classified as cervical intraepithelial neoplasia (CIN), with subtypes of CIN 1, CIN 2, or CIN 3, depending on the extent of epithelial involvement. CIN 3 (carcinoma in situ) is characterized by atypical changes extending through the entire thickness of the epithelium.

81. Pyridostigmine:

a. like neostigmine. b. Medium-duration anticholinesterases!! c. NOTE: Neostigmine and pyridostigmine tend to affect neuromuscular transmission more than the autonomic system, whereas physostigmine and organophosphates show the reverse pattern. d. Also, SOS: neostigmine is a hydrophobic molecule, whereas physostigmine is a lipophilic molecule and therefore ONLY physostigmine can crosss the BBB!!!

72. Oxycodone:

a. morpine like agonist available as a slow-release oral preparation. b. Used for acute and chronic pain. c. Unfortunately, it has become popular among opioid addicts to grind up and inject such tablets as they contain large amounts of the drug. So it has become a major drug of abuse, sometimes referred to as 'hillbilly heroin'.

What is true about ovarian tumors? a. often seen cancers are serous cystadenocarcinomas b. yolk sac tumors are seen in ovaries more often than in testis c. the cystic benign teratomas are more often seen in woman than in men d. granulosa cell tumor might evoke cystic endometrial hyperplasia

a. often seen cancers are serous cystadenocarcinomas c. the cystic benign teratomas are more often seen in woman than in men d. granulosa cell tumor might evoke cystic endometrial hyperplasia

60. Methylphenidate:

a. psychostimulant used in ADHD.

86. Ropinirol:

a. selective Dopamine agonist!! b. Used if levodopa is inadequate to treat PD or if AE from levodopa are too high.

Lesions of lateral column causes a. spastic b. cerebellar sign

a. spastic (UMN) b. cerebellar sign Document found online: Lateral column syndrome --> • spastic paraparesis of the lower limbs • cerebellar signs The same is from our pp, so I guess this is correct :)

Leasion of the lateral columns leads to: a. spastic paraparesis (of lower limbs + cerebellar signs) b. loss of deep sensation c. loss of pain and temperature sensation d. muscle wasting (atrophy) on the lower extremities

a. spastic paraparesis (of lower limbs + cerebellar signs) (RW im just going w this one) Pp from Kellerova: Lateral column syndrome (spastic): o *Spastic paraparesis of the lower limbs* o *Cerebellar signs* Correction: c. should also be correct here, that is the first thing to be compromised in lateral column compression (according to the neurobook).

What is Pregabalin?

a. structural analogue of GABA.

In lumbar disk protrusion (or prolapse) a. the most common sites are L4/5 and L5/S1 disc leasions b. lateral disc protrusion compresses nerve roots c. when central disc protrusion compresses the cauda equina, symptoms and signs are usually bilateral, with sphinctter paralysis, and sensory loss in perineal and sacral areas d. compression of the cauda equina from a central disc protrusion constitutes a neurosurgical emergency (within 24h!)

a. the most common sites are L4/5 and L5/S1 disc leasions b. lateral disc protrusion compresses nerve roots c. when central disc protrusion compresses the cauda equina, symptoms and signs are usually bilateral, with sphinctter paralysis, and sensory loss in perineal and sacral areas d. compression of the cauda equina from a central disc protrusion constitutes a neurosurgical emergency (within 24h!) OBS! Is c) correct? Cauda equina is mostly associated with unilateral and asymmetric symptoms..

What is true about uterine tumors? a. the most common tumors are leiomyoma and leiomyosarcoma b. adenomas are often seen in the cervix c. different types of tumors are seen in cervix and corpus uteri d. mixed mullerian tumors belong to very malignant tumors

a. the most common tumors are leiomyoma and leiomyosarcoma c. different types of tumors are seen in cervix and corpus uteri

69. Nicergolin: (internet..)

a. treating senile dementia..

96. Tizanidine:

a. α2 adrenoceptor agonist. b. Relieves spasticity associated with multiple sclerosis and spinal cord injury!!

Metphenidate can be used for?

aka: Methylphenidate (Ritalin) *ADHD* Major depressive disorder Bipolar disorder (treatment resistant patients) *Narcolepsy* It is a psychostimulant drug (students..)

Catatonic disorders include which of these 1. Flexibilitas cerea 2. Stupor 3. Echomatism 4. Hyperkinesia

all (RW: presumably this means catanonic schizophrenia)

Vascular dementia: - CADASIL: CADASIL (cerebral autosomal dominant arteriopathy with subcortical infarcts and leukoencephalopathy) - Most common form of hereditary stroke disorder - Mutations of the Notch 3 gene on chromosome 19. - Migrines with aura, repeated strokes and subcortical dementia

all is true

Select the correct statements: - Megalomanické delusions are present in mania and depression - Delusions of formal thinking - Synonym for bizarre delusion delusion is discordant with the mood - Antipsychotic medication occupying dopamine receptors only mezolimbické and nigrostriatal path, not tuberoinfundibulární

all wrong (VILDE) -> from pp: Depression sy: micromanic delusions Manic sy: megalomanic delusions

What is used for allergic rhinitis?

alpha 1 mimetic antihistamins (Ipratropium bromide - non-allergic rhinorrhoea)

What are Hirano bodies?

are intracellular aggregates of actin and actin-associated proteins first observed in neurons; Alzheimer's disease and Creutzfeldt-Jakob disease or as a function of age without obvious underlying neurodegeration

Choose correct statement: a) Central paralysis may occur in Brown-Sequard syndrome in the muscle innervated by motor neurons at the level of the lesion b) Resting tremor is the clinical sign of dysfunction of the basal ganglia c) Spinal cord hemisection (Brown-Sequard syndrome) leads to paralysis of the contralateral upper motor neuron below the level of lesion d) Central paralysis may manifest in Brown-Sequard syndrome in the muscles innervated by motorneurons under the level of lesion

b -true d -true a - false (peripheral at the level of lesion, ipsilaterally,central below)

Which of the following is correct a) Midazolam is among SNRI b) Benzodiazepins have anxiolytic effect c) As an anxiolytic we could use donepezil d) Zolpidem provokes extrapyramydal symptoms

b) Benzodiazepins have anxiolytic effect RW: midozaolam - Benzo Donepezil - AchEI Zolpidem - insomnia GABA agonist

7. Choose the right statements a) Benzalkonium is used to improve the viability os spermatozoa b) Benzydamine is used mainly it treating inflammation suspension dental apparatus (dental prothesis) c) Povidone is used to medically induce tears production d) In the treatment of macular degeneration can be used latanoprost

b) Benzydamine is used mainly it treating inflammation suspension dental apparatus (dental prothesis) - (NSAID) c) Povidone is used to medically induce tears production --> it is tear replacement, it does not cause production?? Isn't povidone the czech version of povidone iodine, which is an antiseptic? TH (Latanoprost is used in glaucoma!)

Bilateral stem lesion a) Hemiparesis on same side b) Hemiparesis on opposite side c) Cranial nerve palsy on same side d) Loss of sensation

b) Hemiparesis on opposite side c) Cranial nerve palsy on same side (RW: agree with synne - including sensation) Bilateral stem lesion means the complete lesion of brainstem, so maybe "loss of sensation" should be included as well? Synne: I think sensations should be correct as well. Another thing; if we have complete lesion, wouldn't you be paralyzed on both sides as well? Ida E: enig med synnis, burde det ikke være begge sider? Ida: enig i at bare d) er riktig, for resten skulle vel vært bilateralt

From the following choose the correct ones a) In the cauda equina affection the spinal automatism may be present b) In expantion of extramedullary process patient may have radicular pain and vertebral syndrome c) In the acute transversal lesion of the spinal cord at the chest level the patient has pseudioflaccid paraplegia and anaesthesia under the lesion d) spinal cord lesion can causes central, peripheral or mixed paralysis of limbs

b) In expantion of extramedullary process patient may have radicular pain and vertebral syndrome (contraction and something else..) - TRUE c) In the acute transversal lesion of the spinal cord at the chest level the patient has pseudioflaccid paraplegia and anaesthesia under the lesion - TRUE d) spinal cord lesion can causes central, peripheral or mixed paralysis of limbs - TRUE

Choose the correct statement a) Inflammatory infiltrate in acute bacterial meningoencephalitis is composed primarily of modified histiocytes b) Prion diseases are characterized by the presence of tiny vacuoles in the neuropil c) Anterior acute poliomyelitis is induced by herpesvirus d) Morphological manifestation of neurosyphilis can be the atrophy of cortex rubra

b) Prion diseases are characterized by the presence of tiny vacuoles in the neuropil d) Morphological manifestation of neurosyphilis can be the atrophy of cortex rubra *From pp:* Neurosyphilis • Gumma • Atrophia rubra cortices (progressive paralysis) • Demyelinisation of spinal posterior fascicles

8. Antibodies in myasthenia gravis are against : a) acetylcholine b) acetylcholine receptors c) presynaptic receptors d) acetylcholine esterase

b) acetylcholine receptors (postsynaptic)

a) treatment of stateoacoustic neurinoma is exclusively conservative b) condition for cochlear implantation is a functional vestibulocochlear nerve c) Menieres disease is a diseaes of the middle ear d) sudden hearing loss treatment, headphones can be used

b) condition for cochlear implantation is a functional vestibulocochlear nerve (true)

Which are true: a) damage to wernicke is expressive aphasia b) ncl. ruber is a part of mesencephalon c) telencephalon has four lobes d) medial side of the thalamus forms wall of 3rd ventricle

b) ncl. ruber is a part of mesencephalon c) telencephalon has four lobes d) medial side of the thalamus forms wall of 3rd ventricle Brocas is expressive aphasia.

11. Contraindication to lumbar punction is : a) fever b) pappiledema c) suspicion of brain abscess d) mental retardation

b) pappiledema c) suspicion of brain abscess Makes sense! Main contraindication is increased intracranial pressure, and you can have that in both of these. (They are relative contraindications!) http://emedicine.medscape.com/article/80773-overview#a4

4. For acute purulent meningitis is typical : a) increased level of sugar b) substantially increased no. of polynuclears c) increased level of proteins d) substantially increased no. of lymphocytes

b) substantially increased no. of polynuclears c) increased level of proteins Decreased levels of sugar

a) chronic otitis media needs conservative treatment b) tympanometric B curve represents middle ear liquid c) basic hearing exam method is threshold tone audiometry d) lound speech has the intensity of 40-60dB

b) tympanometric B curve represents middle ear liquid c) basic hearing exam method is threshold tone audiometry d) loud speech has the intensity of 40-60dB I think D is wrong! Synne agrees (other test says 60-80db as well) A normal voice approximates to sound pressure level 70 dB A raised voice approximates to sound pressure level 76 dB A very loud voice approximates to sound pressure level 82 dB A shouting voice approximates to sound pressure level 88 dB

a) in glaucoma treatment, beta mimetics are used b) latanoprost is used in glaucoma treatment c) homatropine is used as mydriatic d) hydroxypropofol is used as fake tears

b,c

What are indications to give Acidum Acetylsalicylicum?

b. Indication: i. Acute therapy for TIA, preventing relapses which are relatively common otherwise. c. In secondary prevention of stroke and TIA aspirin is given in dose of about 100mg-SOS. (cf. With normal aspirin doses of 1 g!) d. Must be given within 48 hours and i.v.

How is the function of Tolperison?

b. Function: i. Acting by hyperpolarization and ii. Agonist at α2 receptors. iii. Blocks voltage-gated Calcium channels in the afferent fibers of the pain pathway.

What is the function of Pregabalin?

b. Function: i. Pregabalin and Gabapentine block Ca2+ influx decreasing Ca2+ intracellulary. NO release of transmitter, no GABA receptor affected!! c. Check Gabapentine. d. Indications: i. Neuropathic pain. ii. Adjunctive treatment of epilepsy. iii. GAD. e. Not metabolized, unaffected by liver failure!! f. Excreted in urine. g. Given orally for long-term treatment/control of seizures. h. NOT for monotherapy, so belongs to the 10% group of people that are treated with more than 2 drugs!

What is the function of Valproate?

b. Functions: i. Na+ channel blocker. ii. Decreases breakdown of GABA . iii. increases sensitivity of GABA receptors!!

What is the function of Gabapentin?

b. Functions: i. Na+ channel blocker. ii. Pregabalin and Gabapentine block Ca2+ influx decreasing Ca2+ intracellulary. NO release of transmitter, no GABA receptor affected!!

What is the function of Topiramate?

b. Functions: i. Na+ channel blocker. ii. kainate, AMPA receptor blocker. c. MONOTHERAPY only!!

What are indications for use of Chlorpromazine?

b. Indication: i. Schizophrenia and other psychoses. ii. Mania iii. Short-term adjunctive management of: a. Severe anxiety. b. Psychomotor agitation. c. Excitement. d. Violent/dangerously impulsive behaviour. iv. Intractabel hiccup. c. It is VERY SEDATIVE. Cf. With haloperidol!! THINK: From BZPs, zolpidem/zaleplone and Chlorpromazine only the 2 latter are useful in AD, since BZPs affect memory!!

What are the indications for use of Venlafaxin?

b. Indications: i. Antidepressant. ii. Neuropathic pain.

What are the indication for Flunarizine use?

b. Indications: i. Brain vasodilator-nootropic!! ii. Prophylaxis of migraine, vertigo. iii. Adjuvant therapy in epilepsy.

Which of ovarian tumors is of metastatic origin? a. Brenner tumor b. Krukenberg tumor c. gestational choricarcinoma d. immature teratoma

b. Krukenberg tumor c. gestational choricarcinoma RR: (......) long discussion with RW :D - c) usually metastasise to lungs (ø ovaries) --> maybe just b)?????

What is true about inflammation of the testis? a. acute purulent orchitis is of tuberculous origin b. acute nonpurulent orchitis might be caused by the parotitis epidemica virus c. acute orchitis is the regular sing of Neisseria gonorrhoea infection d. chronic orchitis might be followed by male sterility

b. acute nonpurulent orchitis might be caused by the parotitis epidemica virus RR: d. chronic orchitis might be followed by male sterility -- should also be correct

Ectopic pregnancy: a. extrauterine pregnancy is synonymous with this condition b. extrauterine pregnancy is a type of this condition c. the frequent localization is in the fallopian tube d. always develop on the base of endometriosis

b. extrauterine pregnancy is a type of this condition c. the frequent localization is in the fallopian tube RR: develops on the basis of pelvic inflammatory disease or endometriosis (but NOT Always)

The clinical manifestation of epilepsy is not: a. epileptic seizures b. headache c. psychic disturbances d. problems with concentration

b. headache

In the unilateral brain stem lesion a. hemiparesis (or hemiplegia) occurs on the same side b. hemiparesis occurs on the opposite side c. cranial nerve palsy is on the same side d. loss of sensation (hemihypestesia) is on the opposite side of the body

b. hemiparesis occurs on the opposite side c. cranial nerve palsy is on the same side d. loss of sensation (hemihypestesia) is on the opposite side of the body

For viral meningitis, the typical CSF finding is a. increased level of sugar b. increased number of lymphocytes or monocytes c. increased number of polynuclears d. glucose consentration is depressed

b. increased number of lymphocytes or monocytes (glucose is normal in viral infections) Appearance: Clear Opening pressure: Normal or elevated WBC count: 10-1000 cells/µL (lymphocytes) Glucose level: >60% serum glucose (may be low in HSV infection) Protein level: Elevated (>50 mg/dL)

The inborn respiratory distress syndrome (IRDS) a. is caused by increased production of surfactant factor b. is evoked by increased surface tension in the alveoli c. increased surface tension maintains atelectasis d. alveolar hyaline membranes are not part of IRDS

b. is evoked by increased surface tension in the alveoli c. increased surface tension maintains atelectasis

Carcinoma of the cervix a. occurs only in postmenopausal women b. is most often of squamous cell type c. grows locally and never metastatize d. cytologic screening in sexually active women can prevent development of this cancer

b. is most often of squamous cell type d. cytologic screening in sexually active women can prevent development of this cancer

Carcinoma of the prostate: a. affects men between 40 and 50 years b. metastatic dissemination is usually to bones c. develops on the base of prostatic hyperplasia d. originates in peripheral parts of the prostate

b. metastatic dissemination is usually to bones d. originates in peripheral parts of the prostate (late symptoms --> late detection)

What is true about tumors of the testis? a. teratomas belong to the most malignant tumors b. the Sertolli cell tumors does not belong to germ cell tumors c. the malignant germ cell tumors affect men between 40 and 50 years d. the choriocarcinoma is a tumor that never appear in men

b. the Sertolli cell tumors does not belong to germ cell tumors

What is true about hydatiform mole? a. there are three types - complete, partial, and metastazing b. there are three types - complete, partial, and invasive c. partial mole is diploid, and complete mole is triploid d. gestational choriocarcinoma develops in 2% of complete moles

b. there are three types - complete, partial, and invasive d. gestational choriocarcinoma develops in 2% of complete moles

Choose the right a) in the cerebrospinal fluid of multiple sclerosis patients we detect characteristic polyclonal Igs in the acidic range b) burned down plaque in Multiple Sclerosis consists mainly oligodendrocytes c) ADEM is an acute demyelinating disease that is often related to viral infection d) prevalence of multiple sclerosis in the Czech Republic about 1 / million

c) ADEM is an acute demyelinating disease that is often related to viral infection - TRUE A - false, should be oligoclonal MS is 1:1000 And the plaques probably consist mostly of astrocytes. K: active plaque consists of: activated mononuclear cells, including lymphocytes, microglia, and macrophages destroy myelin and, to a variable degree, oligodendrocytes. With time, gliosis develops, and plaques reach a burned-out stage consisting of demyelinated axons traversing glial scar tissue...

Early symptoms of MS: a) Muscle atrophy b) Ptosis c) Blurred vision in 1 eye

c) Blurred vision in 1 eye The two other are typical for myasthenia gravis

Which of the following is true a) adjustment disorder is a reaction for example after assault, rape or a car accident b) PTSD develops in every person after some life depressing traumatic events c) characteristics of OCD is repetitive occuring obsessive thoughts or compulsive actions d) occurence of a panic attack is forseeable and you can prevent it by following certain warning signs

c) Characteristics of OCD is repetitive occuring obsessive thoughts or compulsive actions *Adjustment disorders* • A life change that you need to adapt too (*not a traumatic event!*) • Abnormal reaction (in form or content), developing within 1 month after a stressful life event (not of unusual or catastrophic nature); normal adaptation unsuccessful (separation, loss, migration, physical illness, change of roles- school, job, retirement, marriage..)

3. Which auxiliary method supports the diagnostis of myasthenia gravis: a) CT of the head b) MRI of the brain and the spinal cord c) Electromyography d) Cerebrospinal fluid examination

c) Electromyography Not CT of HEAD but CT of chest! :) Oxford handbook: 1. Antibodies for ACh 2. EMG 3. CT of thymus 4. Administerin ice onto affected lid for >2 min, ptosis should improve by >2mm

From the following chose the correct one: a) brain atrophy occurs in intracranial hypertension b) Brainstem death can be confirmed by flat EEG c) Hypoglycemic coma develops rapidly d) Hyperglycemia can cause brain swelling

c) Hypoglycemic coma develops rapidly - I think B should be correct-N

1. Early symptoms/signs of multiple sclerosis include: a) muscle atrophy and fasciculations b) ptosis c) blurred vision on one eye d) facial palsy

c) blurred vision on one eye

4. Treatment option in multiple sclerosis: a) vasoactive and nootropic drugs b) acetylcholinesterase inhibitors c) methylprednisolon or immunosuppressive therapy d) no appropriate therapy exists except vitamins B and E

c) methylprednisolon or immunosuppressive therapy Oxford: 1. steroids 2. Interferons 3. Monocloncal Ab 4. Non-immunosuppressives - Glatiramer, Mitoxantrone 5. other drugs - Azathioprine

Which of the following options applies: a) To pathophysiological processes of neurodegenerative diseases does *not* apply abnormal formation of protein molecules b) Disturbances in protein degradation associated with ubiquitin system are pathognomonic for disorders of motor neuron c) pathophysiological processes of neurodegenerative disorders is attributed to oxygen free radicals d) between peptides which are involved in the pathophysiology of neurodegeneration does not belong prion protein

c) pathophysiological processes of neurodegenerative disorders is attributed to oxygen free radicals If the statement indicates that neurodegeneration is ONLY attributed to oxygen free radicals then this statement is false, depends how you read it. TH

What are the adverse effects of Selgiline?

c. AE: • Autonomic effect: a. Postural hypotension. • CNS effect: a. Agitation. b. Insomnia. c. Hallucinations. • Confusion due to hyponatraemia. • Nausea

What are the adverse effects of Vigabatrin?

c. AE: • Visual field defects. d. Used in combination with other anticonvulsants, not for monotherapy! e. Careful in renal failur, because it is excreted by the kidneys.

What are the adverse effects of Risperidone?

c. AE: • Weight gain. • Significant risk of EPS. d. Best effect in hallucinations but it shows a small increase in risk for stroke?!

What are common adverse effects of Tolperison?

c. Common AE: • Somnolence. • Sedation.

What are the indications for use of Gabapentin?

c. Indications: i. Adjunctive treatment of epilepsy. ii. Treatment of neuropathic pain. iii. GAD. d. NOT for acute control of seizures, see BZP. e. Like PREGABALIN, absorption from the intestine depends on the L-amino acid carrier system and shows the property of SATURABILITY-increasing the dose does NOT proportionately increase the amount absorbed. This makes it a safe drug. f. It is NOT metabolized, but excreted in urine unchanged. g. Only 10% of pts require 2 or more drugs for epilepsy control!

What are the indications to give Sumatripan?

c. Indications: i. Cluster headache. ii. Migraine.

What are indications for use of Haloperidol?

c. Indications: i. Schizophrenia and other psychoses. ii. It has a sedative effect, so for short-term management of psychomotor agitation, excitement, violent/dangerously impulsive behaviour. iii. Intractable hiccup.

What are the indications for Frovatriptan use?

c. Indications: i. Treatment(not prophylaxis) of acute migraine!! (After failure of analgesia.)

Fluoxetine is used to treat?

c. Treatment of: i. Depression. ii. OCD. iii. Bulimia nervosa. iv. GAD and PTSD. d. NOTE: Fluoxetine has a SLOWER ONSET of action than the other SSRIs but its LONG half-life is an advantage in pts with problematic compliance!!

Early symptoms/signs of multiple sclerosis include: a. muscle atrophy and fasciculations b. ptosis c. blurred vision on one eye d. facial palsy

c. blurred vision on one eye

Malformations a. the majority of them have AD trait b. are more often seen in males c. develop due to disorders of main morphogenetic mechanisms d. are congenital, but not always genetically mediated

c. develop due to disorders of main morphogenetic mechanisms d. are congenital, but not always genetically mediated (3 major categories of causes are genetic, environmental and multifactorial)

The Lassegue sign can be caused in addition to the lumbar disk herniation also by a: a. metastasis in femur b. paresis of the thigh flexors c. inflammatory radiculitis d. hypertrophy of prostate e. sciatica

c. inflammatory radiculitis e. sciatica Also seen as a meningeal sign?

Polycystic ovaries - Stein-Leventhal syndrome a. is associated with prononced metrorrhagia b. in ovaries are multiple corpus-luteal cysts c. it appears in young girls with hirsutism, sterility and obesity d. the cause is adenoma of hypophysis

c. it appears in young girls with hirsutism, sterility and obesity

56. Levodopa plus benserazide(des panw) =

cobeneldopa

Lumbar disc protrusion

correct answer was a,c,d (???) Typically seen in lumbar enlargement, needs surgery within 24h.

a) positive and negative schizophrenia symptoms are the result of BDNF defect b) Delusion is a conviction that doesnt have influence on behaviour c) a patient that thinks that his thoughts have been broadcasted on tv for a few months has mania d) Bleulers 4A: autism, blunted affect, ambivalence, loss of association

d) Bleulers 4A: autism, blunted affect, ambivalence, loss of association Klara: I would say that loss of association is wrong - since association stands for deficit of/loose associations...

7. A typical haemorrhage in hypertensive patient is localised in : a) temporal gyrus b) cerebellum c) int. and ext. capsule d) predominantly in the cerebral ventricles

d) Predominantly in the cerebral ventricles Why? Ventricular bleeding is rare.. Pathology pp says: capsula interna + basal ganglia (Marte er enig) RW: agree that it should be int/ext capsule - would say C not D) -- RR agree

Choose the right statements a) Damage of subthalamic nuclei leads to Parkinson disease b) Damage to subthalamic nuclei will cause Korsakoff psychosis c) Hemiparesis develops as a result of damage to the ipsilateral internal capsule d) Risk factors most frequently cited with stroke is hypertension

d) Risk factors most frequently cited with stroke is hypertension (dammage to st nuclei--> hemiballismus) rw Lesions of the genu of the internal capsule affect fibers of the corticobulbar tract. The primary motor cortex sends its axons through the posterior limb of the internal capsule. Lesions, therefore, result in a *contralateral* hemiparesis or hemiplegia.

When should you avoid giving Sumatripan?

d. Avoid if: i. Raynaud disease, TIA, stroke, uncontrolled arterial hypertension, CAD(R. 5HT1D receptors are also found in the coronary vessels.) ii. Kidney or liver failure. iii. Pregnant or lactating woman. iv. Sulfonamide component present which means it can cause hypersensitivity disorder in pt. e. Metabolized by MAO, AVOID MAOI!!

When should you avoid use of Haloperidol?

d. Avoid if: i. Urinary retention or closed-angle glaucoma due to anticholinergic effects. ii. Parkinsonian symptoms, due to anti-dopaminergic effects. iii. Hypotension. Dont give with other drugs that may lower the BP. NOTE: Although it has a HIGH risk of extrapyramidal effects it causes relatively less hypotension!! iv. Phaeochromocytoma. e. Antipsychotic drugs are MORE effective for ACUTE psychotic symptoms of schizophrenia rather than chronic symptoms!

What are the contraindications for Flumazenil use?

d. Contraindications: i. BZP controlling seizures_risk of relapse. ii. BZP dependence_withdrawal syndrome. iii. Increased intracranial pressure or if unknown cause of uncosciousness. e. Give in SMALL doses! f. Overstimulation syndrome may occur if given in excess or too rapidly via i.v. route.

When should you not give Frovatriptan?

d. Do NOT give in: i. CAD=Coronary Arterial Disease. ii. Uncontrolled hypertension. iii. Pts which are taking moclobemide or non-selective MAOI!! iv. Pts with previous TIA/ stroke. e. Give as soon as the aura begins?!(presentation at uni says otherwise) f. Subcutaneous injection is best, avoid i.v.!!!! g. It has a slower effect and better tolerability. h. AE: • Transient increase in BP. • Sensation of tingling, heat, flushing, heaviness-sense of pressure in throat/chest.

What are indications for use of Topiramate?

d. Indication: i. Prophylaxis of migraine. ii. Treatment of seizures.

Dementia before the age of 60 necessitates to exclude in differential diagnosis a. anemia b. alcoholic encephalopathy c. brain tumor d. all of the above

d. all of the above

Which of the below mentioned infections is *not* transmitted transplacentally? a. listerosis b. cytomegalovirus c. syphilis d. influenza

d. influenza

What is Bleuler's four A's in diagnosis of schizophrenia?

disturbances in affect, association, ambivalence and autism

What are the adverse effects of Topiramate?

e. AE: • Myopia. • Acute angle glaucoma.. • Mtb acidosis. • Nephrolithiasis.

What are the adverse effects of Ginkgo biloba?

f. AE: • Intracranial hemorrhages.

What are the adverse effects of Haloperidol?

f. AE: • Sedation. • Neuroleptic malignant syndrome.(R. Dantrolene to treat!!) • EPS(antidopaminergic) a. Acute dystonias. b. Akathisia, Parkinsonian symptoms. c. Tardive dyskinesia(After chronic use!) • Prolactin release causing: a. Gynecomastia. b. Galactorrhoea. c. Menstrual disturbances. • Anticholinergic effects: a. Blurred vision. b. .. • Hypersensitivity phenomena. • Cardiac arrhythmias by prolonging the QT interval!(dont give with TCA) • Increased risk of thromboembolism.

What are contraindications for use of Phenytoin?

f. Contraindications: i. Pregnancy, since it has antifolate action. ii. Porphyria.g. For EPILEPSY: i. Given ORALLY for long-term control of epilepsy. ii. i.v. when rapid control is required as in status epilepticus. iii. Non-linear kinetics of phenytoin-careful dosage required! iv. R. Phenytoin PRODRUG is fosphenytoin for i.v. administration. h. 10-20ug/ml effectve levels, with 18ug/ml highly effective level. OVER 20ug/ml is considered TOXIC level!!

- Antidepressant affects neither gene expression or neurotrophic factors

false

- Enlarged lateral ventricles are specific exclusively to schizophrenia

false

- Phineas gage was a neurosurgeon who described relationship between temporal lobe lesion and epilepsy

false

- Serotoninergic pathways are the main components of reward system

false

Compulsions and obsessions are symptoms of panic disorder?

false

unwanted intrusive throughts are typical for agarophobia?

false

- Hypochondriac delusion is a symptom of psychotic mania

false Hypochondric delusions is the idea of having a stone instead of liver. Probably schizophrenic

Creutzfeld Jakob is not neurodegenerative disorder

false (it is)

What are the adverse effects of Cloazapine?

g. AE: • Decreased BP. • Prolongation of QT interval(same as conventional antipsychotics). h. Almost no Extrapyramidal side effects.

What are the adverse effects of Gabapentin?

h. AE: • Drowsiness. • Sleepiness. • Dizziness. • Ataxia. • Tremor. i. Do NOT give with TCA or mefloquine.

What are adverse effects of Metoclopramide?

h. AE: • EP-antidopaminergic effects: a. Dystonia, oculogyric crisis_give procyclidine, benzatropine. • Prolactine release. i. R. Paracetamol and metoclopramide together!

Devic's disease or Neuromyelitis optica is what?

heterogeneous condition consisting of recurrent and simultaneous inflammation and demyelination of the optic nerve (optic neuritis) and the spinal cord (myelitis).

What are the adverse effects of L-DOPA?

i. AE: • CNS AE: a. Confusion. b. Psychiatric effects-euphoria, agitation, anxiety. c. Involuntary movements. • Peripheral AE: a. Nausea and vomiting. Take levodopa with meals!!! b. Cardiac arrhythmias. c. Hypotension. d. Difficulty with micturition. e. Incontinence. • „Sleep attacks"

What are adverse effects of Phenytoin?

i. AE: • Early: a. CNS and heart depression. b. Skin problems. c. Hypoglycemia_HONK(R.) d. Hematological. e. Acute toxicity. i. Confusion, ii. Slurred speech. iii. Blurred vision. • Late: a. Coarse facies. b. Acne. c. Hirsutism. d. Gingival hyperplasia. e. Fetal hydantoin syndrome!!

What are contraindications for use of interferon?

i. Contraindications: i. Avoid in hepatic insufficiency. ii. Pregnancy. iii. If pt has suicidal ideations. iv. If pt has poorly controlled epilepsy. j. AE: • Flu-like syndrome: a. Fever. b. Headache. c. Myalgia. d. Chills. e. Malaise. f. Nausea and vomiting. • Injection site reactions.

100. Tramadol: a. Function:

i. It is a weak agonist at µ opioid receptors and ii. also a weak inhibitor of noradrenaline reuptake. b. Analgesic for moderate to severe postoperative pain!

Indications for use of Benzalconium?

i. LOW concentrations: BacterioSTATIC. ii. HIGH concentrations: BacteriCIDAL.

Name 2 groups with typical antipsychotics?

i. Phenothiazines - chlorpromazine ii. Butyrophenones - haloperidol Chlorpromazine: low potency, sedative Haloperidol: high potency, high EPS Typical = works on positive symptoms.

When you not use Benzalconium?

i. Pseudomonas. ii. Mycobacterium. iii. Bacterial spores. iv. Many viruses.

Hypoglossal paralysis - difference in UMN and LMN lesion?

i. UMN paralysis: tongue deviates opposite of lesion when protruded ii. LMN paralysis: tongue deviates to the side of lesion when protruded

- Treatment of neuropathic pain - NSAIDs

no

Anxiety is only in panic attacks `

no

Hypoprosexie (attention deficit disorder) is decreased libido?

no

Is hypoxia the same as ischemia?

no

Severe rotation of eye can lead to alexia?

no

Bulbar syndrome causes a central paralysis of hypoglossal nerve?

no Bulbar is LMN lesion

Strongest indication of suicidal tendencies in depressed people is psychosis?

no (?) RW: Risks from ppt (psychosis not mentioned): Risk of suicide assessment * S sex: male * A age: >45, <19 * D depression * P previous attempts* E ethanol abuse * R rational thinking loss (psychosis?)* S social suppot lacking * O organized plan * N no spouse * S sickness (somatic illness with pain)

OCD persons have voices commenting on their actions?

no (usually not)

Prevalence of depression is 40% ?

no 20%

CT myelography is mainly today use in cervical spine?

no ??? (RW: should be lumbar) S+E believes this should correct. VF - "CT myelography involves a lumbar or cervical spine puncture with contrast injected into the spinal canal, followed by CT..." --> can be cervical as well but generally lumbar is more common

For allergic runny nose we used alpha 2 mimetics?

no, alpha 1

Acute anterior poliomyelitis is caused by herpes virus?

no, by polio virus

Chlorhexidine has strong antiviral and antifungal effect?

no, weak Is bactericidic

Prophylactic treatment migraine o Beta blockers o Anti-convulsants o Andi-depressants o Calcium channel blockers o Beta 2 mimetics o Alpha 1 agonists

o Beta blockers - yes o Anti-convulsants - yes o Andi-depressants - yes o Calcium channel blockers - yes o Beta 2 mimetics - NO (it's a broncho- and vasodilator) o Alpha 1 agonists - NO (triptans work on serotonin 1D and 1B receptors that are ONLY located in the cerebral blood vessels, so they're specific. So you can't use alpha 1 agonists because they don't work specifically on these receptors)

Sumatriptan - is given i.v - constricts brain vessels - cause pain relief during migraine attack

o Constriction of brain vessels - Yes (vasoconstrictor) o IV administration - No (tablet, nasal spray, subcut-injection) o Pain relief during migraine attack - naratriptan - Yes

Which of these are used as artificial tears? o Hyaluronic acid o Povidone

o Povidone

What is the standard therapy of tinnitus? o Use of beta blockers o Vespta (?) o Histamine, beta-histine o Adverse effects: accessive weight loss

o Use of beta blockers - no o Vespta (?) o Histamine, beta-histine - stimulates receptors - yes --> H3 histamine receptors o Adverse effects: accessive weight loss (no, it's gain!)

Retrograde amnesia

problem with consolidation

Common sites of berry aneurysms

r. communicans anterior 40% a.cerebri media and it´s branches 34% a. carotis interna 20%

Drugs used in macular degeneration?

ranibizumab, Anecortav, pegaptanib, Verteporfin

Benzodiazepines in elderly increases their risk of falling?

yes

- Anticipatory anxiety is a maintaining factor in a panic disorder

true

- Anxiety in the panic disorder and the generalized anxiety disorder is not triggered by specific situations -

true

- Bipolar disorders share numerous genetic abnormalities with schizophrenia

true

- Dizziness, paraesthesias and cramps during a panic attack are result of hyperventilation

true

- Feeling of anxiety is always accompanied by somatic symptoms of vegetative arousal

true

- New emotional experience is processed through amygdala

true

- The core symptoms of depressions is depressed mood more than 2 weeks, loss of interest and enjoyment, and reduced energy and activity

true

Bleulers schizophrenia is what Kraepelin called Dementia Preacox, true or false?

true

Cerebellar gyri are called folia, true or false?

true

a) muscle atrophy and fascicultions occur early in the first week after the lesion b) in central spastic paresis the affected extremity has higher mm tone, hyperreflexia, positive pyramidal signs and mm weakness c) in peripheral paresis the affected extremity is weak, hypotonic and hyperreflexic d) Pseudflaccid paralysis' central paraplegia (spinal shock stage) you have hypotonia of mm and areflexia

true: A,B,D Does really atrophy and fasciculations start in the 1st week? According to the latest test atrophy and fasciulations DO NOT occur within the 1st week (so A is false). TH

Basic examination of orientation is asking about time and time of year `?

yes

Benzalconium can be used in mouth wash?

yes

- Treatment of neuropathic pain - carbamazepine

yes

15% of depressed people comit suicide?

yes

Anhedonia is the inability to feel joy

yes

Are inclusions in neurodegenerative disorders present in oligodendroglial cells as well?

yes

BAD - the depression are more frq than the manic episodes?

yes

BZD works on GABA receptors?

yes

A combination of aspirin and pyramidol can be used in post-stroke treatment? If no, why not? If yes, why?

yes --> Coronary vasodilation and decreased adhesion to the endothelium

ADEM is often connected with infection?

yes Acute disseminated encephalomyelitis

Subdural hematoma can lead to papillary edema?

yes Also ICP can lead to it!

Lesions of temporal lobe leads to derealization?

yes Derealization or derealisation (sometimes abbreviated as DR) is an alteration in the perception or experience of the external world so that it seems unreal.

Friedreich ataxia is related to frataxin gene?

yes Friedreich's ataxia is a rare inherited disease that causes nervous system damage and movement problems. It usually begins in childhood and leads to impaired muscle coordination (ataxia) that worsens over time.

Bells palsy is due to VII nerve paralysis?

yes Paralysis of the facial nerve causing muscular weakness in one side of the face.

Social withdrawal and affective blunting are typical signs of schizophrenia?

yes Reduced affect display, sometimes referred to as *emotional blunting*, is a condition of reduced emotional reactivity in an individual

around 80% of drugs sold as ecstasy are in fact other types of drugs ?

yes Sick..

Olfactory dysfunction or hallucinations in one nostrill can be caused by a onesided tumor?

yes Tumors in uncus or olfactory groove meningioma

Tactile sensation can be damaged with a posterior column lesion ?

yes Vibration, position sense and fine touch tactile = touch

Chlorhexidine can be used to remove plaque on teeth? What is an adverse effects?

yes Brown discoloration of teeth

Progressive multifocal leukoencephalopathy (PML) can be present in people with HIV/Aids ?

yes Commonly found in immunocompromised patients

If the 12th CN is damaged, the tongue will deviated to the same side of the lesion, when it is protruded?

yes Protrusion --> deviation towards paralyzed side Inside the mouth --> deviates toward healthy side - peripheral lesion

Hypoventilation occurs if a patient has increased intracranial pressure?

yes RW: Disagree"Hyperventilation is one known method of rapidly lowering ICP. Cerebral blood flow is largely dependent on PaCO2. Hyperventilation causes decreased PaCO2 which subsequently leads to arterial vasoconstriction thus lowering cerebral blood flow (CBF), cerebral blood volume, and ICP. This effect is mediated my pH changes in the extracellular fluid which cause cerebral vasoconstriction or vasodilation depending on the pH. There is approximately a 2% decrease in CBF for every 1 mmHg decrease in PaCO2 levels: - but if they are compressing the breathing centers - that is a different story. RR: I agree with the hyperventilation part -- but for treatment. So It should be correct if the breathing centres are being compressed...

Is Tropicamide a mydriatic agent?

yes RW: Shorter-acting muscarinic-receptor antagonists (tropicamide) that produce mydriasis are administered topically as eyedrops or as ointments for refractive measurements and for ophthalmoscopic examination of the retina and other structures of the eye (α-adrenoceptor agonists, such as phenylephrine, are used for simple funduscopic examination without cycloplegia)

GAD increases morbidity for cardivascular disease?

yes (GAD=generalized anxiety disorder)

Does peripheral lesions of VII lead to lagophtalmus? (Inability to close the eye)

yes (NB: not in central - because the upper part of face is innervated bilateralt)

Conversive disorder is a self defence mechanism that a psychological problem moves to a static part of the body?

yes (RW: No, this is somatization disorder? dont really under the english?) (RW: do they mean conversion disorder? * loss of normal integrity (dissociation) of memories, one's own identity, immediate feelings and control of bodily movements * associated with unsolved problems, disturbed relationships affect is transformed into symptoms (hysterical conversion)

Spinal shock leads to removal of all reflexes for a certain time?

yes (all spinal reflexes but not brainstem reflexes?)

Schizophrenia research is focused on posterior cingulate?

yes (some of it at least, related to Toxoplasmosis infection) Other regions showing changes in schizophrenia are: • Temporolimbic regions • Thalamus • Prefrontal cortex • Mesolimbic pathway and Mesocortical pathway

Pick's disease - atrophy of frontal cortex?

yes and temporal cortex RW: Picks Disease: marked Cerebral atrophy with gliosis and loss of cortical neurons, especially affecting temporal and frontal lobes; Pick bodies (round intracytoplasmic inclusions consisting of neurofilaments.) within some neurons, especially in the horn of Ammon; clinically resembles ALZ, more frequent in women

Irritational signs (those that appear when u have a lesion of a central neuron) can be present normally in children untill first year?

yes, like the babinsky

Which receptor does LSD work on?

• Agonist 5-HTR (1a>2a>1d>2c>7) • Agonist DA receptors (D3 > D4 > D2)

What are the adverse effects of Ethosuximide?

• Blood dyscrasias and other Hematopoietic complications. • GIT disturbance. • Nausea and anorexia!!

What are the adverse effects of Dentrolene?

• Can cause severe liver toxicity. Avoid extravasation because it can cause severe local tissue damage!

Which neurotransmitter is important in craving?

• For drug seeking and craving is important DA in prefrontal cortex and amygdala

What are the adverse effects of Dipyridamole?

• GIT disturbance. • Vasodilation: - Hot flushes. - Headache. - Tachycardia.

What are the adverse effects of Carbamazepin?

• Leucopenia in 6% of patients • Generalized erythematous rash. It can cause Stevens Johnson syndrome = Very severe generalized rash that extends to the GIT. Characterized by photosensitive lesions, UV converts the drug to reactive products! • SIADH. CNS effects: a. Visual symptoms. b. Drowsiness. c. Impaired balance. Increases the rate of mtb of OESTROGEN and PROGESTOGENS so it can render Oral Contraceptives ineffective!!SOS h. Q: Carbamazepine is more effective in treating absence seizures than valproate?-NO!(in test)

What are the addverse effects of Donepezil?

• Muscarinic effects: - Abdominal cramps. - BRADYCARDIA. - Sweating. - Hypesalivation. - Increased bronchial secretions. • Nicotinic effects: - Muscle cramps. • Central cholinergic effect: - Increased risk of convulsions.

What are adverse effects of Distigmine?

• Muscarinic: - Abdominal cramps. - Bradycardia. - Hypersalivation. - Sweating. - Increase in bronchial secretions - REVERSED by atropine!! • Nicotinic: - Muscle cramps. • Central cholinergic effect: - Increase risk of convulsions

Therapy of huntington disease is which drugs?

• Neuroleptics - antidopaminergic Reserpine, meserpine, tetrabenazine • Central muscle relaxants Baclofen • Drugs to decrease excitotoxicity (slows the progression of dementia)

What are the Adverse effects of Citalopram?

• The AE are common, about 57% of patients have them, but they are transient. GIT effects: - Nausea and vomiting. - Constipation. - Diarrhoea. • Headache and other CNS effects like akathisia, anxiety, tremor, insomnia. • SIADH(R. Also amitriptyline and carbamazepine share this AE) • They decrease the seizure threshold. (So do MAOI, TCA and bupropion) i. In a nutshell: SSRIs are better than TCA because they have the same effect but SSRIs are better tolerated! j. Decrease the dose in renal/hepatic insufficiency. (or avoid altogether)

Signs of pseudobulbar syndrome (supranuclear) ?

• UMN lesion, central palsy • dysarthria, dysphagia, n.VII,V • masseter reflex increased • emotional lability (unprovoked crying or laughing) • frontal type of the gait


Ensembles d'études connexes

Corporate Accounting 2 - Chapter 23 Statement of Cash Flows

View Set

Genetics Chapter 16 Multiple Choice

View Set

Chapter 15 - The Kennedy and Johnson Years

View Set

GEB 4890 - Business Strategy Chapter 8

View Set

Database Systems, Chapter 2-3 Definitions

View Set

(-80-) Direct object pronouns (los pronombres-DOS): The correct use of me (me), te (you-object), nos (us) and los-a ustedes (you all)

View Set